Download as pdf or txt
Download as pdf or txt
You are on page 1of 62

December 2023

Vol. 44 No. 12
www.pedsinreview.org

Athlete Screening and


Sudden Cardiac Death
Ensuring Emergency
Readiness in the
Pediatric Primary
Care Setting:
An Updated Guideline
Fever: Three Patient
Cases
In Brief
Pneumocystis Pneumonia
Implications of Food and
Drug Administration
Approval of Respiratory
Syncytial Virus
Prophylactic Medication
Visual Diagnosis
Exfoliative Erythematous
Rash in an 11-day-old
Preterm Infant

Downloaded from /pediatricsinreview/issue/44/12


by Hospital de Peditría 'Prof. Dr. Juan P. Garrahan' user
Pediatrics in Review
ARTICLES ••
•••
• •



669 Athlete Screening and Sudden Cardiac Death
Robert Spencer, Shahed Quraishi
MOC



• •
••
•••

682 Ensuring Emergency Readiness in the Pediatric Primary Care Setting: An Updated Guideline
Maria Milla, Elisa Prebble, Gloria Riefkohl, Fernando Llopiz Martin, Danielle Altares Sarik, Paola Garcia-Herreros
To learn how
692 Fever: Three Patient Cases to claim MOC
692 Fever in a 40-day-old Infant points, go to:
Stelios Kasikis, Maame Hayfron, Despoina Galetaki, Risa Bochner https://publications.
aap.org/journals/
694 Ankle Pain 2 Days after a Febrile Illness in a 9-year-old Girl pages/moc-credit.
Maya Heled Akiva, Christos Karatzios, Marina I. Salvadori
697 Acute Right Lower Quadrant Abdominal Pain in a 2-year-old
Sagar D. Mehta, Carrie Ng STAY CONNECTED
701 Commentary: Fever Is Your Friend facebook.com/pedsinreview
Henry M. Adam

twitter.com/AAPJournals
INDEX OF SUSPICION
706 New-Onset Jaundice in an 11-month-old Boy
Sean E. Healton, Devon G. Lawrence, Noah J. Elkins, Leya Schwartz, Patricia A. Hametz
publications.aap.org/journals
710 Early Satiety and Nausea in a 12-year-old Girl
Alexandra S. Hudson, Amira Balbaa, Bryan J. Dicken, Matthew Carroll
713 Intermittent Fussiness in a Well-Appearing 5-week-old Girl
Elizabeth Wikle, Amani Sanchez, Arianna Nassiri, Francis Onyebuchi
716 Fatigue, Weight Loss, and Acute Chest Pain in a 15-year-old Boy
Madeline F.E. Parr, Katharine N. Clouser, Meghan Tozzi, Sejal M. Bhavsar

IN BRIEFS
720 Pneumocystis Pneumonia
Asif Noor, Leonard R. Krilov
723 Implications of Food and Drug Administration Approval of Respiratory Syncytial Virus
Prophylactic Medication
Ashley L. Saint-Fleur, Catherine Kier

VISUAL DIAGNOSIS
e33 Exfoliative Erythematous Rash in an 11-day-old Preterm Infant
Janani Rajkumar, Jamie B. Warren, Jina Park

Answer Key appears on page 719.

Pediatrics in Review® (ISSN 0191-9601) is owned and controlled by the American Academy of Pediatrics. It is published
monthly by the American Academy of Pediatrics, 345 Park Blvd., Itasca, IL 60143.
Statements and opinions expressed in Pediatrics in Review® are those of the authors and not necessarily those of the
American Academy of Pediatrics or its Committees. Recommendations included in this publication do not indicate an
exclusive course of treatment or serve as a standard of medical care.
Subscription price for 2023 for print and online/online only: AAP/CPS Member $235/$180; AAP National Affiliate Member
$185/$135; Nonmember $290/$230; AAP In-training Member $185/$135; Nonmember In-training/Allied Health $210/$155.
Institutions call for pricing (866-843-2271). For overseas delivery, add $120. Current single issue price is $22 domestic, $25
international. Replacement issues must be claimed within 6 months from the date of issue and are limited to three per
calendar year. Periodicals postage paid at ARLINGTON HEIGHTS, ILLINOIS and at additional mailing offices.
© AMERICAN ACADEMY OF PEDIATRICS, 2023. All rights reserved.
Printed in USA. No part may be duplicated or reproduced without permission of the American Academy of Pediatrics.
POSTMASTER: Send address changes to PEDIATRICS IN REVIEW®, American Academy of Pediatrics Customer Service
Center, 345 Park Blvd., Itasca, IL 60143.

Downloaded from /pediatricsinreview/issue/44/12


by Hospital de Peditría 'Prof. Dr. Juan P. Garrahan' user
ARTICLE

Athlete Screening and Sudden


Cardiac Death
Robert Spencer, MD, MS,*† Shahed Quraishi, MD*
*Division of Pediatric Cardiology, Department of Pediatrics, Staten Island University Hospital, Northwell Health, Staten Island, NY
†Donald and Barbara Zucker School of Medicine at Hofstra/Northwell, Hempstead, NY

EDUCATION GAP

Primary care providers play an instrumental role in preventing sudden


cardiac deaths in young athletes. Therefore, primary care providers should
be familiar with warning signs and risk factors of cardiac conditions that can
cause sudden death, preparticipation screening guidelines, and secondary
prevention measures.

OBJECTIVES After completing this article, readers should be able to:


1. Identify the causes of sudden cardiac death in young athletes. AUTHOR DISCLOSURE: Drs Spencer and
Quraishi have disclosed no financial
2. Describe screening guidelines and be aware of controversies in screening. relationships relevant to this article. This
commentary does not contain a
3. Recognize and promote secondary prevention measures. discussion of an unapproved/
investigative use of a commercial
product/device.

INTRODUCTION
ABBREVIATIONS
Sudden cardiac death (SCD) is defined as unexpected and abrupt death caused by a
cardiovascular condition, symptoms of which have begun within the past hour. Al- AAOCA anomalous aortic origin of a
coronary artery
though SCD is rare, each death has a significant and long-lasting effect on the vic- AAP American Academy of Pediatrics
tim's family and community. Athletes with preexisting cardiac conditions are at AC arrhythmogenic cardiomyopathy
increased risk for SCD during training and competition. (1) Conducting preparticipa- AED automated external defibrillator
AHA American Heart Association
tion physical evaluations (PPEs) of athletes is an important step in preventing SCD in ALCA anomalous left coronary artery
susceptible individuals, partly because many of the predisposing conditions would ARCA anomalous right coronary artery
otherwise not be noticeable. To ensure proper screening, primary care providers CERP Cardiac Emergency Response
Plan
should be familiar with cardiac conditions associated with SCD.
CMR cardiac magnetic resonance
Because no screening program can eliminate the risk of SCD, secondary pre- CPR cardiopulmonary resuscitation
ventive measures, such as increasing access to training in cardiopulmonary re- CPVT catecholaminergic polymorphic
ventricular tachycardia
suscitation (CPR) and automated external defibrillators (AEDs), as well as
ECG electrocardiogram
establishing emergency response plans at schools, are essential. This review pro- HCM hypertrophic cardiomyopathy
vides an overview of the epidemiology and causes of SCD in young athletes as ICD Implantable cardioverter-
defibrillator
well as preparticipation screening and secondary prevention.
LQTS long QT syndrome
LV left ventricular
EPIDEMIOLOGY PPE preparticipation physical
evaluation
SCD has an estimated incidence ranging from 1:917,000 to 1:3,000 athletes SCA sudden cardiac arrest
younger than 40 years in the United States per year. (2) Most cases occur at the SCD sudden cardiac death

Vol. 44 No. 12 D E C E M B E R 2 0 2 3 669

Downloaded from /pediatricsinreview/issue/44/12


by Hospital de Peditría 'Prof. Dr. Juan P. Garrahan' user
high school or college level, often during practice or com- Table 1. Cardiovascular Causes of Sudden Cardiac
petition. (3) The overall risk is higher in male compared Death (SCD) in Young Athletes
with female athletes, with a 9:1 ratio, and more than 3 CONDITION SCD, %a
times higher in black versus white athletes. (3)
Hypertrophic cardiomyopathy 36
Although some data suggest that SCD is more common Anomalous coronary artery 19
in nonathletes than in athletes, it is clear that intense Myocarditis 7
Arrhythmogenic cardiomyopathy 5
physical activity increases the risk of SCD in individuals Coronary artery disease 4
with predisposing cardiovascular disease. (4)(5)(6) Certain Mitral valve prolapse 4
Aortic rupture 3
sports are more strongly associated with SCD, such as
Aortic stenosis 2
football, basketball, and baseball in male athletes. In con- Dilated cardiomyopathy 2
trast, female athletes are at greater risk while participating Otherb 18
in basketball, cross-country/track, and soccer. (3) Although a
Percentages are based on 842 young athletes in the United States
speculation attributes the increased risk of SCD in these from 1980 to 2011 with confirmed cardiac causes of sudden death. (3)
b
Other conditions include (in decreasing order of frequency) left
sports to each sport’s popularity, intensity of training, and
ventricular hypertrophy of unresolved etiology, bridged left anterior
genetic predispositions of athletes, further studies are descending artery, long QT syndrome, congenital heart defect,
needed to confirm this possibility. Wolff-Parkinson-White syndrome, myocardial infarction, sarcoidosis,
stroke, conduction system abnormality, cardiac rupture, cardiac tu-
mor, tetralogy of Fallot, electrolyte abnormality, ruptured cerebral
CAUSES OF SUDDEN DEATH arteriovenous aneurysm.
Based on data from the US National Registry of Sudden
Death in Athletes from 1980 to 2011, 40% of sudden deaths
physical examination may reveal a systolic murmur that be-
in young athletes occur in the absence of a preexisting cardiac
comes louder with reduced preload (eg, Valsalva maneuver or
condition. Approximately half of these deaths are due to blunt
standing) and quieter with increased preload (eg, squatting) or
trauma (51%), with commotio cordis composing a much
afterload (eg, handgrip). A family history of sudden unex-
smaller percentage (7%). (3)
plained death is another important clue because HCM is usu-
In large autopsy-based studies of athletes in the United
States, hypertrophic cardiomyopathy (HCM) has consis- ally inherited as an autosomal dominant trait. (10)
The 12-lead electrocardiogram (ECG) is abnormal in most
tently been the most common confirmed cardiovascular
cause of sudden death, followed by coronary artery anoma- patients with HCM, with findings suggestive of left ventricular
lies. Table 1 lists the most common cardiac conditions as- (LV) hypertrophy and repolarization abnormalities (eg, T-wave
sociated with SCD. In this section, we highlight the most inversion). (10) Ambulatory ECG monitoring also provides
prominent causes of SCD. An overview of these condi- valuable information for risk stratification, as nonsustained
tions and some others is provided in Table 2. ventricular tachycardia is a risk factor for SCD. Other risk fac-
tors for SCD in patients with HCM include history of syncope
Hypertrophic Cardiomyopathy or cardiac arrest, family history of SCD, severe LV hypertrophy
HCM is a genetic disorder affecting sarcomere proteins, (in adults, LV maximal wall thickness $30 mm), and abnor-
the basic contractile unit in cardiomyocytes. As previously mal blood pressure response to exercise. (11) However, these
noted, it is the most frequently identified cause of SCD in risk factors are extrapolated from adult data, and pediatric-
young athletes in the United States, with an estimated specific risk factors and calculators are currently under
prevalence of 1:500. (7)(8) However, as of 2019, only investigation.
1:1,250 individuals are clinically diagnosed, suggesting that Echocardiography is the principal diagnostic imaging mo-
60% of affected individuals remain undiagnosed. (9) dality for HCM (Fig 1). Although diastolic LV wall thickness of
More than 900 mutations have been identified in pa- 15 mm or greater without a known underlying cause is diag-
tients with HCM, most commonly located in the genes en- nostic in adults, a body surface–adjusted z score of 2 or greater
coding b-myosin heavy chain (MYH7), cardiac myosin suggests HCM in children. (10)(12) Regardless of age, asym-
binding protein C (MYPBC3), and cardiac troponin (TNNT2). metrical septal hypertrophy is a common and distinctive fea-
The clinical course of the disease varies considerably, with ture of HCM. Cardiac magnetic resonance (CMR), which
some patients presenting in childhood and others remaining offers enhanced spatial resolution and image quality, can be
asymptomatic through adulthood. Chest pain and dyspnea on helpful in patients with diagnostic uncertainty or poor echocar-
exertion are commonly reported symptoms. In addition, the diographic imaging windows. Late gadolinium enhancement,

670 Pediatrics in Review

Downloaded from /pediatricsinreview/issue/44/12


by Hospital de Peditría 'Prof. Dr. Juan P. Garrahan' user
a marker of replacement fibrosis found in approximately compressed, blood flow to the myocardium can be dis-
half of patients, is a risk marker for adverse outcomes in rupted and result in myocardial ischemia, ventricular ar-
HCM. (13) rhythmias, and SCD. Due to the silent nature of coronary
Screening is recommended for first-degree relatives of anomalies, the true prevalence of AAOCA and absolute risk
patients with HCM, who can manifest at any age and thus of SCD are unknown. (18) However, studies suggest that
warrant surveillance imaging every 1 to 3 years. In addi- although anomalous left coronary artery (ALCA) is at least
tion, genetic testing with counseling is offered to individu- 6 times less common than anomalous right coronary artery
als with HCM. If genetic testing reveals a pathogenic (ARCA), it has a much higher risk of SCD. (19)
variant, cascade genetic testing should be offered to first- Although half of SCD cases occur in previously asymptom-
degree relatives, with clinical surveillance for those who atic individuals, patients may present with chest pain, exer-
carry the pathogenic variant. If a pathogenic mutation is tional syncope, or diminished exercise tolerance. (18) Results
not identified in the proband, cascade genetic testing is of physical examination, ECG, and stress testing are often nor-
not recommended for first-degree relatives. (10) mal. Coronary anomalies are typically diagnosed by transtho-
Guidelines, which have historically recommended against racic echocardiography and advanced imaging modalities,
participation in most competitive sports for patients with which should also be directed at identifying high-risk ana-
HCM, are complicated and have recently evolved. Although tomic features that increase the risk of SCD. These high-risk
HCM is the most common cause of SCD in young athletes, anatomic features include slitlike ostium, acute take-off angle,
growing evidence shows that recreational exercise of mild to intramural course (within the wall of the aorta), and interarte-
moderate intensity is safe and beneficial in these patients. rial course (between the aorta and the pulmonary trunk).
(10)(14) As of 2020, class I recommendations permit mild to Restriction from competitive sports is recommended for
moderate exercise in most patients with HCM. (10) Athletes patients who have ARCA with high-risk features, symp-
with HCM are encouraged to undergo a comprehensive eval- toms, arrhythmias, or evidence of ischemia. Athletes can
uation and shared discussion of the potential risks and bene- generally resume competitive sports 3 months after repara-
fits of sports participation with an expert. (10)(15) Although tive surgery if they are asymptomatic and if testing reveals
participation in low-intensity competitive sports is reasonable no evidence of ischemia. For patients with ARCA without
in most cases, if certain conditions are met, participation in symptoms or concerns for arrhythmias or ischemia, partici-
moderate- to high-intensity sports can also be considered pation in competitive sports is acceptable with adequate
through shared decision-making. (10) counseling and a discussion about the potential risk of car-
diac events. In athletes with ALCA, restriction from com-
Coronary Artery Anomalies petitive sports is generally recommended. As with ARCA,
Coronary artery anomalies are a diverse group of congenital athletes with ALCA may be cleared for competitive sports 3
conditions with a wide range of clinical manifestations. An- months after surgery if they are asymptomatic and have no
atomic variations in coronary anatomy are common and evidence of ischemia. (20)
range from benign and not causing myocardial ischemia or
SCD to life-threatening. (16)(17) In fact, anomalous aortic Myocarditis
origin of a coronary artery (AAOCA) is the second most Myocarditis, the third most common cause of SCD in young
common cause of SCD in young athletes. (3) athletes, is an inflammatory disease of the myocardium due
Blood flow to the myocardium is derived from 2 main to infections, systemic diseases, drugs, and toxins. (3) Viral
coronary arteries, each arising from a different location of infections, particularly due to enteroviruses (eg, coxsackievi-
the aorta. In most individuals, the right coronary artery ruses) and adenoviruses, are the most common causes of
arises from the right aortic sinus and supplies blood to myocarditis in the Unites States. Clinical manifestations vary
the right side of the heart, whereas the left main coronary widely, ranging from asymptomatic cases to heart failure
artery arises from the left aortic sinus and bifurcates into and potentially fatal arrhythmias. Common symptoms asso-
the left anterior descending and circumflex arteries, to- ciated with myocarditis are fatigue, shortness of breath, nau-
gether supplying blood to the left side of the heart. sea (and/or abdominal pain), and chest pain. (21)(22) The
In patients with AAOCA, 1 of the coronary arteries arises diagnosis is usually suspected due to the presence of cardiac
from an abnormal location on the aorta, creating an abnor- symptoms, elevated plasma troponin level, nonspecific ECG
mal coronary artery opening (ostium), take-off angle, and abnormalities (ST/T-wave changes), and echocardiographic
course. If the coronary artery becomes stretched, kinked, or findings, such as cardiac chamber enlargement and/or

Vol. 44 No. 12 D E C E M B E R 2 0 2 3 671

Downloaded from /pediatricsinreview/issue/44/12


by Hospital de Peditría 'Prof. Dr. Juan P. Garrahan' user
672
Table 2. Cardiovascular Conditions Associated with Sudden Cardiac Death
PHYSICAL SPORTS GUIDELINES
CONDITION CAUSE EXAMINATION ECG/HOLTER ECHOCARDIOGRAM (USA)a MANAGEMENT NOTES
HCM Sarcomere mutations Systolic murmur LV hypertrophy and/ LV wall thickness $15 Restrict from high- Consider ICD if high High risk: previous cardiac
(mostly AD) or RV hypertrophy, mm or 2 SD for intensity sports, risk arrest or sustained VT;
large R or S waves; weight; small LV encourage mild- β-Blockers may family history of SCD;
left axis deviation; cavity size; abnormal intensity activity; improve symptoms syncope; nonsustained

Downloaded from /pediatricsinreview/issue/44/12


Pediatrics in Review
ventricular mitral valve 1/ shared decision- but do not reduce VT; abnormal blood
arrhythmias mitral regurgitation; makingb mortality pressure response to

by Hospital de Peditría 'Prof. Dr. Juan P. Garrahan' user


LV outflow tract exercise; severe LV
obstruction hypertrophy
AAOCA Abnormal coronary Normal Usually normal; Abnormal coronary Anomalous left coronary artery Coronary angiography is
embryogenesis ischemic changes; artery Restrict from Surgical repair recommended
arrhythmias competitive sports Stress tests may yield
false-negatives
Anomalous right coronary artery Consider sports
Restrict from Surgical repair if participation 3 mo
competitive sports high-risk features, after reparative surgery
if asymptomatic and
if symptoms, symptoms,
arrhythmias, or arrhythmias, or no evidence of
evidence of evidence of ischemia
ischemia ischemia
Myocarditis Acquired; usually Evidence of Nonspecific ST- Wall motion Restrict from sports for Supportive Clearance depends on
viral infection cardiogenic shock, segment and T- abnormalities; at least 3–6 mo normalization of serum
sinus tachycardia wave ventricular markers of cardiac
or abnormalities; dysfunction; mitral injury, normalization of
tachyarrhythmias, ectopy/ regurgitation; systolic function, and
gallop or arrhythmias pericardial effusion resolution of any
hepatomegaly arrhythmias on
exercise ECG
AC Mutations in Normal Epsilon wave; Enlarged RV, LV, or Restrict to low- ICD if high risk Cardiac MRI is
desmosomal localized QRS both; ventricular intensity sportsc recommended
proteins, eg, PKP2 widening; dysfunction High risk: aborted SCD,
(mostly AD) prolonged S-wave sustained VT, severe
Fatty fibrous upstroke; T-wave ventricular dysfunction
tissue replaces inversion in leads
heart muscle V1 through V3;
LBBB, PVCs, or VT
Channelopathies
LQTS Congenital: Normal QTc prolongation Normal Restrict to low- β-blockers Often incidental detection
mutations in Abnormal T-wave intensity sports if ICD if high risk on ECG; consider in
potassium or morphology symptomatic or patients with syncope
sodium channels prolonged QTcc or atypical seizures
(mostly AD) LQTS type 1 Risk of SCD is related
Acquired: genotype should to QTc duration,
medications, avoid competitive genotype, history of
electrolyte swimming symptoms, sex of
abnormalities, etc individual
Continued
Table 2. Cardiovascular Conditions Associated with Sudden Cardiac Death (Continued)
PHYSICAL SPORTS GUIDELINES
CONDITION CAUSE EXAMINATION ECG/HOLTER ECHOCARDIOGRAM (USA)a MANAGEMENT NOTES
Brugada Mutations in sodium Normal Coved ST-segment Normal Restrict to low- Quinidine Highest risk of SCD with
syndrome channels, eg, elevation in leads intensity sportsbc Ablation type 1 Brugada pattern
SCN5A (mostly V1 and V2; If asymptomatic, ICD if high risk on ECG

Downloaded from /pediatricsinreview/issue/44/12


AD) abnormal T-wave participation can be Fevers increase risk;
morphology considered with treat with antipyretics

by Hospital de Peditría 'Prof. Dr. Juan P. Garrahan' user


precautionary
measures
CPVT Calcium Normal Normal Normal Restrict to low- Antiarrhythmic Exercise stress test may
dysregulation intensity sportsc medications (eg, reveal ectopy,
Mutation in nadolol, flecainide) polymorphic VT
ryanodine Left cardiac
receptor 2 (RyR2) sympathetic
(mostly AD) denervation
ICD if high risk
WPW Accessory pathway; Normal Slurred QRS upstroke Normal Asymptomatic: β-Blockers or ablation Return to competitive
syndrome unknown cause (“delta wave”), No restriction useful to prevent sports 4 wk after
short PR interval Symptomatic: SVT radiofrequency
EPS recommended ablation

AAOCA5anomalous aortic origin of a coronary artery, AC5arrhythmogenic cardiomyopathy, AD5autosomal dominant, CPVT5catecholaminergic polymorphic ventricular tachycardia, ECG5electrocardiogram,
EPS5electrophysiology study, HCM5hypertrophic cardiomyopathy, ICD5implantable cardioverter-defibrillator; LBBB5left bundle branch block, LQTS5long QT syndrome, LV5left ventricle, MRI5magnetic reso-
nance imaging, PVC5premature ventricular contraction, RV, right ventricle, SCD5sudden cardiac death, SVT5supraventricular tachycardia, VT5ventricular tachycardia, WPW5Wolff-Parkinson-White.
a
Sports restriction guidelines are from Maron BJ, Zipes DP, Kovacs RJ; on behalf of the American Heart Association Electrocardiography and Arrhythmias Committee of the Council on Clinical Car-
diology, Council on Cardiovascular Disease in the Young, Council on Cardiovascular and Stroke Nursing, Council on Functional Genomics and Translational Biology, and the American College of
Cardiology. Eligibility and disqualification recommendations for competitive athletes with cardiovascular abnormalities: preamble, principles, and general considerations: a scientific statement from
the American Heart Association and American College of Cardiology. Circulation. 2015;132:e256–e261.
b
Individuals who are genotype positive but phenotype negative can continue to participate in all competitive sports.
c
Low-intensity class 1A sports have low static (<20% of maximum voluntary contraction) and low dynamic (<40% of maximum oxygen uptake) components. These sports include billiards, bowl-
ing, cricket, curling, golf, and riflery.

Vol. 44 No. 12 D E C E M B E R 2 0 2 3
673
cause of SCD in young athletes in Italy while accounting for
less than 5% in the United States. (24)(26)(27)
Clinical signs or symptoms of AC such as dizziness,
syncope, or palpitations are rarely recognized before pu-
berty, with onset typically ranging from the late 20s to
early 30s. (27) As a result, the condition is not likely to be
detected based on history and physical examination find-
ings unless there is a family history notable for SCD
or relatives diagnosed as having AC. Although ECG has
Figure 1. Echocardiograms showing left ventricular hypertrophy in a
teenager with hypertrophic cardiomyopathy. low sensitivity for this condition, it may show important
abnormalities, including a widened QRS, epsilon wave
(Fig 2A), and inverted T waves in the right precordial leads
impaired LV systolic or diastolic function. Although endo- (V1 through V3), the latter of which is abnormal after 14 years
myocardial biopsy remains the gold standard, CMR has of age. (25)(28) Although echocardiography is generally nor-
emerged as a more sensitive and less invasive test to confirm mal, especially in the early stages of the disease course, a
the diagnosis. normal study does not exclude the diagnosis. If clinical sus-
To promote the resolution of inflammation, athletes di- picion is high, CMR should be performed to evaluate for
agnosed as having myocarditis should be restricted from myocardial changes consistent with the disease.
exercise for 3 to 6 months, depending on the extent of car- There is no cure for AC. When the diagnosis is made,
diac injury and inflammation on CMR. (15) Because SCD the individual should be restricted from competitive sports
in myocarditis is most likely due to development of ven- with the possible exception of low-intensity class 1A sports,
tricular tachyarrhythmias resulting from myocardial scar- which include billiards, bowling, cricket, curling, golf, and
ring, ambulatory ECG monitoring and/or exercise stress riflery. (15) Patients who meet specific high-risk criteria
testing are used in addition to CMR to evaluate for ar- usually undergo insertion of an implantable cardioverter-
rhythmias before clearing patients for return to competi- defibrillator (ICD). (29)
tive sports. (23)
Ion Channelopathies
Arrhythmogenic Cardiomyopathy Ion channelopathies are a group of hereditary defects in the
Arrhythmogenic cardiomyopathy (AC), previously known as membrane channel proteins that can cause lethal arrhyth-
arrhythmogenic right ventricular cardiomyopathy, is an inher- mias and SCD in individuals with structurally normal
ited cardiomyopathy characterized by progressive replacement hearts. These disorders are suspected to account for a sig-
of myocardium with fibrofatty tissue and a predisposition for nificant proportion of cases in which the autopsy reveals a
ventricular arrhythmias and SCD. Although structural abnor- structurally normal heart with no identifiable cause of SCD.
malities of the right ventricle predominate, LV involvement is (30) Long QT syndrome (LQTS), Brugada syndrome, and
also possible. (24)
AC is usually inherited in an autosomal dominant
manner with incomplete penetrance and variable expres-
sivity. Most cases involve mutations in genes encoding
proteins in desmosomes, the membrane structures that
regulate intercellular adhesion and maintain structural
integrity of tissues during mechanical stress. Mutations
in the plakophilin-2 (PKP2) gene are the most frequent
cause. (25) AC predominantly affects men, whereas
women with an associated gene mutation have a lower
chance of expressing the disease and are more likely to be
asymptomatic carriers. (25)
Figure 2. Electrocardiographic findings associated with cardiac diseases.
The prevalence of AC is challenging to estimate because A. Epsilon wave in arrhythmogenic right ventricular cardiomyopathy.
it is often discovered postmortem. Its association with the B. Prolonged QT interval in long QT syndrome. C. Coved-type ST-segment
elevation in Brugada syndrome. D. Delta wave in Wolff-Parkinson-White
Mediterranean region explains why it is the most common syndrome.

674 Pediatrics in Review

Downloaded from /pediatricsinreview/issue/44/12


by Hospital de Peditría 'Prof. Dr. Juan P. Garrahan' user
catecholaminergic polymorphic ventricular tachycardia sympathetic denervation to reduce adrenergic stimulation of
(CPVT) are the most common channelopathies associated the heart. (37)
with SCD. Patients with these disorders may report palpita- Brugada syndrome commonly involves autosomal dom-
tions and syncope (particularly exertional or with auditory inant mutations in the cardiac sodium channel SCN5A
stimulation) and a family history notable for early and/or gene, although pathogenic variants in other genes have
unexplained death. Although findings from physical exami- been identified. (38) Patients with this condition may re-
nation and echocardiography are generally normal, each port a history of palpitations or syncope triggered by fever
channelopathy has a unique ECG fingerprint. and a family history of Brugada syndrome or sudden
LQTS, which can be inherited or acquired (eg, QT- death. The cardiac examination is usually normal; how-
prolonging drugs), is characterized by prolonged ventricu- ever, the diagnosis is based on ECG abnormalities that
lar repolarization leading to ventricular arrhythmias, clas- may occur spontaneously or be unmasked by a sodium
sically torsade de Pointes (“twisting of peaks” in French). channel blocker (ie, provocative drug testing). ECG may
The risk of arrhythmias and SCD is directly related to the demonstrate pathognomonic abnormalities, including a
duration of the QTc interval; each 10-millisecond increase “coved-type” ST-segment elevation (Fig 2C) or “saddle-
in QTc has been shown to increase the risk of malignant back” ST-segment elevation in the right precordial leads,
arrhythmias. (31) QTc intervals are considered prolonged V1 through V3, whereas Holter monitoring is useful to
when they are greater than 460 milliseconds in males or evaluate for asymptomatic arrhythmias. Lethal arrhyth-
greater than 470 milliseconds in females and highly ab- mias may be triggered by increased vagal tone (eg, during
normal regardless of sex when greater than or equal to exercise recovery or sleep) as opposed to during exercise.
500 milliseconds (Fig 2B). Although QTc prolongation is (39) Despite the absence of a clear association between ex-
a hallmark of LQTS, 40% of patients with genetically con- ercise and SCD in Brugada syndrome, previous US guide-
firmed LQTS have a normal QTc duration on baseline lines recommended restriction from competitive sports
ECG, with only subtle T-wave abnormalities. (32)(33) In with the potential exception of class 1A sports. (39)(40)
such cases of concealed LQTS, exercise stress testing can Current guidelines specify that participation in competitive
unmask ECG abnormalities. On the opposite end of the sports may be considered once appropriate precautionary
spectrum, LQTS is also frequently overdiagnosed, espe- measures and treatments are in place provided that the ath-
cially in patients with prolonged QTc secondary to vasova- lete has been asymptomatic for at least 3 months. (36) Treat-
gal syncope. (34) ment has historically been limited to drugs and ICDs for
At least 17 different genes are associated with LQTS, of high-risk patients, although catheter ablation—a minimally
which 90% are accounted for by 1 of 3 major genes: invasive procedure used to destroy abnormal tissue—has
KCNQ1 (LQTS1), KCNH2 (LQTS2), and SCN5A (LQTS3). also been suggested as a therapeutic option. (38)
(35) SCD associated with mutations in each have signature CPVT results from genetic mutations—most commonly
triggers: exercise, especially swimming, for LQTS1; arousal, autosomal dominant in the RyR2 gene encoding a ryano-
especially a sudden loud noise (along with one-third of dine receptor in the sarcoplasmic reticulum—that cause ab-
SCDs during exercise) for LQTS2; and during sleep (and normal calcium release in cardiomyocytes. The disease is
<5% of SCDs during exercise) with LQTS3. (35) characterized by adrenergic-induced ventricular tachyar-
Current guidelines recommend restriction of individuals rhythmias, including bidirectional ventricular tachycardia, a
with LQTS to class 1A sports if they have symptoms, ICDs, rare tachyarrhythmia in which dual QRS morphologies al-
or significant QTc prolongation (males: >470 milliseconds; ternate on a beat-to-beat basis. In this condition, syncope or
females: >480 milliseconds), although participation may be sudden death is usually triggered by acute emotional stress
considered after initiation of treatment and appropriate pre- or exercise. History and physical examination findings are
cautionary measures. (36) Genetically positive individuals often normal, with a positive family history of exercise/
who have normal QT intervals can compete in sports with- emotion syncope in approximately one-third of cases. (41)
out restriction, except for individuals with symptomatic Although resting ECG is usually normal, ventricular ectopy
LQTS1, who should be restricted from competitive swim- and arrhythmias can be provoked by exercise (ie, exercise
ming. b-Blockers, particularly nadolol and propranolol, are stress testing) or epinephrine. With mortality in this condition
the first-line therapy for patients with LQTS and are most ef- high if untreated (30%–50% by age 40 years), those diag-
fective in LQTS1. Other therapeutic options reserved for nosed as having CPVT are generally restricted from compet-
high-risk patients include ICD placement and left cardiac itive sports and treated with antiarrhythmic medications,

Vol. 44 No. 12 D E C E M B E R 2 0 2 3 675

Downloaded from /pediatricsinreview/issue/44/12


by Hospital de Peditría 'Prof. Dr. Juan P. Garrahan' user
including b-blockers (ie, nadolol) and sodium channel condition, it may pose an increased risk of SCD. (49) Fi-
blockers (ie, flecainide), left cardiac sympathetic denerva- nally, atherosclerotic cardiovascular disease is a major cause
tion, and occasionally ICD implantation. (36)(41)(42) of SCD in older athletes but rarely causes SCD in young
athletes, although the likelihood is higher if there are risk
Commotio Cordis factors such as a history of Kawasaki disease. (50) Although
Commotio cordis (Latin etymology, “agitation of the heart”) performance-enhancing drugs can increase cardiac risk, evi-
refers to a disruption of the normal heart rhythm due to a dence has been inconclusive about their involvement in
direct blow to the chest and is particularly distressing be- SCD. (51)
cause commotio cordis usually occurs in an otherwise
healthy athlete with no identifiable heart problem. Each NORMAL FINDINGS IN ATHLETES
year in the United States, there are approximately 10 to 20 The physiological and benign profile of an athlete’s heart
cases of this condition, which is primarily an electrical phe- (so-called athlete’s heart) can be difficult to differentiate
nomenon that does not result from structural damage to from cardiovascular abnormalities. Because ECG changes
the myocardium. (43) For ventricular fibrillation to be in- are observed in approximately 40% of trained athletes, in-
duced, the blunt impact must involve a minimum pressure ternational consensus standards for normal, abnormal,
of 250 mm Hg and must occur during a narrow window of and borderline ECG findings in this population have been
vulnerability during the cardiac cycle (ie, during the T-wave established. (52)(53) Examples of findings that are consid-
upslope). (43) ered normal for trained athletes include high QRS ampli-
Approximately 95% of cases occur in males, with a tude (meeting voltage criteria for LV hypertrophy), early
mean age of 15 years. (43) Baseball has the highest inci- repolarization, sinus bradycardia, sinus arrhythmia, and
dence of commotio cordis. Although commercial protec- first-degree atrioventricular block. Such findings are attrib-
tive equipment may be helpful in preventing commotio uted to intense athletic conditioning resulting in structural
cordis, it can still occur when the victim is wearing a chest cardiac remodeling and increased vagal tone and do not
guard. (44)(45) warrant further evaluation.
During the past 2 decades, survival rates of patients Morphologic remodeling associated with intense ath-
with commotio cordis have increased from 15% to 60%, letic training, such as increased LV chamber dimensions
largely due to improved recognition of sudden cardiac ar- and wall thickness, can mimic features of genetic and ac-
rest (SCA), bystander-initiated CPR, and the widespread quired heart disease such as cardiomyopathies. Certain
availability of AEDs at sporting events. (46) In fact, the echocardiographic features are helpful in distinguishing
survival rate drops to 3% when resuscitation is delayed be- between benign adaptive remodeling and heart disease,
yond 3 minutes. (44) If no underlying cardiac abnormality and a complete description is beyond the scope of this re-
is identified, survivors of commotio cordis are allowed to view. However, a thorough evaluation of LV wall thickness
resume training and competition on full recovery. (46) and morphology, diastolic LV cavity size, atrial size, sys-
tolic function, and diastolic function is required. For cases
Other Causes in which echocardiographic findings are equivocal, re-
Several other conditions are associated with SCD, includ- peated evaluation after a period of deconditioning (ie,
ing severe obstructive lesions such as aortic stenosis or avoiding exercise) can be helpful because ECG and echo-
pulmonary stenosis, aortopathies such as Marfan syn- cardiographic changes associated with an athlete’s heart
drome (ie, due to aortic rupture or dissection), primary should normalize during this time. (54) The distinction
pulmonary hypertension, sarcoidosis, and sickle cell trait. between physiological and pathological changes in athletes
Patients with complex congenital heart defects, including is important because an incorrect diagnosis can have sig-
those status post repair or palliative procedures, are at risk nificant consequences, such as exclusion from competitive
for arrhythmias and SCD due to surgical scarring, hemo- sports or inappropriate reassurance and a missed opportu-
dynamic abnormalities, residual lesions, or ventricular dys- nity for therapeutic intervention.
function. (47) Wolff-Parkinson-White syndrome (Fig 2D) is
considered a rare cause of SCD, which likely occurs due to SCREENING GUIDELINES
the rapid conduction of atrial fibrillation to the ventricles via The primary goal of screening is to identify conditions
the accessory pathway, resulting in ventricular fibrillation. (48) that predispose individuals to SCA or SCD. Personal his-
Although mitral valve prolapse is typically considered a benign tory, family history, and physical examination findings are

676 Pediatrics in Review

Downloaded from /pediatricsinreview/issue/44/12


by Hospital de Peditría 'Prof. Dr. Juan P. Garrahan' user
the core components of screening. The American Acad- in a prospective study. These questions were designed to be
emy of Pediatrics (AAP) recommends that all children un- simple and easy to incorporate into a family questionnaire.
dergo screening for the risk of SCD regardless of athletic (55) The first question focuses on personal history of sudden
status. (55) Screening should occur during the PPE, a min- loss of consciousness, particularly events triggered by exertion
imum of every 3 years, or on entry into middle or junior or sudden loud noises, which tend to occur in patients with
high school and into high school. Four main screening channelopathies such as LQTS. The second question focuses
questions are recommended: on symptoms of chest pain or shortness of breath on exertion,
which can signal myocardial ischemia, congestive heart fail-
1. Have you ever fainted, passed out, or had an unexplained ure, arrhythmias, and valvar disease. The other 2 questions in-
seizure suddenly and without warning, especially during quire about family members with heart conditions, potentially
exercise or in response to sudden loud noises, such as pointing to a familial inheritance pattern. Steinberg et al (56)
doorbells, alarm clocks, and ringing telephones? previously showed that cardiac abnormalities can be identified
2. Have you ever had exercise-related chest pain or short- in nearly one-third of first-degree relatives of SCA survivors or
ness of breath? SCD victims.
3. Has anyone in your immediate family (parents, grandpar- Meanwhile, the American Heart Association (AHA) has
ents, siblings) or other more distant relatives (aunts, uncles, published guidelines for preparticipation screening of com-
cousins) died of heart problems or had an unexpected sud- petitive athletes, which consists of a 14-element (previously
den death before age 50 years? This would include unex- 12-point) history and physical examination (Table 3). Use of
pected drownings, unexplained car accidents in which the this 14-element checklist for preparticipation screening is a
relative was driving, or sudden infant death syndrome. class I recommendation based on a scientific statement jointly
4. Are you related to anyone with HCM or hypertrophic ob- published by the AHA and American College of Cardiology.
structive cardiomyopathy, Marfan syndrome, AC, LQTS, (57)(58) As with the AAP’s screening questions, the AHA’s 14-
short QT syndrome, Brugada syndrome or CPVT, or a con- element screening tool was developed based on expert opinion
dition requiring implantation of a pacemaker or ICD at and over time has become the most commonly accepted
younger than 50 years? screening method for young athletes. (55)(57)(58) In addition,
it has also been incorporated into other screening guidelines,
The AAP’s recommended screening questions are based such as a revised version with changes in language and word-
on expert consensus and have not been scientifically validated ing that has been incorporated into the PPE: Preparticipation

Table 3. The 14-Element American Heart Association Recommendations for Preparticipation Screening of
Competitive Athletes (57)

Personal history:
1. Chest pain/discomfort/tightness/pressure related to exertion
2. Unexplained syncope/near-syncopea
3. Excessive exertional and unexplained dyspnea/fatigue or palpitations, associated with exercise
4. Previous recognition of a heart murmur
5. Elevated systemic blood pressure
6. Previous restriction from participation in sports
7. Previous testing for the heart, ordered by a physician
Family history:
8. Premature death (sudden and unexpected, or otherwise) before age 50 y attributable to heart disease in $1 relative
9. Disability from heart disease in a close relative aged <50 y
10. Hypertrophic or dilated cardiomyopathy, long QT syndrome or other ion channelopathies, Marfan syndrome, or clinically significant
arrhythmias; specific knowledge of certain cardiac conditions in family members
Physical examination:
11. Heart murmurb
12. Femoral pulses to exclude aortic coarctation
13. Physical stigmata of Marfan syndrome
14. Brachial artery blood pressure (sitting position)c
a
Judged not to be of neurocardiogenic (vasovagal) origin; of particular concern when occurring during or after physical exertion.
b
Refers to heart murmurs judged likely to be organic and unlikely to be innocent; auscultation should be performed with the patient in
both the supine and standing positions (or with the Valsalva maneuver), specifically to identify murmurs of dynamic left ventricular outflow
tract obstruction.
c
Preferably taken in both arms.

Vol. 44 No. 12 D E C E M B E R 2 0 2 3 677

Downloaded from /pediatricsinreview/issue/44/12


by Hospital de Peditría 'Prof. Dr. Juan P. Garrahan' user
Physical Evaluation, 5th Edition, a monograph published by the CPR and AEDs
AAP in collaboration with 5 other medical societies. (59) SCD can be prevented through prompt recognition, admin-
However, despite being endorsed by medical societies, istration of high-quality CPR, and early defibrillation. This
the 14-element screening tool remains heavily underused. “Chain of Survival” is highly dependent on public engage-
A recent study in Pediatrics found that a minority (27%) of ment. Administration of bystander CPR is associated with
the 48 states providing PPE forms incorporate all 14 ele- increased survival and improved neurologic outcome. (65)
ments in their screening forms. (60) Moreover, a quality Although previous CPR training increases bystanders’ will-
review by Miliaresis et al (61) found that only one-third of ingness to perform CPR, only a small percentage (<3%) of
pediatricians are familiar with the AHA’s recommended the US population receives training each year. (66)(67)(68)
screening form and that, on average, only 3.5 of 14 ele- In recent years, self-directed online courses have emerged
ments are documented in visit notes. Barriers to PPE use as an acceptable alternative to in-person courses. Further-
include lack of awareness of the 14-element screening more, statewide laws requiring CPR training in high
form, use of a different form, and time constraints. Of schools have also translated to more laypersons learning
note, the authors did increase use of the PPE through this lifesaving skill, although the quality of such training is
standard quality improvement methods. (61) inconsistent and has striking geographic disparities. (69)
Although the need for preparticipation screening of Currently, the AHA recommends training students as early
young athletes is widely supported, the optimal approach has as middle school on how to perform high-quality CPR.
long been debated. Studies have shown that the 14-element Early defibrillation is another important intervention af-
screening has sensitivity and specificity of 20% and 68%, fecting outcomes after SCA, and arguably the most impor-
respectively, for cardiac risk factors among high school ath- tant determinant of survival. Despite the fact that most
letes. (62) The incorporation of ECG screening would cases of SCA involve a shockable rhythm (ie, ventricular fi-
certainly increase screening sensitivity, as evidenced by a brillation), the probability of surviving SCA caused by ven-
meta-analysis published in 2015, based on screening of tricular fibrillation diminishes rapidly over time, declining
47,137 athletes from several regions, that revealed a 5-fold approximately 10% each minute defibrillation is delayed.
(70)(71) From 2000 to 2006, exercise-related SCA in the
and 10-fold increase in sensitivity of ECG in detecting car-
young had an overall survival rate of 11%. (72) A more re-
diac conditions relative to history and physical examination,
cent study analyzing events from 2014 to 2018 reported a
respectively. (63) However, incorporating ECG screening
higher overall survival rate of 68% among student athletes,
would also involve a substantial cost that medical societies
with a survival rate of 85% if an on-site AED is used. (73)
such as the AHA view as prohibitive. With nearly 10 mil-
lion student athletes in the United States annually, the
Emergency Preparedness
widespread implementation of ECG screening would cost
Schools have a responsibility to prepare for cardiac emer-
billions of dollars. One must also consider the additional
gencies that goes beyond the mere presence of an AED or
monetary, legal, and emotional cost of dealing with false-
bystander with CPR training. The AHA recommends that
positive and false-negative test results. Therefore, ECG is
schools and/or school districts implement Cardiac Emer-
usually reserved for patients at increased risk for SCD
gency Response Plans (CERPs), which are associated with
based on routine screening with the PPE.
a lower incidence of SCD. (74)(75) Core components of a
CERP include establishing a Cardiac Emergency Response
SECONDARY PREVENTION
Team (a team of athletic trainers, teachers, and other staff
Preparticipation screening is useful for identifying athletes members with CRP/AED training), creating an emergency
with high-risk cardiac conditions, but no screening strat- activation system, ensuring AED equipment is easily acces-
egy can single-handedly prevent SCD. In addition, despite sible and properly maintained, offering frequent CPR/AED
improvements in survival rates after SCA, disparities in training, performing practice drills, and reviewing the plan
outcomes based on race and socioeconomic status continue on an ongoing and annual basis. (76) Documents needed to
to exist, with studies showing worse cardiac emergency pre- implement a CERP are available on the AHA’s website. (77)
paredness and lower survival rates in low-income neighbor- The AHA also recommends that states enact legislation
hoods versus high-income neighborhoods. (64) To address that requires and funds the creation and maintenance of
these disparities, it is essential to target these demographics CERPs in schools. (76) State legislation requiring schools
in the implementation of secondary prevention measures. to have an AED has been shown to significantly increase

678 Pediatrics in Review

Downloaded from /pediatricsinreview/issue/44/12


by Hospital de Peditría 'Prof. Dr. Juan P. Garrahan' user
AED availability across high schools, with public schools
• A focused history and physical examination is the
being most affected by such legislation. (78) As of 2022,
recommended preparticipation screening method,
according to data collected by the Sudden Cardiac Arrest
although the American Heart Association’s 14-element
Foundation, only 20 states require AEDs. (79)
screening tool has low sensitivity. (Based on
research evidence and consensus) (58)(62)
FUTURE DIRECTIONS
• Universal electrocardiographic screening is not
The absolute risk of SCD is low, even among athletes with
recommended in the United States. This is partly due
high-risk conditions, and the known detriments of lack of
to prohibitive cost and the potential for false-positives
physical activity have led to a recent focus on shared deci-
resulting in additional unnecessary testing. (Based on
sion-making. Moreover, machine learning algorithms can
research evidence and consensus) (57)(58)(84)
potentially help clinicians diagnose the conditions that
cause SCD and can already detect HCM and LQTS from • Providers should follow US guidelines for
standard ECGs with excellent sensitivities and specificities, determining the appropriate sports restriction
and it is possible that artificial intelligence can similarly for individuals with high-risk cardiac conditions.
be trained to facilitate more accurate risk stratification, re- However, it is also important to recognize that
sulting in a lower burden of unnecessary sports disqualifi- these conditions have a low risk of SCD and that
cation. (80)(81) Finally, gene therapy is an emerging area shared decision-making has emerged as an
of interest in SCD prevention, as investigators recently important framework for the contemporary sports
published the first hybrid gene therapy for LQTS1, which eligibility discussion. (Based on research evidence
is now advancing to animal model studies. (82) and consensus) (10)(15)(36)
• High-quality cardiopulmonary resuscitation and
CONCLUSION prompt defibrillation are integral to preventing
SCD once sudden cardiac arrest (SCA) has occurred.
SCD is a rare but devastating event that is often prevent-
The probability of surviving SCA caused by ventricular
able. When SCD occurs in a young athlete, it is frequently
fibrillation is reduced by 10% each minute
due to an underlying cardiac condition. Due to lack of dem-
defibrillation is delayed. (Based on research evidence
onstrated efficacy as well as prohibitive costs and feasibility
and consensus) (71)(75)
of large-scale ECG and echocardiography screening, routine
PPE is limited to history and physical examination. Primary • Schools and/or school districts should implement
care providers and pediatric cardiologists can prevent SCD Cardiac Emergency Response Plans, which can help
by identifying red flags associated with the most common schools prepare for SCAs and prevent SCDs. (Based on
causes of SCD. Because the PPE is imperfect at identifying some research evidence and consensus) (74)(75)(77)
athletes with high-risk conditions, secondary measures are • Machine learning could play an important role in
also essential to preventing SCD. SCD prevention in the future. In addition, new
gene therapies may be available for individuals
diagnosed as having predisposing conditions. (Based
Summary
on some research evidence) (80)(81)(82)
• Preparticipation screening for young athletes
is important because it can potentially reduce
the risk of sudden cardiac death (SCD). (Based on ACKNOWLEDGMENTS
research evidence and consensus) (55)(58)(59)(60)(83)
We dedicate this article to Frank J. Reali III, who lost his
• Several cardiac conditions are associated with life to sudden cardiac death, and we thank his family,
SCD. Hypertrophic cardiomyopathy, coronary who, through the Protecting One Young Heart at a Time
artery anomalies, and myocarditis are the 3 most Foundation, has generously supported local screenings for
common identifiable causes. Ion channelopathies high school athletes on Staten Island. We also thank Dr Philip
may account for a significant number of autopsy- Roth for his insightful suggestions and comments.
negative SCDs. (Based on research evidence and
References and teaching slides for this
consensus) (3)(30)
article can be found at
https://doi.org/10.1542/pir.2023-005975.

Vol. 44 No. 12 D E C E M B E R 2 0 2 3 679

Downloaded from /pediatricsinreview/issue/44/12


by Hospital de Peditría 'Prof. Dr. Juan P. Garrahan' user
PIR QUIZ

1. A 16-year-old male basketball player is brought to the pediatric outpatient


clinic by his parents for evaluation of chest pain and syncope with exertion.
The boy’s father and paternal uncle have been diagnosed as having
hypertrophic cardiomyopathy (HCM). On physical examination the patient
has a systolic murmur that becomes louder when he performs a Valsalva
maneuver. Of the following cardiac imaging findings, which one is
considered a distinctive feature of HCM diagnosis?
REQUIREMENTS: Learners can
A. Asymmetrical septal hypertrophy. take Pediatrics in Review quizzes
B. Early gadolinium enhancement on cardiac magnetic resonance imaging. and claim credit online only at:
C. Impaired left ventricular diastolic function. http://pedsinreview.org.

D. Impaired left ventricular systolic function. To successfully complete 2023


E. Left ventricular wall thickness greater than 30 mm. Pediatrics in Review articles for
AMA PRA Category 1 Credit™,
2. A 17-year-old female athlete is diagnosed as having HCM based on family
learners must demonstrate a
history and echocardiography findings. She is asymptomatic. She has minimum performance level of
undergone extensive evaluation and is found not to exhibit high-risk 60% or higher on this
features of HCM. The patient and her family have had a comprehensive assessment. If you score less
discussion about the risks and benefits of physical activity for individuals than 60% on the assessment,
you will be given additional
with HCM. They also ask whether the patient requires placement of an opportunities to answer
implantable cardioverter-defibrillator (ICD). Which one of the following questions until an overall 60%
management and physical activity recommendations is most appropriate to or greater score is achieved.
discuss with this patient and her parents?
This journal-based CME activity
A. Do not place an ICD, and clear her for participation in only low-intensity is available through Dec. 31,
physical activity. 2025, however, credit will be
B. Do not place an ICD, and clear her for participation in moderate- to recorded in the year in which
the learner completes the quiz.
high-intensity physical activity.
C. Place an ICD, and subsequently clear her for low-intensity physical activity.
D. Place an ICD, and subsequently clear her for moderate- to high-intensity
physical activity.
E. Place an ICD, and subsequently recommend that she not participate in
any physical activity. 2023 Pediatrics in Review is
3. A 17-year-old female athlete is brought to the outpatient clinic by her parents for approved for a total of 30
Maintenance of Certification
sports preparticipation clearance. She was diagnosed as having viral myocarditis
(MOC) Part 2 credits by the
3 months ago after she developed chest pain and dyspnea with exercise. American Board of Pediatrics
She recently completed follow-up testing; echocardiogram demonstrated (ABP) through the AAP MOC
normal ventricular function, exercise testing and 24-hour Holter monitor showed Portfolio Program. Pediatrics in
Review subscribers can claim up
no evidence of arrhythmia, and cardiac magnetic resonance imaging did not show
to 30 ABP MOC Part 2 points
any cardiac injury or inflammation. The girl is hoping to return to competitive upon passing 30 quizzes (and
basketball. Which one of the following is the most appropriate recommendation claiming full credit for each
of the timing to return to play in this patient? quiz) per year. Subscribers can
start claiming MOC credits as
A. In 3 months after an additional round of testing. early as November 2023. To
B. In 6 months without additional testing. learn how to claim MOC points,
C. In 9 months without additional testing. go to: https://publications.aap.
org/journals/pages/moc-credit.
D. She can now return to training and competition.
E. She should be restricted from high-intensity physical activity permanently.

680 Pediatrics in Review

Downloaded from /pediatricsinreview/issue/44/12


by Hospital de Peditría 'Prof. Dr. Juan P. Garrahan' user
4. An adolescent male athlete with a family history of sudden unexplained
death dies after collapsing while playing football. There is no history of
trauma to the chest. The autopsy does not reveal findings of any structural
heart or lung disease. Which one of the following is the most likely cause of
death in this patient?
A. Arrhythmogenic cardiomyopathy.
B. Commotio cordis.
C. Long QT syndrome.
D. Myocarditis.
E. Sarcoidosis.
5. An adolescent presents to the outpatient pediatric office for preparticipation
physical evaluation before the soccer season. The patient’s pediatrician
discusses the 14 elements of the American Heart Association’s (AHA)
screener for heart disease with the medical students attending the clinic.
The AHA screening criteria include which one of the following elements?
A. Blood pressure in supine and standing positions.
B. Echocardiogram.
C. Electrocardiogram.
D. Family history of ion channelopathies.
E. Radial and pedal pulses.

Vol. 44 No. 12 D E C E M B E R 2 0 2 3 681

Downloaded from /pediatricsinreview/issue/44/12


by Hospital de Peditría 'Prof. Dr. Juan P. Garrahan' user
ARTICLE

Ensuring Emergency Readiness in the


Pediatric Primary Care Setting:
An Updated Guideline
Maria Milla, MD, MPH, FAAP,* Elisa Prebble, MD,* Gloria Riefkohl, MD, MPH, MSHSA, FAAP,*
Fernando Llopiz Martin, MD, FAAP,* Danielle Altares Sarik, PhD, APRN, CPNP-PC,*
Paola Garcia-Herreros, MSN, APRN, CPNP-PC*
*Nicklaus Children’s Hospital, Miami, FL

PRACTICE GAPS

In pediatric primary care, the misconception that emergencies rarely occur


can lead to a barrier in effectively preparing for and responding to life-
threatening situations. Often, practices may not be in close proximity to a
hospital, (1) and providers may not feel adequately prepared with pediatric
emergency skills. (2) Gaps in in-office protocols for emergencies may also
present challenges for adequate responses (3) as well as limitations in
available supplies and up-to-date emergency equipment and medications. (1)

OBJECTIVES After completing this article, readers should be able to:

1. Provide structured, evidence-based recommendations for the pediatric


primary care provider and practice regarding common urgent health
concerns and emergencies.
2. Review strategies for increasing the readiness of primary care office staff,
AUTHOR DISCLOSURE Drs Milla, Prebble,
including needed education, equipment, and training. Riefkohl, Llopiz Martin, and Sarik and Ms
Garcia-Herreros have disclosed no
3. Identify gaps and potential areas of improvement for effective emergency
financial relationships relevant to this
response in the pediatric primary care office. article. This commentary does not
contain discussion of an unapproved/
4. Identify the most common pediatric emergencies in the primary care investigative use of a commercial
setting. product/device.

5. Promote the implementation of proper protocols and training for


emergency response in the primary care setting. ABBREVIATIONS

AAP American Academy of Pediatrics


AHA American Heart Association
BLS basic life support
INTRODUCTION AND BACKGROUND ED emergency department
EMS emergency medical services
Pediatric primary care providers (PCPs) are highly competent, compassionate, IV intravenous
and trusted professionals. Many parents and caregivers rely on these providers ORT oral replacement therapy
PALS pediatric advanced life support
and their offices to support appropriate growth and development, provide acute PCP primary care provider
care, and serve as a source of advice for the journey through childhood. When SABA short-acting b-agonist

682 Pediatrics in Review

Downloaded from /pediatricsinreview/issue/44/12


by Hospital de Peditría 'Prof. Dr. Juan P. Garrahan' user
patients and families encounter challenging situations, it is Considering that pediatric patients present to the pri-
often the PCP who guides them to the correct resources. (4) mary care office with an emergency/urgent complaint be-
Emergencies are no different; emergencies happen sud- fore their first birthday, and that a patient’s first office visit
denly, leading parents to seek care at their PCP’s office. In is likely to occur before 8 weeks of age, it is imperative
this situation, the question arises as to whether the PCP that clinicians are ready and able to confront emergency
and the primary care office are well-equipped and prepared challenges for children of any age.
to handle emergencies. Offices may often be situated geo- Reports on presentation to the office setting indicate
graphically near a hospital or emergency department (ED), that the most common illnesses experienced by children,
facilitating a quick transfer of a patient in an emergency. in order from most to least frequent, are respiratory dis-
However, for practices in rural areas, or for critical emergen- tress, seizures, severe infections in young children, dehy-
cies when immediate stabilization is required, PCPs must dration, anaphylaxis, choking, and head trauma. (7)
be prepared. When faced with a new and infrequent emergency inci-
In 2020, unintentional injuries caused 33% of infant dent in the office, such as the evaluation and treatment of
deaths, of which 10.7% were caused by respiratory dis- a critically ill child, a PCP faces the challenge of crafting
tress. (5) Other leading causes of accidental injury deaths rapid clinical decisions and actions while avoiding poten-
in this population included suffocation, motor vehicle acci- tially common errors. As in any medical emergency, hav-
dents, drowning, poisoning, falls, and burns. Leading ing a standard algorithm enhances reliability.
causes of nonfatal ED visits for children vary slightly, with
unintentional falls, foreign body aspiration, being struck PARENT AND PATIENT EDUCATION
by or against an object, unintentional inhalation or suffo- The research (4) suggests that by using effective patient and
cation, unintentional cuts, dog bites, unintentional poison- parent education, emergencies that present to the office can
ing, and unintentional overexertion topping the list. (5) be prevented. Cardiopulmonary resuscitation and first aid
Although some patients and families may opt to pursue training along with anticipatory guidance can be lifesaving.
care in the ED setting, others will visit their trusted PCP Parents should be able to recognize an emergency, respond
office for assessment and treatment for these common appropriately to an emergency, and know how to prevent inju-
conditions. In addition to accidental injury, acute illness ries before they happen. Education should include knowing
PCP visits may be scheduled due to other common condi- where to go for help in the case of a non–life-threatening
tions, such as asthma, dehydration, and croup. Patients emergency. PCPs should educate families about when to use
who present to the PCP office with these conditions can the office, urgent care, ED, or subspecialty office. During
experience rapid deterioration or changes in acuity. To be health supervision visits, anticipatory guidance should include
prepared to address these situations, PCP offices should when and how to access emergency medical services (EMS)
create policies and train staff, as well as have clear and ac- such as 9-1-1, the Poison Control Center (1-800-222-1222),
cessible protocols and procedures in place to handle emer- and after-hours advice. Furthermore, it is important to discuss
gencies. From a structural perspective, confirming that the need for treatment consent for minors, any limitations to
proper equipment and medications are available and ac- emergency care and referrals dictated by the family’s insur-
cessible is a necessary step in ensuring preparedness. ance plan, and which facilities to access in a true medical
emergency. Family education materials, such as The Injury
EPIDEMIOLOGY Prevention Program (8)(9) and the emergency information
The pediatric primary care office contributes significantly to card, (10) both available through the American Academy of
the pediatric emergency care system. The outpatient office Pediatrics (AAP), could be offered as a resource in the office
serves as the primary care entry point across various geo- or practice website. Before the development of an emergency,
graphic settings. Although some suburban pediatric practices PCPs should discuss advance directives and limitations of life-
report an annual median of 24 emergency incidents, (4) saving treatments with caregivers. Although this may be an
studies show that pediatric offices are unprepared to manage uncomfortable conversation, it is of critical importance to dis-
emergencies in the office setting owing to documented lack cuss advance directives and limitations of lifesaving treat-
of preparedness in equipment and training. (6) Management ments for children with special medical needs or at increased
of pediatric emergencies is challenging in ambulatory set- risk for experiencing a life-threatening emergency. One option
tings because it requires ongoing, realistic preparedness and is to have this conversation with parents as part of the annual
planning aimed at addressing specific medical situations. care plan and health assessment. In some states, the law does

Vol. 44 No. 12 D E C E M B E R 2 0 2 3 683

Downloaded from /pediatricsinreview/issue/44/12


by Hospital de Peditría 'Prof. Dr. Juan P. Garrahan' user
not allow for EMS personnel to respect pediatric advance di- patient may last 30 minutes or more. In this scenario, the
rectives. (4) Any out-of-hospital do not resuscitate or “accept office needs to be equipped with an automated external defi-
natural death” orders should be discussed ahead of time with brillator and the right tools to maintain an airway. (4)(7) Es-
local EMS medical directors. This helps ensure that when sential and nonessential medication, equipment, and fluid
EMS personnel are asked to administer comfort measures recommendations can be found in Appendix A.
only, the EMS personnel are acting from a preapproved posi-
tion and remain clear of any liability. (4)
In addition, children with special health-care needs should POLICIES, PROTOCOLS, AND PROCEDURES
have a comprehensive summary of important information Emergencies in the primary care setting require unique re-
available to share with hospital and EMS providers. Use of sources and training. Common emergencies are reviewed in
the emergency information card and medical alert jewelry this article. Despite the wide variety of emergencies that
can be helpful during an emergency. (4)(11)(12) may present at the primary care office, at a minimum, basic
life support (BLS) and reference materials on full arrest
TRAINING THE STAFF should be readily available. These references can be found
Frontline staff (clinical and administrative) in the pediatric of- on the American Heart Association’s (AHA) website (www.
fice require training on how to recognize emergencies, acti- heart.org) under “Pediatric Algorithms.”
vate the emergency response system, and understand EMS
roles and capabilities. Clinical staff should be provided with
the chance and the incentive to reinforce knowledge and HEALTH-CARE PROFESSIONAL SKILLS AND
skills related to pediatric office-based emergencies. Providers DOCUMENTATION
and office staff should be strongly encouraged to keep their Provider Skills
certification in basic lifesaving or pediatric advanced lifesaving Most health-care systems have a well-delineated list of re-
courses current. Staff can also engage in mock codes and on- quired courses and certifications necessary to practice.
line case studies. Preparation and practice for primary care of- However, such requirements may not exist for providers
fice emergencies can be achieved through mock codes, in in private practice or community settings or for those not
which all members of the office staff regularly take part in affiliated or on staff with a local hospital. It is best practice
mock codes and practice as a team. Such exercises can in- for all pediatric providers to be certified in BLS and to be
crease staff and practitioner confidence and reduce anxiety prepared to administer these services in cases of emer-
about having to perform lifesaving care. Resources are avail- gency. PCPs should be able to provide BLS in the office
able to help guide practices in developing and implementing setting. The AHA’s certification for health-care providers
these trainings/tasks. (13)(14) The role of staff and providers is titled “BLS for Healthcare Providers.”
during code situations should be defined and documented. Other courses beneficial for pediatric PCPs include the
After a simulated experience, the use of debriefing can help following:
the team identify gaps in the care process as well as opportu-
nities for improvement. Videotaping the mock code provides • The pediatric advanced life support (PALS) course is of-
valuable feedback, education, and improvement opportunities. fered by the AHA for health-care providers who care for
When possible, including local EMS and disaster prepared- children and infants. The course teaches health-care pro-
ness teams can be a useful strategy. (7)(11) viders how to assess, identify, and treat respiratory dis-
tress/failure, cardiac arrest, shock, and arrhythmias.
EQUIPPING THE OFFICE • The neonatal resuscitation program was developed and
In an emergency, office staff as well as providers need to is maintained by the AAP. The focus of this program is
have access to the right equipment and medications. (15) basic resuscitation skills for newborns and infants.
Knowing the exact location of supplies is key to ensuring ef- • The Advanced Pediatric Life Support course is similar in
ficiency during an emergency. All office staff needs to be content to the PALS course, but the former is certified
trained to know where to locate resuscitation equipment. In by the AAP.
an office located in or near a hospital, basic airway equip-
ment may be the only thing necessary to initiate rescue. Education and recertification need to be periodically re-
However, if the office is located in a remote setting with viewed and renewed to ensure that providers maintain
prolonged emergency response times, efforts to stabilize the their skills and knowledge.

684 Pediatrics in Review

Downloaded from /pediatricsinreview/issue/44/12


by Hospital de Peditría 'Prof. Dr. Juan P. Garrahan' user
Documentation Children with immediate life-threatening asthma exacer-
Proper documentation is a key component of all patient en- bations may have dyspnea that prevents them from speak-
counters, which include office emergencies. Not only will ED ing, drowsiness or lethargy, bradycardia based on the child’s
providers depend on these notes to determine further man- age, paradoxical thoracoabdominal movements, and/or ab-
agement of the patient, but proper documentation is key to sence of wheezing and pulsus paradoxus, both of which
reducing the risk of litigation. Proper documentation helps suggest respiratory muscle fatigue. (16) Infants with imme-
determine whether the care provided during an office emer- diately life-threatening exacerbations may not cry at all.
gency meets the current standard of care. Documentation These infants and children may have minimal to no relief
can also help identify the learning needs of the office staff. with SABAs. (16)
It is best practice that all staff are trained in proper docu- Treatment should be started immediately while await-
mentation before an office emergency. During an office ing transfer to an acute care setting. The main goals of
emergency, stress levels are elevated. Therefore, training the treatment are to rapidly reverse airflow obstruction and
staff before an emergency occurs and using mock codes can correct hypoxemia, based on pulse oximetry reading.
increase staff confidence, reduce stress, and improve the A SABA, typically albuterol, should be administered im-
management of emergencies. It is important to keep medi- mediately, and supplemental oxygen should be started.
cation and fluid logs as well as a resuscitation log when (16) Although the onset of action for SABAs is typically
needed. Noting the exact timing of emergency events as less than 5 minutes in children with severe exacerbations,
well as the elapsed time during an emergency intervention note that the duration of action is typically significantly
is another critical element of proper documentation. shorter than that seen in more stable children. Therefore,
multiple doses of a SABA can and should be given be-
CONDITION-SPECIFIC MANAGEMENT OF
cause this will result in incremental bronchodilation.
COMMON PEDIATRIC EMERGENCIES
However, be aware that because of ventilation-perfusion
In the following subsections, common conditions that chil- mismatch, pulse oximetry O2 may decrease after SABA
dren experience in the primary care setting are reviewed. therapy. A SABA can be given continuously, and there is
Management of these conditions is subject to changes in evidence that in patients with severe asthma, continuous
best practices, and as such, it is critical that PCPs remain rather than intermittent administration of a SABA is the
current on diagnosis and treatment. most effective way to reverse airflow obstruction. (17) Fi-
nally, if available, the initial 3 doses of a SABA should be
Asthma
given in combination with ipratropium or another anticho-
Asthma is a common disease in children, although its pre-
linergic agent because multiple doses of this combination
sentation can be quite variable. When treating a child with
may decrease hospitalization rates when given on initial
an acute asthma exacerbation in the office, it is crucial
presentation. (18)
that the pediatrician be able to recognize not only the dis-
There is no single assessment tool used to monitor re-
ease itself but also the severity of the exacerbation. Asthma
sponse to treatment, although it is recommended that the
should be distinguished from other causes of respiratory
PCP assess multiple signs, including shortness of breath,
distress in children, including bronchiolitis, croup, pneu-
vital signs including pulse oximetry, alertness, use of ac-
monia, and asphyxiation, because their treatments differ.
cessory muscles, chest tightness, and timing and quality
Signs of severe asthma exacerbation in the child in-
of wheezing. (16)
clude dyspnea at rest (the child may speak in single words
rather than in complete sentences), agitation, preference
to sit upright, use of accessory muscles, inspiratory and Status Epilepticus
expiratory wheezing, tachypnea, tachycardia (with normal Although relatively rare, affecting 0.5% to 1% of children,
rates dependent on the child’s age), and pulsus paradoxus (19)(20) epilepsy remains a disease commonly encoun-
with a fall of blood pressure between 20 and 40 mm Hg tered in both the outpatient and inpatient pediatric set-
with inspiration. (16) Infants with severe asthma exacerba- tings. Therefore, it is crucial that the PCP be able to
tion will have difficulty feeding and will produce a weaker, recognize and appropriately manage seizure activity in the
shorter cry. Infants and children with severe exacerbations outpatient setting. There are several forms of seizures that
may often have only partial relief from frequently inhaled can be seen in a child with status epilepticus that the PCP
short-acting b-agonists (SABAs). (16) must be able to recognize, including convulsive seizures,

Vol. 44 No. 12 D E C E M B E R 2 0 2 3 685

Downloaded from /pediatricsinreview/issue/44/12


by Hospital de Peditría 'Prof. Dr. Juan P. Garrahan' user
nonconvulsive seizures, and repetitive focal seizures with- been having a seizure for 20 minutes, (21) at which point the
out altered consciousness. (21) child should be transferred to an acute care setting, where IV
A seizure is classified as brief if it lasts less than 5 minutes, placement can be secured and recommended medications
whereas a prolonged seizure lasts between 5 and 30 minutes. can be administered.
(21) A child is in status epilepticus when the child has 30 minutes
or more of continuous seizure activity or when the child experien- Dehydration
ces 2 or more consecutive seizures without an intermittent return Dehydration is a common worldwide cause of pediatric
to neurologic baseline. However, it is typically recommended that death. Dehydration can be associated with a multitude of pe-
treatment of the seizure be initiated after 5 minutes of epileptic diatric illnesses as a result of excessive fluid loss and poor
activity because children having a seizure for this duration are fluid intake. Therefore, the PCP must be comfortable with
highly likely to continue having a seizure without medical inter- recognizing dehydration in children, assessing the severity of
vention. (21) the dehydration, and recognizing associated complications
The main goal of therapy in the child with status epilepti- such as metabolic acidosis and electrolyte disturbances.
cus is to rapidly terminate both clinical and electrical epilep- A child with mild to moderate dehydration has typically
tic activity because prolonged seizure activity can lead to lost 3% to 9% of their body weight. (23) These children
permanent brain injury. Additional and equally important can present with normal mental status, but they can also
goals are to support respiration, maintain blood pressure, be fatigued, irritable, or restless. (23) Unlike children with
and identify and treat the underlying cause of the seizure. severe dehydration, these children are often thirsty. On
The first 5 minutes of treating seizure activity can be physical examination these children will be noted to have
thought of as the stabilization phase. During this time, the normal to decreased pulses and normal to elevated heart
goals of the PCP should be to stabilize the patient’s airway, rates. Their eyes are often sunken in, mucus membranes
breathing, and circulation. Supplemental oxygen should be are dry, and capillary refill is prolonged. Abnormal skin
administered by either nasal cannula or mask. If possible, turgor is often noted, with skin return to normal more
electrocardiographic monitoring is recommended, although than 2 seconds after being pinched. Extremities are often
the necessary equipment may not be available in the outpa- cool, and urine output and tear production are decreased.
tient setting. Neurologic examinations should be performed (23)
intermittently while the child is having a seizure. During this Children with severe dehydration have lost greater than
time, the blood glucose level should be assessed. If the glu- 9% of their body weight. On presentation they are lethar-
cose level is less than 60 mg/dL (<3.33 mmol/L), then the gic and unable to drink. On physical examination they will
blood glucose level should be corrected. (21) The intravenous be noted to have an elevated heart rate, although in the
(IV) route is preferred; however, in a suspected hypoglycemic most severe of cases they can present with bradycardia.
patient who is unable to swallow or is unconscious, intramus- (19) Pulses are typically weak or nonpalpable, and children
cular or intranasal glucagon may be administered. (22) will often be noted to be breathing deeply. Extremities are
After 5 minutes of seizure activity, children should, accord- often cold and mottled and may be cyanotic. Skinfold re-
ing to current recommendations, be given either IV loraze- coil is greater than 2 seconds. Urine output is minimal,
pam or IV diazepam. However, a child initially presenting to and these children do not make tears. (23)
the pediatric PCP office is highly unlikely to have an IV in It is important to note that diminished or absent
place, and rapid proper placement of an IV may not be feasi- bowel sounds can be an indication of hypokalemia, and
ble. In this case, either rectal diazepam or midazolam, given deep respirations can be seen with metabolic acidosis.
in intranasal, intramuscular, or buccal formulations, can be Hypokalemia and metabolic acidosis often accompany
given. (21) These medications should be administered as a dehydration. (23)(24)(25)
full single dose rather than divided into multiple small doses. The main goal of treatment is to rapidly replace the child’s
In the event that the child is nonresponsive to initial treat- fluid deficit. Although in the United States this is typically ac-
ment, first-line therapies should typically not be repeated, ex- complished with IV fluid replacement in the hospital setting,
cept in the case of IV lorazepam or diazepam, which are oral replacement therapy (ORT) has been shown to be equally
rescue medications that have been shown to be safe and ef- effective in children with mild to moderate dehydration sec-
fective. (21) Second-line therapies, including fosphenytoin, ondary to acute gastroenteritis, a common cause of dehydra-
valproic acid, and levetiracetam, are all recommended in the tion worldwide. (26) Therefore, because IV placement is
IV formulation and are typically given once the child has often not feasible in the outpatient facility, it is reasonable to

686 Pediatrics in Review

Downloaded from /pediatricsinreview/issue/44/12


by Hospital de Peditría 'Prof. Dr. Juan P. Garrahan' user
begin ORT when the child with acute gastroenteritis is mildly expiratory, with severe retractions and lethargy. (31) These
to moderately dehydrated. ORT can be effective even with children should be given dexamethasone, which if given in
children who are vomiting, although they must be observed oral form may be given once more if the child vomits, as
closely. One dose of ondansetron may be given before start- well as nebulized racemic epinephrine, which is often not
ing ORT in children with acute diarrhea and vomiting (age available in the outpatient setting. Therefore, any child pre-
$6 months as approved by the Food and Drug Administra- senting in the outpatient setting requiring racemic epineph-
tion [FDA]) (27) because use of ondansetron has been shown rine should be transferred to the ED. There, children will be
to result in improved hydration. (28) If the children do not re- observed for recurrence of respiratory distress after adminis-
spond to ORT, they will require IV placement for IV fluid tration of racemic epinephrine.
therapy. Although croup is the most common cause of upper air-
ORT should not be started in the child with paralytic il- way obstruction, the differential diagnosis includes other
eus or glucose malabsorption. (23) ORT should not be less frequent conditions requiring emergency treatment.
given as replacement of IV therapy in children with shock These conditions include epiglottitis, bacterial tracheitis, and
or severe dehydration because IV fluid resuscitation re- foreign body aspiration. Epiglottitis typically lacks the barky
mains first-line treatment for these conditions. (23) In the cough of croup, and affected children will appear anxious,
situations mentioned previously herein, an IV should be may be drooling, and will prefer sitting in an upright posi-
placed as soon as possible, even before transfer to the tion. (32) These children should be left undisturbed, kept
acute care setting if possible. Once an IV is placed, a nor- calm, and sent immediately to the ED. Bacterial tracheitis
mal saline bolus can be given to replace the fluid deficit. can also present with stridor and respiratory distress and,
Special care must be taken in caring for the severely similar to croup, also presents after upper respiratory tract
malnourished child with dehydration given the risks of infection symptoms. Bacterial tracheitis should be consid-
heart failure and pulmonary edema. Also, if glucose is ered in children who show little improvement after nebu-
given with fluid replacement, the refeeding syndrome and lized epinephrine, and given the complications associated
hypophosphatemia may occur. Apart from cases of shock, with bacterial tracheitis, these children should be transferred
these children should not receive emergency IV rehydra- to the ED. (32)
tion because they may be hypokalemic and hypernatremic.
(29)(30) Foreign Body Aspiration and Ingestion
Young children are at high risk for choking given both
Croup common exploratory behaviors and the smaller diameter
Laryngotracheobronchitis, generally known as croup, is a of their airways. Because air flow is inversely related to the
common cause of upper airway obstruction in children. Pre- radius of the airway to the fourth power, any decrease in
sentation varies, with signs and symptoms ranging from the cross-section of the airway can lead to respiratory dis-
mild to severe. Croup should be expected when a child is tress. Respiratory distress worsens as mucus and secre-
noted to have a barky cough, inspiratory stridor, hoarseness, tions accumulate around the foreign body, reducing the
or respiratory distress. (31) Typically, children will have 1 to airway’s radius even further. (33) Similar to croup, foreign
2 days of nonspecific upper respiratory tract infection signs body aspiration can also present with stridor. The onset of
and symptoms preceding the abrupt onset of a barky cough stridor is sudden and, unlike croup, does not typically pre-
and respiratory distress. (32) Mild disease courses are char- sent with prodromal viral-like signs and symptoms unless
acterized by a barky cough without stridor or respiratory the retained object has caused an associated secondary
distress. These children can be given oral dexamethasone bacterial infection. (32) There is typically no barky cough
0.6 mg/kg and discharged home with anticipatory guidance. or hoarseness, but there is often respiratory distress. (34)
(31) Children who present with moderate disease, character- Children will require radiographic evaluation. Children
ized by frequent coughing, retractions at rest, and normal with a highly suspected tracheobronchial foreign body
mental status along with stridor, can be given dexametha- must undergo bronchoscopy and, therefore, should be
sone (31) 0.6 mg/kg as a 1-time dose. In this case, the intra- transferred to the nearest medical center. Pending trans-
muscular route is preferred over oral. After dexamethasone fer, the airway should be acutely managed per BLS guide-
administration, children should be observed for improve- lines. (35)
ment for approximately 4 hours. Severe disease presents Children who have ingested foreign objects are often
with stridor at rest that is mostly inspiratory but may be asymptomatic, but they may present with drooling, coughing,

Vol. 44 No. 12 D E C E M B E R 2 0 2 3 687

Downloaded from /pediatricsinreview/issue/44/12


by Hospital de Peditría 'Prof. Dr. Juan P. Garrahan' user
gagging, vomiting, hematemesis, pain, food refusal, and fussi- certifications, equipment, and medications as well as in
ness, among other signs and symptoms. (34) Physical exami- training office staff and parents to identify emergency situa-
nation findings are often normal, but when examining the tions and react appropriately in such instances.
child, the PCP should assess for crepitus or swelling of the
neck, findings that are concerning for esophageal perforation,
Summary
inspiratory stridor, or expiratory wheezing, which are concern-
ing for airway obstruction, or abdominal tenderness, which is 1. Increasing readiness of the primary care office
concerning for intestinal obstruction or perforation. (34) Coins and staff, including necessary education, equip-
are the most commonly ingested foreign body but must be ment, and training, has been shown to improve
distinguished from button batteries, which appear to have a patient outcomes. (Based primarily on consensus
double ring on radiographs with a step-off on lateral views. due to lack of relevant clinical studies) (4)
(34) Children with highly suspected foreign body ingestion re- 2. The most common emergencies in the pri-
quire radiographic evaluation so that high-risk foreign bodies, mary care setting have been identified and
such as button batteries, which require urgent management, ranked in order of most to least common. (Based
are not misdiagnosed. Therefore, it is recommended that the primarily on consensus due to lack of relevant
child with suspected foreign body ingestion be transferred to clinical studies) (7)
a medical center for further evaluation and treatment. 3. Implementation of mock codes has been shown
to increase staff and practitioner confidence and
DISCUSSION reduce anxiety of having to perform lifesaving
Although this article provides general guidance and best care. Resources exist to help guide practices in
practices for the pediatric provider and the clinical facility, developing and implementing these trainings/
each primary care setting is unique. As such, the capability tasks. (Based primarily on consensus due to lack
to diagnose and treat an emergency in the office setting of relevant clinical studies) (10)(12)
must be evaluated and determined before such an emer-
gency. For offices close to a tertiary care site, stabilization
and transfer of care is often the safest approach. However, KEY PRACTICE POINTS
practices in rural settings or practices without adequate ac- • Recent studies reveal the positive effects of pediatric of-
cess to tertiary care facilities for whatever reason must be fice readiness on ensuring good outcomes for children
trained and equipped to handle immediate stabilization, pro- treated in emergency situations. (4)
viding supplemental oxygen if available, and delivering sub- • Investigators have examined the effect of improved of-
sequent life support until such time as a safe transfer can fice readiness on the quality of pediatric emergency
be made. Maintaining emergency readiness requires time care. Findings suggest a direct link between office readi-
and effort from a dedicated staff member or team, recom- ness and superior outcomes. (4)
mended training for staff, maintaining staff competency,
and having a fully functional emergency cart with appropri- SUGGESTED QI PROJECT TOPICS COULD
ate medical supplies that have not expired. Without ade- INCLUDE
quate resources dedicated to these ongoing efforts, office 1. Improving the emergency training of staff.
readiness will be very difficult to achieve. 2. Improving adherence to protocol.
3. A process improvement for keeping supplies up to date.
CONCLUSION
Pediatric emergencies can present in primary care offices Acknowledgments
frequently and without warning. It is imperative that the Thank you to Dr Rani S. Gereige for his review of the man-
uscript and to Tatiana Consuegra from Nicklaus Children’s
pediatric PCP and office staff have proper training, not
Hospital for her support throughout the process.
only in triaging patients and redirecting care of such
emergencies but also in effectively handling the initial References and teaching slides for this
phase of the emergency situation to improve patient article can be found at
https://doi.org/10.1542/pir.2021-005076.
outcome. It is crucial that pediatric PCPs invest in

688 Pediatrics in Review

Downloaded from /pediatricsinreview/issue/44/12


by Hospital de Peditría 'Prof. Dr. Juan P. Garrahan' user
APPENDIX A (12)(13)

Recommended Office Recommended Equipment Recommended Equipment


Emergency Drugs for Pediatric Office Essentials for Pediatric Office Nonessentials
Essentials Airway management Airway management
Albuterol (solution for inhalation: Bag-valve-mask (450 and 1,000 mL) Endotracheal tubes (uncuffed 2.5–5.5;
1.25 mg/3 mL; 2.5 mg/3 mL; 5 mg/mL cuffed 6.0–8.0) or supraglottic devices
or MDI 90 lg per puff) (LMA, Combitube)
Epinephrine (1:1,000, IM autoinjector: Clear oxygen masks, breather and Esophageal intubation detector or end-
0.1, 0.15, 0.3 mg) nonrebreather, with reservoirs (infant, tidal carbon dioxide detector
child, adult)
Oxygen Nebulizer (or MDI with spacer/mask) Laryngoscope blades (0–2 straight and
2–3 curved)
Nonessentials Oropharyngeal airways (sizes 00–5) Laryngoscope handle (pediatric, adult)
with extra batteries, bulbs
Activated charcoal Oxygen delivery system Nasogastric tubes (sizes 6–14F)
Antibiotics Pulse oximeter Nasopharyngeal airways (sizes 12–30F)
Anticonvulsant agents (diazepam, Suction device, tonsil tip, bulb syringe Magill forceps (pediatric, adult)
lorazepam)
Atropine sulfate (0.1 mg/mL) Miscellaneous equipment and supplies Stylets (pediatric, adult)
Corticosteroids (parenteral/oral) Cardiac arrest board/backboard Suction catheters (sizes 5–16F) and
Yankauer suction tip
Diazepam (IV) (injection: 5 mg/mL) or Color-coded tape or preprinted drug Miscellaneous equipment and supplies
(rectal) (rectal gel: 2.5, 5.0, 7.5, 10.0, doses
12.5, 15.0, 17.5, or 20.0 mg)
Diphenhydramine (parenteral: Sphygmomanometer (infant, child, adult, Automated external defibrillator with
50 mg/mL) thigh cuffs) pediatric capabilities
Epinephrine (1:10,000) Splints Heating source (overhead warmer/
infrared lamp)
Glucagon (injection: 1 mg/mL) Sterile dressings Spot glucose test
Insulin (injection: 100 U/mL) plus Stiff neck collars (small/large)
dextrose (10%; injection: 100 mg/mL)
Ipratropium (nebulized solution: Vascular access and fluid management
0.25 mg/mL or MDI 18 lg per puff)
Naloxone (0.4 mg/mL) Arm boards, tape, tourniquet
Ondansetron (injection: 2 mg/mL; ODT: Butterfly needles (19–25 gauge)
4 mg; solution: 0.8 mg/mL)
Ranitidine (injection: 25 mg/mL; syrup: Catheter-over-needle device
15 mg/mL) (14–24 gauge)
Sodium bicarbonate (4.2%) Intraosseous needles (16 and 18 gauge)
Fluids (nonessentials) Intravenous tubing, microdrip
Dextrose (glucose) (injection: D10W,
D25W, D50W)
0.9% sodium chloride, Ringer lactate
solution (250, 500, 1,000 mL)

D10W/D25W/D50W 5 10%/25%/50% dextrose in water, IM 5 intramuscular, IV 5 intravenous, LMA 5 laryngeal


mask airway, MDI 5 metered-dose inhaler, ODT 5 orally disintegrating tablet.

Vol. 44 No. 12 D E C E M B E R 2 0 2 3 689

Downloaded from /pediatricsinreview/issue/44/12


by Hospital de Peditría 'Prof. Dr. Juan P. Garrahan' user
PIR QUIZ

1. A pediatric resident is working in your clinic during their Community


Pediatrics rotation. You explain that the clinic’s emergency supply cart is
very different from the hospital’s emergency department cart based on
expected office presentations. Which one of the following is the most
common type of childhood illness presenting to the primary care pediatric
office setting?
A. Anaphylaxis.
B. Dehydration.
C. Head trauma.
D. Respiratory distress.
E. Seizures.
REQUIREMENTS: Learners can
2. A 3-year-old boy is brought to the clinic by his parents because of concern take Pediatrics in Review quizzes
that the child is having another “asthma attack.” The clinician assesses the and claim credit online only at:
child to determine the severity of this asthma exacerbation episode. The http://pedsinreview.org.

patient is placed on supplemental oxygen because of low oxygen saturation To successfully complete 2023
and is administered the first dose of a short-acting b-agonist. Of the Pediatrics in Review articles for
following signs or symptoms, which one is the least likely to be seen in a AMA PRA Category 1 Credit™,
severe asthma exacerbation in this patient? learners must demonstrate a
minimum performance level of
A. Difficulty speaking in full sentences. 60% or higher on this
B. Dyspnea at rest. assessment. If you score less
C. Preference to lay flat on the examination table. than 60% on the assessment,
you will be given additional
D. Rapid respiratory rate for age.
opportunities to answer
E. Use of accessory respiratory muscles. questions until an overall 60%
3. A 9-year-old with hemiplegic cerebral palsy and a known seizure disorder or greater score is achieved.

begins having a seizure during a health supervision visit in your office. The
This journal-based CME activity
parent realizes that the child did not get yesterday’s and this morning’s is available through Dec. 31,
doses of levetiracetam. Despite successfully stabilizing the child and giving 2025, however, credit will be
supplemental oxygen, the seizure has persisted for more than 5 minutes. recorded in the year in which
Administration of which one of the following therapies is the most the learner completes the quiz.

appropriate immediate next step in the management of this patient?


A. Fosphenytoin.
B. Intramuscular glucagon.
C. Rapid-onset benzodiazepine.
D. The morning levetiracetam dose. 2023 Pediatrics in Review is
E. Valproic acid. approved for a total of 30
Maintenance of Certification
4. The parents of a high school football player bring him to your office after he (MOC) Part 2 credits by the
collapsed at school. As part of the football team’s summer conditioning American Board of Pediatrics
program, there are extended periods of exercise but no body contact drills. (ABP) through the AAP MOC
The patient has no recollection of the event and is frustrated with being in Portfolio Program. Pediatrics in
Review subscribers can claim up
the office. His weight is down approximately 7% from the weight recently
to 30 ABP MOC Part 2 points
recorded as part of his preparticipation physical. During today’s physical upon passing 30 quizzes (and
examination, which one of the following clinical signs and symptoms are claiming full credit for each
most likely to be observed in this patient? quiz) per year. Subscribers can
start claiming MOC credits as
A. Bradycardia. early as November 2023. To
B. Decreased bowel sounds. learn how to claim MOC points,
C. Deep respirations. go to: https://publications.aap.
org/journals/pages/moc-credit.
D. Lethargy.
E. Prolonged capillary refill time.

690 Pediatrics in Review

Downloaded from /pediatricsinreview/issue/44/12


by Hospital de Peditría 'Prof. Dr. Juan P. Garrahan' user
5. A 2-year-old is brought to the office by his parents because of worsening
“noisy breathing.” The patient seems to have had progressive stridor during
the past 2 hours. There were no preceding respiratory symptoms or
associated fever. He has started to drool and cough more since their arrival
in the office. Which one of the following is most likely to represent your
working diagnosis in this case?
A. Asthma exacerbation.
B. Bacterial tracheitis.
C. Croup.
D. Epiglottitis.
E. Foreign body aspiration.

Vol. 44 No. 12 D E C E M B E R 2 0 2 3 691

Downloaded from /pediatricsinreview/issue/44/12


by Hospital de Peditría 'Prof. Dr. Juan P. Garrahan' user
CME: THREE PATIENT CASES

Fever: Three Patient Cases


Presented here are three patients who have a common chief complaint. All three cases have discussions on presentation,
the differential diagnosis, and management that collectively serve as a Review article. Following the three cases, an expert
weighs in in a short commentary with 5 questions for CME credit.

Case 1: Fever in a 40-day-old Infant


Stelios Kasikis, MD,* Maame Hayfron, MD,* Despoina Galetaki, MD,* Risa Bochner, MD*
*Department of Pediatrics, State University of New York Downstate Health Sciences University and New York City Health and Hospitals/Kings County,
Brooklyn, NY

PRESENTATION
A previously healthy, 40-day-old girl presents with 1 day of fever (100.8 F
[38.2 C]). She is sleeping more than usual and having more frequent bowel
movements but normal consistency. She has no cough, congestion, eye redness,
respiratory distress, emesis, rash, or change in appetite or urine output. She was
born at term via normal spontaneous vaginal delivery to a mother of Japanese
descent and a father of Western European descent. Maternal prenatal infection
screening was negative, including group B Streptococcus. She had a routine post-
natal course, is formula fed and breastfed, and is growing well.
On presentation she is febrile (temperature, 100.5 F [38.1 C]). Her heart rate
is 176 beats/min, respiratory rate is 48 breaths/min, and blood pressure is
93/53 mm Hg. She is well-appearing, with no dysmorphic features. The anterior
fontanelle is open and flat. There is no conjunctivitis or nasal congestion. Oral
mucosa appears pink and moist, without lesions. There is no rash or lymphade-
nopathy. Her neurologic examination findings are normal. The remainder of the
examination findings are normal.
Initial laboratory results are as follows: white blood cell count, 13,640/mL
(13.64 × 109/L) with a normal differential count; hemoglobin level, 13.3 g/dL
(133 g/L); platelet count, 349 × 103/mL (349 × 109/L); C-reactive protein level,
1.1 mg/dL (11 mg/L); procalcitonin level, 18 ng/dL (0.18 mg/L) (reference range,
0–50 ng/dL [0–0.5 mg/L]); cerebrospinal fluid (CSF) white blood cell count, 5/mL
(0.05 × 109/L); glucose level, 54 mg/dL (3 mmol/L); protein level, 0.051 g/dL
(0.51 g/L); and Gram-stain negative. Comprehensive metabolic panel and urinalysis
findings are normal. The respiratory viral panel, including severe acute respiratory
syndrome coronavirus 2, adenovirus, and enterovirus, is negative. Chest radiography is
normal. She is admitted to the hospital and started on empirical antibiotics pending AUTHOR DISCLOSURE FOR CASE 1
Drs Kasikis, Hayfron, Galetaki, and
blood, urine, and CSF culture results. Interval development of additional physical exami- Bochner have disclosed no financial
nation findings and increasing inflammatory markers support her ultimate diagnosis. relationships relevant to this article. This
After being admitted, she continued to spike high-grade fevers (maximum tempera- commentary does not contain a
discussion of an unapproved/
ture of 103.8 F [39.2 C]). On hospital day 2 she developed a diffuse, polymorphous mac- investigative use of a commercial
ulopapular rash (Fig 1) that started on her torso and eventually expanded to the product/device.

692 Pediatrics in Review

Downloaded from /pediatricsinreview/issue/44/12


by Hospital de Peditría 'Prof. Dr. Juan P. Garrahan' user
conjunctivitis, cracked erythematous lips, and erythema of the
palms and soles), in the absence of an alternative source and
failure to improve with antibiotic drug therapy, led to the diag-
nosis of complete KD. (2)

The Condition
KD is a rare systemic inflammatory disease of unknown
etiology. The leading theory involves an immune-mediated
inflammatory response initiated by an unknown, likely in-
fectious, trigger in a genetically susceptible child. (3) The
highest annual incidence rates are reported in Japan, with
359 per 100,000 children younger than 5 years affected in
2018. (4) The annual number of hospitalizations for KD
Figure 1. A maculopapular rash over the trunk, face, and extremities.
in the United States ranged from 1,652 to 1,796 in 2016
to 2019 and decreased to 1,382 in 2020 amid the COVID-19
extremities, face, and inguinal area. Her palms and soles became
pandemic. (5) In the United States, children of all racial and
erythematous, and she developed bilateral, nonpurulent, limbic-
ethnic backgrounds are impacted. (5) Children younger than
sparing conjunctival injection. Her lips became erythematous
5 years and males are predominantly affected. (3) KD in chil-
and cracked. Repeated laboratory studies revealed increasing in-
flammatory marker values, with a maximum C-reactive protein dren younger than 12 months is quite rare. (6)(7)(8)(9)(10)
level of 14.5 mg/dL (145 mg/L) and a maximum procalcitonin One study of 443 patients diagnosed as having KD found
level of 207 ng/dL (2.07 mg/L). that 13% were younger than 12 months and just 3% were
younger than 6 months. (4)(8) A hurdle in the diagnosis of
DISCUSSION KD in young infants is that 75% present as incomplete KD
with few or no classic symptoms. (8) KD should be considered
Differential Diagnosis
in infants with prolonged fever even if there are no accompa-
Diagnostic considerations include infectious diseases (staphy-
nying symptoms. (1)(2)
lococcal or streptococcal toxin-mediated disease, measles,
adenovirus, Epstein-Barr virus, cytomegalovirus, parvovirus
Treatment/Management
B19, enterovirus, rickettsial infections, leptospirosis), inflamma-
Infants younger than 6 months with KD are at particularly
tory disorders (systemic juvenile idiopathic arthritis, reactive
high risk for coronary artery aneurysms (CAAs) and giant
arthritis), and dermatologic conditions (drug reactions such
CAAs, with poor outcomes. (2)(4)(8) This is in part attrib-
as Stevens-Johnson syndrome). (1) After initial blood, urine,
uted to late diagnosis given atypical presentations of KD
and CSF cultures were negative, further investigation included
human herpesvirus 6, parvovirus B19, Epstein-Barr virus, in infants with subtle or transient clinical findings aside
cytomegalovirus, human immunodeficiency virus, a CSF from fever. (2)(4)(8) Timely therapy with intravenous immu-
meningitis encephalitis panel, and a stool gastrointestinal noglobulin (IVIg) is recommended by illness day 10 or as
pathogen panel, which were all negative. Measles was less soon as possible after diagnosis and improves outcomes.
likely given maternal immunity with passive immunity to (2)(8) IVIg significantly decreases the risk of CAA in chil-
the infant. Multisystem inflammatory syndrome in chil- dren with KD. (11) KD treatment also includes acetylsalicylic
dren was considered but thought unlikely with negative se- acid (ASA) at moderate (30–50 mg/kg per day) to high
vere acute respiratory syndrome coronavirus 2 polymerase (80–100 mg/kg per day) doses for anti-inflammatory activity
chain reaction and nucleoprotein antibody. Drug reactions, and at low doses (3–5 mg/kg per day) for antiplatelet activity.
including Stevens-Johnson syndrome, were also consid- (2) Practices vary across institutions regarding ASA dosing
ered but thought unlikely given no medication exposure and duration of treatment. (2) In addition, there is no evi-
before symptom onset and that mucus membrane was dence that treatment with ASA reduces CAAs. (2) Some cen-
limited to only cracked lips. ters treat patients with moderate- to high-dose ASA until
they have been afebrile for 48 to 72 hours before transition-
Actual Diagnosis ing to a low dose. (2) Treatment is typically continued for
The presence of high-grade fever for 5 days with 4 of 5 clinical 6 to 8 weeks until follow-up echocardiography is performed
criteria for Kawasaki disease (KD) (rash, bilateral nonpurulent to evaluate for CAAs. (2) If no CAAs are identified, ASA is

Vol. 44 No. 12 D E C E M B E R 2 0 2 3 693

Downloaded from /pediatricsinreview/issue/44/12


by Hospital de Peditría 'Prof. Dr. Juan P. Garrahan' user
typically discontinued, but ASA may be continued indefi- scientific statement suggests moderate- to high-dose ASA
nitely if a CAA is identified. (2) Patients at high risk for until fever resolution, followed by low-dose ASA, moderate-
CAAs may benefit from adjuvant therapies in addition to to high-dose ASA was not used in this case due to the
IVIg and ASA. (2) Options for adjuvant therapies include rapid improvement in clinical symptoms and resolution
corticosteroids, infliximab (anti–tumor necrosis factor a of fever after IVIg treatment. A baseline echocardiogram
monoclonal antibody), and etanercept (soluble tumor ne- was obtained before initiating treatment and did not reveal
crosis factor receptor). (2) A 2017 meta-analysis including any abnormal findings other than a small patent foramen
7 trials and 922 children with KD found that corticosteroid ovale. Her inflammatory markers returned to normal lev-
use during the acute phase of illness was associated with a els during the following days. She was eventually dis-
decreased incidence of CAAs. (12) The benefits of cortico- charged on ASA treatment until cardiology follow-up and
steroids were greatest for children with high risk scores
was later noted to have peeling of her hands and feet. Re-
and prolonged corticosteroid courses. (12) In addition, high-
peated echocardiography at 2 and 6 weeks revealed no
risk patients with CAAs seen on baseline echocardiograms
abnormalities.
had less CAA progression with treatment of corticosteroids
or infliximab in addition to IVIg. (13) Patients who have
persistent or recrudescent fevers at least 36 hours and less Lessons for the Clinicians
than 7 days after completion of the first IVIg infusion are • Pediatricians should have a high index of suspicion for
considered IVIg resistant. (2) Treatment options for IVIg- Kawasaki disease when evaluating infants with unex-
resistant KD include a second IVIg infusion, IVIg plus plained fever for more than 5 days. (1)
prednisolone, infliximab, cyclosporin, anakinra, cyclophos- • Kawasaki disease is a diagnosis of exclusion. It is impor-
phamide, and plasma exchange. (2)
tant to evaluate for the infectious, inflammatory, and
dermatologic etiologies that compose the differential di-
Patient Course
agnoses for Kawasaki disease. (1)
Our patient was treated with a single dose of 2 g/kg IVIg,
after which she defervesced and was started on low-dose References for this article can be found at
oral ASA. Although the American Heart Association KD https://doi.org/10.1542/pir.2021-005397.

Case 2: Ankle Pain 2 Days after a Febrile Illness in a


9-year-old Girl
Maya Heled Akiva, MD, MSc,*† Christos Karatzios, MD,*† Marina I. Salvadori, MD*†
*Montreal Children’s Hospital, Montreal, Quebec, Canada
†McGill University, Montreal, Quebec, Canada

PRESENTATION
A previously healthy 9-year-old girl presents to the emergency department with a 1-
day history of ankle pain and inability to bear weight. Four days before presentation,
she had fever, sore throat, vomiting, and a light cough that lasted for 2 days. As she
defervesced, she developed a diffuse rash over her trunk and arms described by the
mother as slightly raised, 2-mm nonpruritic papules. A throat culture, performed as
an outpatient, was negative for b-hemolytic streptococci. As the rash began to fade,
AUTHOR DISCLOSURE FOR CASE 2
approximately 24 hours after its appearance, she started complaining of right ankle Drs Akiva, Karatzios, and Salvadori have
pain and was reluctant to walk. There was no history of trauma. She lives in a Lyme disclosed no financial relationships
disease–endemic region and acknowledges having 2 dogs at home. When presenting relevant to this article. This commentary
does not contain a discussion of an
to the emergency department due to increasing ankle pain, she is afebrile (99.1 F unapproved/investigative use of a
[37.3 C]). On examination she is calm and cooperative; there is no rash, oral lesions, commercial product/device.

694 Pediatrics in Review

Downloaded from /pediatricsinreview/issue/44/12


by Hospital de Peditría 'Prof. Dr. Juan P. Garrahan' user
tonsillar erythema, or exudates. The right ankle is swollen, septic and postinfectious arthritis with the consideration
tender, and warm, with mild erythema and limited range of of Lyme arthritis according to the local epidemiology.
motion on extension and flexion. The remainder of the physi- Other less common etiologies would include rheumato-
cal examination findings are normal. logic diseases such as IgA vasculitis (Henoch-Sch€ onlein
Laboratory findings reveal a white blood cell (WBC) purpura), juvenile idiopathic arthritis, and lupus. The
count of 8,500/mL (8.5 × 109/L) (reference range [RR], most common bacterial causes for septic arthritis in this
4,500–13,000/mL [4.5–13.0 × 109/L]), a hemoglobin level age group are Staphylococcus aureus and Streptococcus pyo-
of 11.7 g/dL (117 g/L) (RR, 11.5–15.5 g/dL [115–155 g/L]), genes. (1) In younger children (ages 6 months to 4 years)
and a platelet count of 288 × 103/mL (288 × 109/L) (RR, with septic arthritis, K kingae is a common pathogen that
150–450 × 103/mL [150–450 × 109/L]). Her C-reactive protein is frequently associated with a preceding upper respiratory
level is 7.5 mg/dL (75 mg/L) (RR, 0–0.5 mg/dL [0–5 mg/L]), illness. (1)(2) Lyme arthritis can also present similarly;
erythrocyte sedimentation rate is 48 mm/h, C3 level is ele- however, in this case, serologic test results were negative.
vated (197 mg/dL [1.97 g/L] [RR, 79–152 mg/dL (0.79–1.52 Postinfectious arthritis can be divided into 3 categories:
g/L)]), C4 level is normal (34 mg/dL [0.34 g/L] [RR, 17–47 postviral arthritis (described with parvovirus B19, rubella,
mg/dL (0.17–0.47 g/L)]), and antistreptolysin O level is 136 and hepatitis B infections), poststreptococcal arthritis as
IU/mL (RR, #200 IU/mL). Radiography of the right tibia/ part of or independent of rheumatic fever, and reactive
fibula, ankle, and foot shows no fracture or bony abnormality, arthritis, commonly seen in patients with genetic predis-
with moderate perimalleolar soft tissue swelling over the lateral position (HLA-B567). (3) In this case, clinical improve-
malleolus and a small to moderate tibiotalar joint effusion. Mag- ment was noted after aspiration and irrigation of the
netic resonance imaging with contrast of the right ankle shows joint and initiation of antibiotics, suggesting a septic ar-
no evidence of osteomyelitis, small tibiotalar and subtalar joint thritis. This was validated by the growth of a gram-nega-
effusions, mild signs of acute synovitis, and focal tenosynovitis tive bacillus from the synovial fluid.
of the flexor tendons. From a bedside aspiration and irrigation K kingae is a gram-negative bacillus but is uncommon
of the right ankle, 10 mL of turbid, reddish synovial fluid is ex- in this age group. Other gram-negative bacilli are much
less common as pathogens causing septic arthritis in im-
tracted and sent for bacterial culture and cell count, revealing a
munocompetent hosts (Table 1). These often are linked to
WBC count of 9,790/mL (9.79 × 109/L), with 93% neutrophils,
animal exposures. Identification of the organism isolated
and a red blood cell count of 0.40 × 106/mL (0.40 × 1012/L). On
was unsuccessful by means of mass spectrometry, a phe-
initial synovial Gram-stain, no organisms are noted.
notypic approach to bacterial identification. Therefore, the
Although the patient was afebrile and had a normal WBC
sample is then sent for nucleic acid 16S ribosomal RNA
count, her physical examination findings and high inflamma-
tory markers suggest an infectious etiology. She is, therefore, sequencing for genomic identification. In this process, the
admitted for concern of septic arthritis and started on intrave- 16S subunit gene of the well-conserved bacterial ribosome
nous (IV) cefazolin. After 48 hours of IV cefazolin, she starts is first amplified and sequenced, then matched through
to bear weight; she is switched to oral cephalexin 1,000 mg/ databases of available sequences, revealing the identifica-
dose 3 times a day for a planned duration of 3 weeks. She was tion of the bacterium. This technique allows for recogni-
afebrile in the hospital, and her C-reactive protein level de- tion of bacteria that may not be able to be identified by
creased to 1.63 mg/dL (16.3 mg/L) at discharge. After approxi- phenotypic methods. (4)
mately 80 hours of incubation, a preliminary growth from
the synovial fluid was reported as gram-negative bacilli. The Actual Diagnosis
synovial multiplex polymerase chain reaction was negative; Two weeks after discharge she still had a minimal limp.
this assay detects the following bacteria: Staphylococcus au- When pressed on possible exposures, the family disclo-
reus, Streptococcus pyogenes, Streptococcus group B, Streptococ- ses that the girl has 2 pet rats not mentioned during the
cus pneumoniae, and Kingella kingae. hospitalization. Septic arthritis in the context of rat expo-
sure is suspicious for a zoonotic etiology, and the differ-
DISCUSSION ential diagnosis in these cases should include rat-bite
Differential Diagnosis fever (caused by either Streptobacillus moniliformis or Spi-
When assessing a child with arthritis subsequent to a fe- rillum minus), leptospirosis, and Pasteurella spp infec-
brile illness, the main differential diagnosis includes tions. (5)

Vol. 44 No. 12 D E C E M B E R 2 0 2 3 695

Downloaded from /pediatricsinreview/issue/44/12


by Hospital de Peditría 'Prof. Dr. Juan P. Garrahan' user
Table 1. Gram-Negative Bacilli That Cause Septic Arthritis in Children
RECOMMENDED FIRST-LINE
BACILLUS GRAM-STAIN AGE GROUP EXPOSURES/RISK FACTORS ANTIBIOTIC TREATMENT
Kingella kingae Gram-negative <5 y Preceding upper respiratory Cefazolin
(most coccobacillus tract infection and/or oral
common) ulcers
Salmonella spp Gram-negative All ages Sickle cell anemia Ceftriaxone
bacillus Connective tissue disease
Reptile exposures
Haemophilus Gram-negative <5 y Unimmunized Ceftriaxone or cefuroxime or
influenzae spp coccobacillus amoxicillin-clavulanic acid
Pasteurella spp Gram-negative All ages Animal exposure (especially Frequently polymicrobial and
coccobacillus after adjacent dog and cat should include
bites or scratches) antistaphylococcal and
antistreptococcal coverage
Amoxicillin-clavulanic acid or
cefuroxime
Pseudomonas Gram-negative All ages Intravenous drug use Antipseudomonal b-lactam ±
aeruginosa bacillus aminoglycoside
Brucella spp Gram-negative All ages Livestock exposure Doxycycline or trimethoprim-
coccobacillus Unpasteurized dairy products sulfamethoxazole 1 gentamicin ±
rifampin
Bartonella Gram-negative All ages (more Cat exposure Doxycycline or azithromycin ±
henselae coccobacillus common in children) rifampin
Streptobacillus Gram-negative More common in Rat exposure (bites) Penicillin G
moniliformis bacillus children Contaminated oral ingestion

A few days after her follow-up, the results from the 16s is essential for a rat-bite fever septic arthritis diagnosis,
ribosomal RNA sequencing identified the isolated bacte- which may affect the choice and duration of antibiotics. (8)
rium from the synovial fluid as S moniliformis, giving the The recommended treatment for rat-bite fever is IV penicil-
diagnosis of rat-bite fever septic arthritis. lin G for 5 to 7 days with a switch to oral for a total 2- to-4-
week course. (6) Ceftriaxone has also shown good results
The Condition and can be considered as an alternative. There are limited
There are 2 pathogens that cause rat-bite fever: S monilifor- data about the use of other cephalosporins. (6)(8) S monili-
mis (a gram-negative rod common in North America) and formis septic arthritis is rare, there are no clear guidelines
S minus (a spirochete seen predominantly in Asia). Trans- for its management, and regimens described in the litera-
mission of S moniliformis is usually by a rat bite. Contami- ture are variable. The expected prognosis for streptobacillus
nated food and water outbreaks are also described, but septic arthritis is excellent; all published cases, regardless
this mode of transmission is less common. Historically, the regimen chosen, had successful outcomes with resolu-
people living in poverty were the population at risk, with tion of symptoms. (5)(8)
children accounting for approximately 50% of the cases in
the United States. Recently, adoption of rats as pets has
Patient Course
become popular, causing a shift in the epidemiology to in-
Once diagnosed as having confirmed rat-bite fever septic ar-
clude pet owners (also usually children), pet store workers,
thritis, treatment was changed to amoxicillin for an additional
and laboratory workers. (6) The typical presentation in-
4 weeks. At the end of therapy, she had no sign of infection or
cludes fever, rash, chills, and polyarthralgia. (7)(8) Polyar-
residual pain and was back to doing her regular activities. The
thralgia and migratory polyarthritis may last for months to
pet rats were removed from the household because reinfection
years even after adequate treatment and is believed to be
and infection of other household members are possible.
reactive or immune-mediated. (8) Complications of rat-
bite fever can vary and include pneumonia, meningitis,
myocarditis, pericarditis, endocarditis, vasculitis, and sep- Lessons for the Clinician
tic arthritis, which may involve multiple joints. Although 1. Exotic pets such as rats, other rodents, and reptiles are
joint involvement is a prominent feature of the clinical syn- increasing in popularity. Such exposure may give
drome, septic arthritis is rare. Microbiological confirmation important clues to disease etiology, and clinicians

696 Pediatrics in Review

Downloaded from /pediatricsinreview/issue/44/12


by Hospital de Peditría 'Prof. Dr. Juan P. Garrahan' user
should be aware that patients may not offer this the use of molecular techniques such as synovial
information without specific questioning. multiplex polymerase chain reactions or 16s ribosomal
2. An index of suspicion for less common causes of RNA sequencing. Molecular testing should also be
arthritis should be considered in cases of patients with considered if identification of an isolated bacterium is
animal exposures causing zoonotic arthritis. not readily identified.
3. In cases of suspected septic arthritis, synovial sampling
has unequivocal diagnostic importance. In complicated References for this article can be found at
or atypical culture-negative cases, one should consider https://doi.org/10.1542/pir.2022-005593.

Case 3: Acute Right Lower Quadrant Abdominal Pain in a


2-year-old
Sagar D. Mehta, MD,*† Carrie Ng, MD*†
*Division of Pediatric Emergency Medicine, Emory University School of Medicine, Atlanta, GA
†Children's Healthcare of Atlanta, Atlanta, GA

PRESENTATION
A previously healthy 2-year-old girl presents to the pediatric emergency depart-
ment (ED) with 4 days of right lower quadrant abdominal pain. Three days ago,
she had presented to an outside adult ED where she was diagnosed as having
constipation despite having regular soft stools and was sent home with a poly-
ethylene glycol prescription. The day before our present presentation she devel-
oped a fever to 102.5 F (39.2 C) and was referred to the local children’s ED by
her pediatrician. The family denies any vomiting, and she has had 1 episode of
nonbloody diarrhea since starting the polyethylene glycol. She has a slightly de-
creased appetite but continues to snack on foods and is making her normal
number of wet diapers. Her twin sister recently recovered from a likely mild vi-
ral gastrointestinal illness. There is no history of abdominal trauma, and the pa-
tient had been growing and developing well before this illness.
Her vital signs are notable for a temperature of 101.3 F (38.5 C), heart rate of
154 beats/min, blood pressure of 121/85 mm Hg, respiratory rate of 38 breaths/min,
and oxygen saturation of 100% on room air. On physical examination she is alert
and interactive but appears uncomfortable. She has equal and reactive pupils,
clear tympanic membranes, and clear nares and oropharynx with moist mucous
membranes. Her lungs sound clear bilaterally, and her cardiac examination is no-
table for regular rate and rhythm without any murmurs. Her abdominal examina-
tion is notable for normoactive bowel sounds, focal right lower quadrant
tenderness with guarding and mild distention, without any appreciated masses or
hepatosplenomegaly, although examination was limited by patient guarding. She
AUTHOR DISCLOSURE FOR CASE 3
has pain with coughing, and a negative Rovsing sign. Her genitourinary examina- Drs Mehta and Ng have disclosed no
tion findings are normal, and she has a sexual maturity rating of 1. She has no financial relationships relevant to this
rashes or bruises. She is alert and appropriately oriented for age, and the remainder article. This commentary does not
contain a discussion of an unapproved/
of her neurologic examination is nonfocal. Point-of-care ultrasonography (POCUS) investigative use of a commercial
of the right lower quadrant point of maximal tenderness could not visualize the ap- product/device.

Vol. 44 No. 12 D E C E M B E R 2 0 2 3 697

Downloaded from /pediatricsinreview/issue/44/12


by Hospital de Peditría 'Prof. Dr. Juan P. Garrahan' user
Figure 2. Right lower quadrant abdominal point-of-care ultrasonography demonstrating echogenic free fluid (star). The bowel (triangle), psoas muscle
(square), and iliac vessel (circle) are indicated.

pendix but shows a large area of echogenic free fluid (Fig 2). appendix but shows fluid and debris in the right lower quad-
Laboratory data are notable for a white blood cell count of rant. While scanning the area they note an abnormal appear-
13,640/mL (13.6 × 109/L) (reference range, 5,000–15,500/mL ance adjacent to the kidney and so extend the imaging study
[5.0–15.5 × 109/L]) with 56.7% neutrophils, a hemoglobin to the right upper quadrant and find a lobular vascular het-
level of 9.5 g/dL (95 g/L) (reference range, 11.5–13.5 g/dL erogeneous mass measuring 6.8 cm superior to the right
[115–135 g/L]), a mean corpuscular volume of 84 mm3 (84 kidney (Fig 3). An abdominal computed tomographic (CT)
fL) (reference range, 75–87 mm3 [75–87 fL]), a C-reactive scan with contrast is ordered and is notable for a vascular bi-
protein level of 6.0 mg/dL (60 mg/L) (reference range, lobed mass between the liver and the right kidney (anterior
<1.0 mg/dL [<10 mg/L]), and negative urinalysis results. to the kidney) with hypodense and hyperdense components,
Given a high concern for perforated appendicitis with de- some of which appear hemorrhagic. The mass appears to in-
veloping phlegmon and abscess, she is sent for radiology ap- vade the right abdominal wall and appears separate from the
pendix ultrasonography, which is unable to visualize the liver. The right adrenal gland is normal in appearance, and

–5width.
Figure 3. Radiology abdominal ultrasonography demonstrating the size of the mass. 15height, X5length, X

698 Pediatrics in Review

Downloaded from /pediatricsinreview/issue/44/12


by Hospital de Peditría 'Prof. Dr. Juan P. Garrahan' user
Figure 4. Abdominal axial computed tomographic scan of the mid-abdomen. The heterogenous mass (star), mildly distended bowel loops (square), and
normal kidneys (triangles) are visualized.

the mass does not involve the right kidney. Complex fluid is appendicitis or gastroenteritis, (2)(3) it is crucial to deter-
noted in the right and left lower quadrants (Fig 4). She is ad- mine which patients require additional imaging after a
mitted for further evaluation, including a comprehensive POCUS or a limited radiology ultrasonography. In this
metabolic panel and tumor markers, and underwent subse- case, the finding of echogenic free fluid in the pelvis
quent biopsy of the mass, with pathologic analysis confirm- should prompt concern for possible hemoperitoneum and
ing the diagnosis. consideration of full abdominal imaging such as complete
abdominal ultrasonography or cross-sectional imaging such
DISCUSSION as magnetic resonance imaging or CT.
The peak incidence of pediatric malignant abdominal tu-
Differential Diagnosis
mors is between 1 and 5 years of age. Neuroblastoma and
The differential diagnosis of acute right lower quadrant abdom-
Wilms tumor are the most common intra-abdominal tumors
inal pain in a 2-year-old is very broad, with common causes in-
in the pediatric population, composing 6% and 5% of all
cluding gastroenteritis, constipation, intussusception, urinary
childhood cancers, respectively, (4) with less common causes
tract infection, appendicitis, and mesenteric adenitis. Less com-
including leukemia/lymphoma, hepatic tumors, ovarian tu-
mon etiologies include ovarian cysts or torsion, testicular tor-
mors, germ cell tumors, and soft tissue sarcomas. Wilms tu-
sion, acute bowel obstruction, inflammatory bowel disease,
mor and neuroblastoma are found more commonly in
and abdominal tumors. (1) The patient’s history with acute-on-
infants and toddlers, and leukemia or lymphomas of the liver,
set abdominal pain and recent fever raised suspicion in the dif-
spleen, or retroperitoneal lymph nodes occur more commonly
ferential diagnosis to possible infectious or inflammatory
in older children. (5) Wilms tumor may present after sponta-
etiologies such as urinary tract infection, appendicitis, mesen-
neous rupture or rupture after blunt abdominal trauma caus-
teric adenitis, and gastroenteritis. The right lower quadrant PO- ing intra-abdominal bleeding and abdominal pain. (6)
CUS finding of echogenic free fluid along with the elevated
white blood cell count and C-reactive protein level raised con- Actual Diagnosis
cern for perforated appendicitis with developing phlegmon and The patient’s subsequent laboratory values were notable for
abscess. a normal b–human chorionic gonadotropic tumor marker
Because some abdominal tumors present with fever, and an elevated a-fetoprotein (AFP) level of 34,184 ng/mL
anorexia, vomiting, and/or diarrhea mimicking acute (34,184 mg/L) (reference range, 1–11 ng/mL [1–11 mg/L]).

Vol. 44 No. 12 D E C E M B E R 2 0 2 3 699

Downloaded from /pediatricsinreview/issue/44/12


by Hospital de Peditría 'Prof. Dr. Juan P. Garrahan' user
Pathologic analysis of the peritoneal fluid and abdominal namely, localized, regional, and distant disease, compared with
mass biopsy both showed a diagnosis of yolk sac tumor. postpubertal tumors of the same stage. (7)(9) Posttreatment
There was suspicion that a hemorrhage of the tumor led to monitoring is conducted with AFP monitoring and repeated
the patient’s fever and imaging findings of complex fluid in imaging as indicated. (7)(12)
the lower abdominal quadrants.
Patient Course
The Condition The patient was formally diagnosed as having a stage III
Yolk sac tumor falls under a classification of germ cell tu- abdominal yolk sac tumor and received 4 cycles of chemo-
mors, which originate from the primordial germ cell and therapy with bleomycin, etoposide, and cisplatin. She had
encompasses teratomas, germinomas, yolk sac tumors, em-
residual mass resection 2 months later, with repeated
bryonal carcinomas, and choriocarcinomas. (7) Germ cell tu-
pathologic analysis showing no viable residual tumor. A
mors compose approximately 3.5% of childhood cancers for
repeated scan 6 months later did not show residual or re-
children younger than 15 years. (8) Malignant germ cell tu-
current disease, and her repeated AFP levels were within
mors in especially young children (<4 years) are primarily
normal limits. She continues to do well and follows up in
all yolk sac tumors and may arise from a preexisting tera-
the hematology/oncology clinic for routine monitoring.
toma. They have a bimodal frequency of distribution, with
the first peak observed in the first 4 years after birth and a
LESSONS FOR CLINICIANS
second peak observed after 12 years, coinciding with the on-
set of puberty. (9) Their clinical presentation can be variable, • Oncologic abdominal presentations can mimic other
with a tendency to grow rapidly over weeks to months, and acute abdominal etiologies, such as appendicitis.
may lead to development of abdominal enlargement and • Echogenic free fluid in the pelvis of a young child is ab-
acute abdominal pain. (10)(11) normal and needs to be investigated further with com-
Diagnostic evaluation should include a complete blood plete abdominal ultrasonography and/or further cross-
cell count with differential count, serum electrolytes, blood sectional imaging.
urea nitrogen, creatinine, urinalysis, and tumor markers • The peak incidence of malignant abdominal tumors in
such as human chorionic gonadotropic, AFP, and lactate pediatrics is between ages 1 and 5 years.
dehydrogenase. The AFP level is commonly elevated in • Neuroblastoma and Wilms tumor are the most common
yolk sac tumors and indicates the presence of malignant intra-abdominal tumors in pediatrics, and yolk sac tu-
components; however, it is not tumor specific. Ultrasonog- mor, a type of germ cell tumor, occurs more rarely.
raphy is often chosen as the initial imaging modality, fol- • Abdominal mass evaluation should include a com-
lowed by advanced imaging such as CT or magnetic plete blood cell count, a comprehensive metabolic
resonance imaging as clinically indicated. (7) panel, a urinalysis, tumor markers (human chorionic
gonadotropic, a-fetoprotein, lactate dehydrogenase),
Management ultrasonography, and consideration of cross-sectional
Management depends on the stage of the tumor and may imaging.
involve surgical resection and chemotherapy with bleomy-
cin, etoposide, and cisplatin. The survival rate is higher References for this article can be found at
for prepubertal pediatric yolk sac tumors at each stage, https://doi.org/10.1542/pir.2023-005993.

700 Pediatrics in Review

Downloaded from /pediatricsinreview/issue/44/12


by Hospital de Peditría 'Prof. Dr. Juan P. Garrahan' user
COMMENTARY

Commentary: Fever Is Your Friend


Henry M. Adam, MD
Pediatrics in Review Associate Editor

Fever, the theme of our 3 patient cases, is often looked on as a disease in itself, a misconception we have helped foster
by the language we use: scarlet and yellow fevers, typhoid and Rocky Mountain spotted fevers, also rheumatic, hay, den-
gue, cat-scratch, and rat-bite fevers. Reinforcing this impression of fever as a disease, rather than as a symptom or sign
of an underlying illness, contributes to parents’ fear of fever and their frequent compulsion to treat it.
An element of the human inflammatory response, fever in contrast to hyperthermia rarely threatens a child’s well-
being. Given that it is an energy-consuming process, metabolically costly, fever almost certainly has a protective effect or
it would not have persisted through its long evolutionary history. A controlled rise in the body’s temperature mediated
by the anterior hypothalamus, fever is a reaction to any insult that sets off the body’s inflammatory response. Akin to a
thermostat, the hypothalamic set point regulates body temperature. Some inciting factor, a viral infection most often in
children, stimulates the release of cytokines that act as pyrogens, which increase levels of prostaglandin E2 at the ante-
rior hypothalamus, causing a rise in the set point. With actual body temperature lower than the new set point, the pa-
tient at the onset of fever feels chilled and the body’s response is to shiver, generating more internal heat, and to
vasoconstrict at the skin and close down sweat glands, reducing heat loss: in effect, a controlled turning up the furnace
and shutting the windows.
Hyperthermia is fundamentally different from fever. An unregulated rise in temperature above the set point, it can
result from the body producing too much heat (thyroid storm), from the inability to disperse heat (an overbundled
baby), or from a combination of both (inappropriate exertion on a hot, humid day). Instead of the chill accompanying
the onset of fever, the body experiences intense flushing as blood vessels dilate and sweat glands go into overdrive to
shed heat to the environment: the opposite of fever, the furnace is heating out of control and the windows are thrown
wide open.
Evidenced by the epidemic of heat stroke during this past summer’s heat waves, hyperthermia can send body temper-
ature dangerously high. But fever, a homeostatic process, is kept within physiologic limits, rarely rising above 106 F
(41.1 C): a counter-acting element of the febrile response is the release of cryogens that check the effect of pyrogens as
fever rises. In fact, only approximately 0.05% of pediatric emergency patients present with a temperature higher than
106 F (>41 C). And although it seems intuitive that the higher a child’s fever the more likely is a serious bacterial infec-
tion, even at high temperatures a viral source is far more common. Temperature itself becomes dangerous only when it
reaches 107 F or higher ($41.7 C), in which case the underlying fever is likely to be exacerbated by a component of hy-
perthermia, often dehydration.
Fever, it turns out, has a long history widespread among animals as a protective response. When they become in-
fected, some fish and lizards, which are cold-blooded, seek out a warmer environment where their body temperatures
rise, in effect an induced fever. This behavior has been shown to confer a survival benefit that can be reversed by the ad-
ministration of antipyretic agents. Fever inhibits the growth of many viruses and bacteria, and hinders the absorption of
iron, without which many pathogenic bacteria cannot survive. It promotes the activity of neutrophils and the production
of superoxides, and it enhances the proliferation of T cells and the action of interferon. As a metabolically costly phe-
nomenon that increases the expenditure of energy by 7% to 10% for each rise in temperature of approximately 2 F
(–16.7 C), fever’s long and widespread endurance argues for it as a protective process with survival benefit.
Why then are we, both parents and pediatricians, so adamant about treating fever? Barton Schmitt coined the term
fever phobia in his study reporting parents’ overwhelming confusion about fever, their pervasive beliefs that it is harmful,

Vol. 44 No. 12 D E C E M B E R 2 0 2 3 701

Downloaded from /pediatricsinreview/issue/44/12


by Hospital de Peditría 'Prof. Dr. Juan P. Garrahan' user
can cause brain damage, and can rise to 110 F (43.3 C) or And that reassurance is less likely to be accepted when
higher if not treated. Not surprising, then, that most pa- their child is febrile than as part of routine counseling: fe-
rents worried “lots” about fever, considered it high even at ver is actually a protective process, not in itself a threat
less than 102 F (<39 C), and treated even temperatures in that without treatment can climb to frightening heights;
the normal range with antipyretic agents. Frequently, pa- sensible care means simple measures such as adequate
rents cited doctors and nurses as their primary source of hydration, attention to ambient room temperature, and
knowledge about fever, an assertion made credible by a avoiding overbundling. Our message should be that it is
survey of AAP members in Massachusetts, two-thirds of the underlying illness, not the fever, that may put their
whom regularly suggested treating fevers less than 102 F child in jeopardy, and we need to teach about the symp-
(<39 C) because they can themselves be dangerous, and one- toms and behaviors that should trigger concern and signal
quarter of whom believed that a fever of 104 F (40 C) could the need for professional involvement.
cause brain damage and even death (Pediatrics. 1992;90: When the illness underlying a fever puts a child at risk,
851–854). clearly it must be treated; but how to deal with the fever,
A temperature of 100.4 F (38 C) or greater is usually beyond attention to hydration and ambient conditions, is a
cited as signifying a fever. But as with any physiologic question of judgment. Intervening to make a child more
measure, the limit of normal is not defined by a single comfortable is certainly appropriate, and the decision to
temperature but by a range of values that account for indi- give an antipyretic should not be based on a specific tem-
vidual variations, fluctuations by time of day, and differ- perature but rather on how the child looks and behaves:
ences related to age as well as to how and where the for many children, a fever of 104 F (40 C) or even higher
temperature is taken. Body temperature varies throughout is tolerated without ill effect, whereas others become
the day, being higher late in the afternoon and early in the droopy and upset when their temperature barely reaches
evening than in the morning or at night. Infants have the threshold of fever. Another reason often given for treat-
higher body temperature than older children and adults. ing a fever is that lowering the temperature helps separate
Rectal temperature is the closest to core temperature, de- children with trivial illness from those with serious infections.
fined as blood’s temperature in the pulmonary artery, but In reality, how a febrile child responds to a dose of antipyretic
aside from the discomfort that limits its use in many chil- drug does not reliably distinguish dangerous from nonthreat-
dren it can be inaccurate because of improper technique ening disease. A healthy child with a transitory viral illness is
or the presence of stool. As a rule, oral temperature is ap- not endangered by a fever, but a child with an underlying
proximately 1 F (–17.2 C) less than rectal temperature, but chronic illness or a serious acute infection may not deal well
this difference varies from person to person, and its preci- with the metabolic stress that fever can impose: tachycardia
sion relies on the cooperation of the patient; recent intake and tachypnea, as well as higher oxygen consumption and
of cold or hot liquids or foods can also affect its accuracy, greater insensible water losses, can compromise a child who
as can tachypnea in an anxious child. Axillary tempera- is anemic, septic, or in shock, or who is at special risk from a
ture, although easy and convenient to take, does not have disease that strains homeostasis.
a reliable correlation with rectal temperature so can be dif- Aside from antipyretic medication, another common
ficult to interpret. Infrared thermometers, despite their approach to treating fever is physical cooling. Acetamino-
convenience, do have limitations. Faulty placement in the phen and nonsteroidal anti-inflammatory drugs are effec-
ear canal and the presence of ear wax can limit the reli- tive antipyretics because they inhibit the production of
ability of tympanic thermometers, and forehead thermom- prostaglandin E2, which then lowers the hypothalamic set
eter readings can be affected by ambient conditions such point back toward normal, turning off the stimulus for the
as direct sunlight, cold room temperature, or sweaty skin. body to generate heat. In contrast, physical measures such
Least dependable are so-called fever strips and pacifier as cool baths or sponging do not affect the set point,
thermometers. which remains high. Instead, they draw heat from the
The management of fever should begin before the child body: rather than turning the furnace off, these measures
is febrile. In giving anticipatory advice, we pediatricians open the windows wide. The hypothalamus detects a widen-
should acknowledge our role in generating fever phobia. ing gap between its high set point and the body’s lowering
When we recommend the routine use of antipyretics for temperature, with the paradoxical effect that it responds by
any rise in temperature, we undermine our reassurance to raising the set point still higher. Once the bath or sponging is
parents that they need not worry about the fever itself. done, the stimulus remains to produce more heat by raising

702 Pediatrics in Review

Downloaded from /pediatricsinreview/issue/44/12


by Hospital de Peditría 'Prof. Dr. Juan P. Garrahan' user
the metabolic rate and causing muscles to shiver, which may alarming, but very rarely is it dangerous. Furthermore, no
cause the child discomfort and lead to a rebound in the fever. convincing evidence establishes that this intervention is ef-
Physical cooling can have an appropriate role in the man- fective: the seizure can precede the fever, putting causality
agement of fever when the triggering illness makes the into question so that preventing the fever may not abort
child dangerously vulnerable to the metabolic stress result- the seizure. Once a child has had a febrile seizure, the
ing from the rise in temperature. In such a case, the combi- urge to prevent a recurrence becomes more intense. The risk
nation of cooling with an antipyretic brings the fever down of a recurrent seizure increases the lower the temperature
quickly while also lowering the set point to prevent a re- with the first seizure. But low-grade fevers can be difficult
bound rise in temperature. to detect, so that parents have less sense of control and risk
A difficult issue is the role of antipyretic drugs in pre- becoming more aggressive about taking temperatures and
venting febrile seizures, which are most common in chil- resorting to an antipyretic medication—neither is healthy to
dren 6 months to 6 years of age, the same age group to a young, developing child. And again, without supporting
most often have benign viral infections. Advising parents evidence, it is questionable that even 24-hour ongoing an-
routinely to treat the fever of these self-limited illnesses to tipyretic treatment can prevent another seizure. What pa-
prevent a febrile seizure contributes both to the fear of fe- rents clearly deserve from us is reassurance that neither
ver and to their anxiety about the risk a febrile seizure fever nor a febrile seizure is a fault in their care or likely to
poses to their child. Granted, a febrile seizure is certainly threaten their child’s well-being.

With this last commentary, Dr Henry Adam concludes his 28 years of contributions as Associate Editor for the Pediatrics
in Review feature “In Brief.” We are indebted to Dr Adam for his thoughtful guidance and for his wit and wise insight at
the end of In Brief articles over the years, especially his focus on parental concerns, from his first In Brief comment in
July 1995: “ … parents are likely to need help overcoming the almost predictable revulsion of the thought of their child
having ‘worms’; counseling may be at least as important as mebendazole,” to his last on fever: “What parents clearly
deserve from us is reassurance that neither fever nor a febrile seizure is a fault in their care or likely to threaten their
child’s well-being.” Thank you, Henry, for a job well done.

Joseph A. Zenel, MD, Editor in Chief

Vol. 44 No. 12 D E C E M B E R 2 0 2 3 703

Downloaded from /pediatricsinreview/issue/44/12


by Hospital de Peditría 'Prof. Dr. Juan P. Garrahan' user
PIR QUIZ

PIR QUIZ (for IOS Cases plus commentary)


1. A 6-week-old male infant with prolonged fever is brought to the emergency
department (ED) by his parents. Following an extensive work-up, the infant is
diagnosed with Kawasaki Disease (KD) and is admitted to the hospital for further
management. The treating clinician explains to the family the importance of the
timeliness of certain interventions on patient outcomes and disease
complications. Which of the following is the most appropriate immediate next
step in management of this patient with the highest impact on outcomes?
A. High dose acetylsalicylic acid (ASA).
B. Intravenous fluids. REQUIREMENTS: Learners can
take Pediatrics in Review quizzes
C. Intravenous immunoglobulins (IVIG). and claim credit online only at:
D. Low dose ASA. http://pedsinreview.org.
E. Watchful waiting.
To successfully complete 2023
2. The patient in the vignette in question #1 received IVIG and was placed on Pediatrics in Review articles for
high-dose ASA. A baseline echocardiogram showed no evidence of coronary AMA PRA Category 1 Credit™,
artery aneurysms. On day 4 after completing the IVIG infusion, the patient learners must demonstrate a
continued with daily spiking fevers. Which of the following is the most likely minimum performance level of
60% or higher on this
cause of the persistent fevers in this patient?
assessment. If you score less
A. Drug fever. than 60% on the assessment,
B. IVIG-resistance. you will be given additional
opportunities to answer
C. Superimposed bacterial infection.
questions until an overall 60%
D. Superimposed fungal infection. or greater score is achieved.
E. Superimposed viral infection.
This journal-based CME activity
3. A 6-year-old girl is brought to the clinic by her parents because of one-day
is available through Dec. 31,
history of fever, chills, and generalized joint pains. On the day of 2025, however, credit will be
presentation, she was noted to have a diffuse rash over her trunk, arms, and recorded in the year in which
legs. She also started complaining of right knee pain and limping. On the learner completes the quiz.
physical examination she is afebrile and is alert, non-toxic, and in no acute
distress. There is a diffuse blanching rash over her trunk and extremities. The
right knee is swollen, erythematous, warm, and tender to touch with
limitation of flexion due to pain and swelling. Small puncture wounds are
noted slightly above the right knee. The parents report that the child just
received a pet rat for her birthday a few weeks prior and that the wounds 2023 Pediatrics in Review is
approved for a total of 30
are due to a bite by the pet rat. The clinician suspects rat-bite fever as the
Maintenance of Certification
diagnosis. Which of the following pathogens is the most likely cause of the (MOC) Part 2 credits by the
right septic knee in this patient? American Board of Pediatrics
(ABP) through the AAP MOC
A. Group A beta-hemolytic Streptococcus.
Portfolio Program. Pediatrics in
B. Salmonella typhi. Review subscribers can claim up
C. Staphylococcus aureus. to 30 ABP MOC Part 2 points
D. Streptobacillus moniliformis. upon passing 30 quizzes (and
E. Streptococcus pneumoniae. claiming full credit for each
quiz) per year. Subscribers can
start claiming MOC credits as
early as November 2023. To learn
how to claim MOC points, go to:
https://publications.aap.org/
journals/pages/moc-credit.

704 Pediatrics in Review

Downloaded from /pediatricsinreview/issue/44/12


by Hospital de Peditría 'Prof. Dr. Juan P. Garrahan' user
4. The mother of a 15-month-old boy noted a “fullness” on the left side of the
abdomen while bathing him. The child has had normal growth and
development with no diarrhea or constipation and normal appetite. She is
worried about a malignancy and brings him to the clinic today for further
investigation. Among the following causes of intra-abdominal tumors, which
one is most likely to be the cause in a child of this patient’s age?
A. Hepatoblastoma.
B. Leukemia.
C. Lymphoma.
D. Rhabdomyosarcoma.
E. Wilms Tumor.
5. In a child who presents with an intra-abdominal mass and is diagnosed with
a yolk sac tumor, which of the following is the most accurate statement
about yolk sac tumors in children?
A. Elevated alpha-fetoprotein is a specific tumor marker.
B. Peak incidence is in children younger than 2 years of age.
C. Radiation therapy is the mainstay of treatment.
D. Survival of prepubertal is higher than postpubertal pediatric yolk sac
tumors of the same stage.
E. They are slow growing tumors.

Vol. 44 No. 12 D E C E M B E R 2 0 2 3 705

Downloaded from /pediatricsinreview/issue/44/12


by Hospital de Peditría 'Prof. Dr. Juan P. Garrahan' user
INDEX OF SUSPICION

New-Onset Jaundice in an 11-month-old Boy


Sean E. Healton, MS,* Devon G. Lawrence, MD,† Noah J. Elkins, MD,† Leya Schwartz, MD,† Patricia A. Hametz, MD†
*Albert Einstein College of Medicine, Bronx, NY

The Children’s Hospital at Montefiore, Bronx, NY

PRESENTATION
A previously healthy 11-month-old boy presents to the emergency department
with new-onset jaundice. One day before admission the patient was picked up
from an extended stay with his grandparents. At that time, the parents noted yel-
lowing of the skin, fussiness, and abdominal pain relieved by 1 dose of acet-
aminophen (exact dose unknown). The patient is otherwise well, without
vomiting, diarrhea, change in color of stool or urine, fatigue, pruritus, changes
in activity, or decreased oral intake or urine output. He has no history of jaun-
dice during the neonatal period or infancy, no recent sick contacts or illnesses,
no recent travel, and according to his mother, no new known exposures. He
takes formula prepared with tap water and eats some table foods, such as soup.
The infant’s mother is unaware of any new introductions to his diet or different
water consumption while at his grandparents. The patient’s medical history,
birth history, and development are noncontributory. His immunizations are up
to date, and he has not yet received his hepatitis A vaccination. He lives at home
with his mother and father, who are originally from Ecuador but have lived in
the United States for the past 6 years. There is a family history of unspecified
liver disease in the patient’s father, maternal grandfather, and paternal aunt.
There is no known family history of splenectomy or early gallbladder diseases.
Physical examination reveals a well-appearing, well-developed infant in no
acute distress. Vital signs are within normal limits for his age. There is scleral
icterus and jaundice from head to abdomen, in addition to hepatomegaly. The
remainder of the examination findings are normal, without abdominal tenderness
or masses. Right upper quadrant ultrasonography demonstrates a borderline en-
larged, hyperechoic liver. Laboratory findings are significant for anemia (hemoglo-
bin level, 8.4 g/dL [84 g/L] [reference range, 11.3–12.5 g/dL (113–125 g/L)]) and
indirect hyperbilirubinemia (total bilirubin level, 9.7 mg/dL [165.91 mmol/L]
[reference range, <1.2 mg/dL (<20.52 mmol/L)]; direct bilirubin level, 0.3 mg/dL
[5.13 mmol/L] [reference range, <0.5 mg/dL (<8.55 mmol/L)]). Transaminase and
albumin levels are within normal limits.

DISCUSSION
AUTHOR DISCLOSURE: Drs Healton,
Differential Diagnosis Lawrence, Elkins, Schwartz, and Hametz
The differential diagnosis for jaundice, specifically secondary to indirect hyperbi- have disclosed no financial relationships
relevant to this article. This commentary
lirubinemia, includes etiologies related to increased production of bilirubin,
does not contain a discussion of an
reduced uptake by hepatocytes, and decreased conjugation; an appreciation of unapproved/investigative use of a
bilirubin metabolism (Fig 1) offers a systematic approach to the evaluation. commercial product/device.

706 Pediatrics in Review

Downloaded from /pediatricsinreview/issue/44/12


by Hospital de Peditría 'Prof. Dr. Juan P. Garrahan' user
RBCs
Increased production

Hemolytic anemias
Extrinsic - RBC destruction
1
- Immune mediated or microangiopathic
- Infection Bilirubin
- Toxins Reduced uptake
2
Heme - Reduced hepatic blood flow
Intrinsic - RBC fragility - Drugs
proteins
- Enzyme deficiencies
- RBC membrane defects
- Hemoglobinopathies
1 Production
Decreased conjugation Bilirubin
- Genetic disorders 3 2 Uptake
- Drugs Conjugated
bilirubin 3 Conjugation

Enterohepatic
recirculation
Conjugated
bilirubin

Unconjugated
bilirubin

Excretion
(urobilinogen,
sercobilinogen)

Figure 1. Overview of bilirubin metabolism. Critical steps where derangement results in indirect hyperbilirubinemia are numbered 1, 2, and 3; common
etiologies underlying each are highlighted in red. See the text and Table 1 for more details. RBC 5 red blood cell. Figure created using BioRender.com.

Overproduction of bilirubin is most commonly secondary to due to right-sided heart failure, Budd-Chiari syndrome) (1)
hemolysis and release of heme, which is converted to biliru- and certain drugs (rifampin, probenecid). (2) Decreased or
bin. The differential diagnosis for hemolytic anemia (Table 1) absent bilirubin conjugation occurs secondary to genetic
includes autoimmune etiologies, hemoglobinopathies, heredi- disorders that affect uridine diphosphoglucuronate glucur-
tary enzyme deficiencies, and erythrocyte membrane defects. onosyltransferase activity, including Crigler-Najjar (types I
Uptake of bilirubin by hepatocytes can be impaired by reduc- and II) and Gilbert syndromes, or certain drugs (some
tion in hepatic blood flow (eg, cirrhosis, hepatic congestion viral protease inhibitors used in human immunodeficiency

Table 1. Overview of Common Etiologies of Hemolytic Anemia


CATEGORY EXAMPLES
Intrinsic to RBCs
Enzymatic defects G6PD deficiency
Glycolytic enzyme deficiencies: pyruvate kinase deficiency, phosphofructokinase deficiency, etc
RBC membrane defects Hereditary spherocytosis
Hereditary elliptocytosis
Hereditary pyropoikilocytosis
Hemoglobinopathies HgbSS, HgbSC, HgbSβ thalassemia
a- and β-thalassemias
Other Paroxysmal nocturnal hemoglobinuria (rare in children)
Extrinsic to RBCs
Microangiopathic hemolytic Hemolytic uremic syndrome, thrombotic thrombocytopenic purpura, ADAMTS13 deficiency
anemia Artificial heart valves
Disseminated intravascular coagulation; HELLP (hemolysis, elevated liver enzymes, low platelets) syndrome
Immune-mediated Autoimmune hemolytic anemia (eg, in systemic lupus erythematosus)
Delayed hemolytic transfusion reactions
Mycoplasma and Epstein-Barr virus (both due to cold agglutinin)
Infectious Sepsis, malaria, babesiosis
Systemic disease Liver disease, uremia, extensive burn injuries
Toxins and venoms Clostridia infections, snake bites, spider bites inducing spherolytic hemolysisa

G6PD 5 glucose-6-phosphate dehydrogenase, HgbSS/SC/Sβ 5 hemoglobin SS/SC/Sβ, RBC 5 red blood cell.
a
Mechanism not fully understood, thought to be complement-mediated.

Vol. 44 No. 12 D E C E M B E R 2 0 2 3 707

Downloaded from /pediatricsinreview/issue/44/12


by Hospital de Peditría 'Prof. Dr. Juan P. Garrahan' user
the onset of jaundice—habas, Spanish for fava beans, a popular
snack in Ecuador. Fava beans contain the compounds vicine
and convicine, which, when metabolized to divicine and iso-
uramil, cause significant oxidative stress to RBCs in G6PD
deficiency. Further genetic testing showed 2 hemizygous
G6PD mutations resulting in the G6PD A– variant.

The Condition
G6PD deficiency is the most common erythrocyte enzyme
disorder. (5) A link between the ingestion of fava beans
and disease (“favism”) was known in the time of Pythagoras,
but the modern origins of the discovery of G6PD deficiency
Figure 2. Peripheral blood smear. Light microscopy of peripheral blood lie in the observation, made in the early 20th century, that
from the patient demonstrating bite cells (white arrows) and schistocytes certain patients developed hemolytic anemia while being
(red arrow).
treated for malaria. (6) The genetic basis of the disorder
virus treatment—indavinir and atazanavir; some chemothera- was elucidated in the 1950s as an X-linked recessive disor-
peutic agents, tyrosine kinase inhibitors—erlotinib, nilotinib). der; most of those affected are male.
(3)(4) Physical examination findings are often nonspecific, Nicotinamide adenine dinucleotide phosphate (NADPH)
and, thus, laboratory findings guide diagnosis. is involved in multiple enzymatic reactions that mitigate the
physiologic oxidative burden in the red blood cell (RBC) and
Diagnosis is, therefore, crucial for protection from these free oxygen
Subsequent laboratory findings were consistent with a hemo- radicals. NADPH formation occurs only via reduction of the
lytic anemia (haptoglobin level, <10 mg/dL [<100 mg/L] glucose-6-phosphate by G6PD, which is normally activated
[reference range, 30–200 mg/dL (300–2,000 mg/L)]; reticu- by oxidative stress. The RBC is devoid of DNA, RNA, and
locyte count, 4.3% [reference range, 1.0%–3.1%]). The reticu- protein synthesis, and, therefore, the G6PD is consumed as
locyte index was calculated to be 2.07 based on a normal the lifespan of an RBC passes, thus making older erythro-
hematocrit level of 34% for an 11-month-old boy and a matu- cytes particularly sensitive to oxidative stress in the setting of
ration factor of 1.5, which is appropriate proliferation in the reduced G6PD activity. Acute hemolysis can occur in pa-
setting of hemolysis. Hemoglobin electrophoresis results tients with G6PD deficiency variants when oxidizing free
were normal; evaluation for infectious etiologies, including radicals are generated, such as after ingesting certain drugs,
Epstein-Barr virus, was negative; and direct antiglobulin eating fava beans, or during an acute infection, (7) and are
(Coombs) test results were negative. Peripheral blood smear unopposed in the absence of NADPH formation (Fig 3).
demonstrated schistocytes and bite cells (Fig 2), consistent Common causative agents are listed in Table 2. Infection
with hemolytic anemia secondary to glucose-6-phosphate de- is the primary precipitant of hemolytic episodes in those
hydrogenase (G6PD) deficiency. The G6PD screen was noted with an established diagnosis. Although common and long-
to be deficient, which was confirmed by quantitative G6PD studied, the nonhematopoietic effects of G6PD deficiency
testing (4.8 U/L [reference range, 7.0–20.5 U/L]), supporting remain underexplored; a recent review suggests the possible
this diagnosis. On further questioning, the parents discovered involvement of this enzyme deficiency in neurodegenerative
that the grandparents had fed the patient a new food before disorders. (8)

G6P NADP GSH Oxidants (H2O2)

e
Glutathione e
Glutathione
G6PD
D e
reductase peroxidasee

6-phosphogluconate NADPH GSSG H2O


Figure 3. The role of glucose-6-phosphate dehydrogenase (G6PD) in intracellular nicotinamide adenine dinucleotide phosphate (NADPH) production.
G6PD is necessary to convert NADP to NADPH in red blood cells (RBCs), a key step in the pathway for reduction of oxidants; without G6PD, RBCs are sub-
jected to oxidative damage. NADPH is then required to convert glutathione disulfide (GSSG) to glutathione (GSH), which finally reduces oxidants to water
(H20) via glutathione peroxidase. H202 5 hydrogen peroxide.

708 Pediatrics in Review

Downloaded from /pediatricsinreview/issue/44/12


by Hospital de Peditría 'Prof. Dr. Juan P. Garrahan' user
Table 2. Common Triggers to Avoid in Glucose-6-phosphate dehydrogenase Deficiency
CATEGORY NAME
Antibiotics Dapsone
Trimethoprim-sulfamethoxazolea
Nitrofurantoin
Fluoroquinolones (ciprofloxacin, moxifloxacin, ofloxacin, norfloxacin)
Antimalarials Primaquine, pamaquine, chloroquine
Analgesics Aspirina
Other drugs Methylene blue
Dimercaprol
Probenecid
Rasburicase
Foods Fava beans (also called broad beans)
Chemicals Naphthalene (moth balls)
Henna dyes
a
Agents that usually do not cause clinically significant hemolysis in A– variant.

Of note, diagnosis of G6PD deficiency during an acute Patient Course


episode of hemolysis may rely on clinical suspicion alone. The patient was admitted to the hospital and developed
Enzymatic activity of G6PD is higher in reticulocytes than worsening anemia during admission, with a nadir hemo-
in mature erythrocytes. Therefore, during hemolysis— globin level of 6.4 mg/dL (0.064 g/L) on hospital day 3.
when mature erythrocytes are hemolyzing and reticulo- Transfusion was not required, however, because he re-
cytes are high—G6PD levels may be falsely normal. In situa- mained clinically stable with steadily decreasing serum in-
tions with high clinical suspicion for G6PD deficiency and a direct bilirubin levels. Anemia improved on hospital day 4
negative screen, it is recommended that testing is repeated to a hemoglobin level of 6.8 mg/dL (0.068 mg/L). Given
when hemolysis resolves. (5) his stable and rising hemoglobin level, established diagno-
sis, and family education for avoidance of triggers, the pa-
Treatment/Management tient was discharged with hematology-oncology follow-up.
Treatment of G6PD deficiency relies primarily on avoid-
ance of triggering agents (Table 2) and management of he- Lessons for the Clinician
molytic episodes. Breastfeeding mothers of an infant with • Indirect hyperbilirubinemia and anemia in an infant or
G6PD deficiency should also avoid these triggers. There neonate should prompt evaluation for hemolytic anemia.
are several variants, which differ in severity. The precipitated • Glucose-6-phosphate dehydrogenase (G6PD) deficiency
anemia may be mild to severe depending on the precise mu-
should be suspected in a child with hemolytic anemia
tation involved. The notable forms of G6PD include the
and possible new food exposures. There is a higher preva-
G6PD Mediterranean variant (563C>T), which is associated
lence in males, those with a positive family history, and
with severe hemolysis. This form is contrasted to the G6PD
those from particular geographic areas correlated with pre-
A– variant (biallelic mutation: 202G>A and 376A>G) found
vious endemic malaria (Africa, the Mediterranean region,
in the present patient, which is most often associated
the Middle East, and southern Asia).
with a mild to moderate degree of hemolysis. (6) The G6PD
• Currently, newborn screening for G6PD deficiency occurs
A– variant tends to preferentially affect only mature eryth-
only in Washington, DC, and Pennsylvania. (See the Baby’s
rocytes and results in a mild-moderate anemia. Acute
First Test website [https://www.babysfirsttest.org/] for a full
management for anemia in G6PD deficiency does not differ
state-by-state list of conditions on newborn screens.)
significantly from anemia due to other etiologies. Aggressive
• Screening for G6PD during acute episodes may be falsely
hydration and transfusion when indicated are the mainstays
negative. A high index of suspicion should prompt repeated
of treatment. Despite several studies in the 1980s suggesting
screening after resolution of hemolysis.
improved outcomes with administration of vitamin E or sele-
nium, (9)(10) these treatments have not been incorporated References for this article can be found at
into standard practice. https://doi.org/10.1542/pir.2021-005158.

Vol. 44 No. 12 D E C E M B E R 2 0 2 3 709

Downloaded from /pediatricsinreview/issue/44/12


by Hospital de Peditría 'Prof. Dr. Juan P. Garrahan' user
INDEX OF SUSPICION

Early Satiety and Nausea in a 12-year-old Girl


Alexandra S. Hudson, MD, FRCPC,* Amira Balbaa, MD,† Bryan J. Dicken, MSc, MD, FRCSC, FAAP,‡
Matthew Carroll, BMed (Hons), MHSc (Clin Epi), FRACP*
*Division of Pediatric Gastroenterology and Nutrition, Department of Pediatrics; †Department of Pediatrics; and ‡Division of Pediatric Surgery, Department
of Surgery, University of Alberta, Edmonton, Alberta, Canada

PRESENTATION
A 12-year-old girl with a history of attention-deficit/hyperactivity disorder pre-
sents to the emergency department with persistent nausea, fatigue, and mild
bloating. She has rare nonbloody and nonbilious vomiting, which consists of re-
cently ingested food. She has no dietary restrictions. She is experiencing early
satiety and has had an 18-lb (8.2-kg) weight loss (from 78.9 lb [35.8 kg] [16th per-
centile] to 60.9 lb [27.6 kg] [0.24 percentile]) during the past 3 months. This cor-
responded to a drop in BMI from 14 (0.96 percentile) to 12 (<0.01 percentile).
She has no diarrhea, hematochezia, or melena. Two weeks earlier she was
started on a proton pump inhibitor (PPI). With no symptom change after a
week, she discontinued its use 1 week before presentation. There is no family
history of inflammatory bowel disease or other gastrointestinal disorders. Recent
outpatient blood work shows a normal complete blood cell count with differential
count, electrolytes, thyrotropin, creatinine, urea, albumin, and urinalysis. Magnetic
resonance imaging performed to rule out increased intracranial pressure as an etiology
of her persistent nighttime and early-morning nausea was normal.
She is admitted to the hospital to establish the etiology of her significant
weight loss, in addition to monitoring her for refeeding syndrome when enteral
nutrition is initiated. A full physical examination is normal aside from a thin
body habitus. Baseline blood work, including a celiac screen, complete blood cell
count with differential count, electrolytes, calcium, magnesium, phosphate, lipase,
albumin, liver enzymes, and C-reactive protein, is unremarkable. An upper gastro-
intestinal contrast study finds a distended gastric bubble but no obstructive lesion,
with contrast freely flowing into the duodenum (Fig 1). Pediatric gastroenterology is
consulted. Upper endoscopy reveals a dilated stomach, residual gastric contents,
and a significantly narrowed pylorus of unclear etiology (Fig 2). There is no stom-
ach nodularity. General surgery is consulted intraprocedure and decides to take her
for a pyloroplasty the following day.

DISCUSSION
Differential Diagnosis
AUTHOR DISCLOSURE: Drs Hudson,
Gastric outlet obstruction of any etiology can occur in up to 5 per 1,000 pediatric Balbaa, Dicken, and Carroll have disclosed
patients. (1) Hypertrophic pyloric stenosis, most often affecting infants aged 2 to no financial relationships relevant to this
article. This commentary does not
12 weeks, is the most common of all pediatric etiologies. Other, rarer anatomic
contain a discussion of an unapproved/
causes, such as gastric webs, malrotation, polyps, or pyloric duplication cysts, investigative use of a commercial
should also be considered. External compression from a malignancy is an product/device.

710 Pediatrics in Review

Downloaded from /pediatricsinreview/issue/44/12


by Hospital de Peditría 'Prof. Dr. Juan P. Garrahan' user
Figure 1. Upper gastrointestinal contrast study showing a dilated stomach with contrast movement into the intestine.

important diagnosis to rule out, particularly with rapidly had vomiting and was ultimately found to have a mechanical
progressive symptoms, which may parallel a rapidly grow- outlet obstruction instead.
ing mass (eg, Burkitt lymphoma). Foreign body ingestions
and bezoars can present at any age. Trichotillomania lead- Diagnosis
ing to a hair bezoar should be considered in older children Intraoperatively, the pylorus is noted to be scarred and has
and teenagers. Caustic ingestions can also lead to scarring a thick fibrotic band coursing over it. There is a small lu-
and obstruction. Stricturing Crohn disease may be present minal ulcer, identifying peptic ulcer disease (PUD) as the
at initial inflammatory bowel disease diagnosis. etiology of her partial gastric outlet obstruction. Endoscopic
Gastroparesis is an important consideration in any pediat- culture is negative for Helicobacter (Helicobacter pylori).
ric patient presenting with early satiety, nausea, and reduced Pathology identifies chronic gastric inflammation with acute
oral intake leading to weight loss. (2) Recurrent vomiting is patchy duodenal inflammation. There are no granulomas
the most common symptom. Its diagnosis requires the ex- that suggest Crohn disease.
clusion of a mechanical outlet obstruction. The gold standard
for diagnosis is a gastric emptying study using a solid-phase The Condition and Treatment
meal. Up to three-quarters of pediatric cases are idiopathic, PUD is rarer in pediatric patients than in adults; however,
with the next most common causes being medication- it remains a clinically significant disease with potential for
related, postsurgical, and diabetic. This diagnosis was cer- serious complications. It occurs when inflammation dam-
tainly a consideration in our patient; however, she very rarely ages the gastric or duodenal mucosa down to its third

Vol. 44 No. 12 D E C E M B E R 2 0 2 3 711

Downloaded from /pediatricsinreview/issue/44/12


by Hospital de Peditría 'Prof. Dr. Juan P. Garrahan' user
Treatment includes treating the underlying etiology if appli-
cable (eg, stopping NSAIDs, treating an H pylori infection)
and using a PPI for a minimum of 4 to 8 weeks continu-
ously for acid suppression.

Patient Course
Given the significant peptic stricture that had developed,
our patient needed surgical correction instead of sole medi-
cal management. After the surgery, our patient feels imme-
diate symptomatic relief of her nausea and bloating. She
progresses to a full diet with no issues and is discharged
from the hospital 1 week later. She is kept on a PPI contin-
Figure 2. Endoscopic image showing narrowing, friability, and bleeding uously for an 8-week course and fully recovers. Ultimately,
at the pyloric opening (solid arrow). White patches (dashed arrow) are re-
sidual contrast from the upper gastrointestinal contrast study completed
her diagnosis is classified as idiopathic PUD because no
2 days earlier and are visible throughout the stomach. cause (eg, infection, medication, endocrine disorder, inflam-
matory bowel disease, corrosive ingestion, stress [3]) was
layer (muscularis mucosa). In a review of several interna- identified.
tional studies of children undergoing endoscopy for any
reason, the incidence of PUD has been found to be 2% to Lessons for the Clinician
8%. (3) Risk factors include persistent gastroesophageal re- • In a patient with nausea and weight loss, the absence of
flux disease, nonsteroidal anti-inflammatory drug (NSAID) vomiting does not preclude gastric outlet obstruction.
use, corticosteroid use, H pylori infection, immunosuppres- • Peptic ulcer disease may not present with classic upper
sion, and severe illness. The most common presenting abdominal pain and may instead present with symptoms
symptom in the pediatric patient is abdominal pain, partic- secondary to ulcer complications.
ularly postprandial epigastric pain. Our patient did not pre- • Upper gastrointestinal contrast studies may not always
sent with this classic symptom and instead presented with identify a partial gastric outlet obstruction; further inves-
symptoms secondary to PUD complications. In her case, tigations such as upper endoscopy should be sought if
the inflammation had led to obstruction, which explained the clinical suspicion remains high.
her main presenting symptoms of nausea and early satiety.
Gastric outlet obstruction results in incomplete gastric emp- Consent
tying. Residual food and liquid leads to gastric distention, Informed consent was obtained from the patient and the
which can trigger nausea. Other potential complications of patient’s family for publication of this case.
PUD that can be the primary presenting symptom or sign
include gastrointestinal bleeding, hemodynamic instability References for this article can be found at
secondary to gastric or intestinal perforation, and anemia. https://doi.org/10.1542/pir.2021-005495.

712 Pediatrics in Review

Downloaded from /pediatricsinreview/issue/44/12


by Hospital de Peditría 'Prof. Dr. Juan P. Garrahan' user
INDEX OF SUSPICION

Intermittent Fussiness in a Well-Appearing


5-week-old Girl
Elizabeth Wikle, MD,*‡ Amani Sanchez, DO,*†‡ Arianna Nassiri, MD,*†‡ Francis Onyebuchi, MD*‡
*Department of Pediatrics and †Department of Pediatric Emergency Medicine, Dell Children’s Medical Center, Austin, TX

University of Texas at Austin-Dell Medical School, Austin, TX

PRESENTATION
A 5-week-old term infant with a normal perinatal course and born via an uncompli-
cated vaginal delivery to a gravida 2 para 1 mother with good prenatal care presents
to the emergency department with 3 days of fussiness. Mother reports that the in-
fant has had intermittent episodes of crying as if she is in pain and that nothing
consoles her. The crying episodes last up to an hour and are associated with breath-
holding and facial redness. She has not had fever, emesis, or hematochezia. The in-
fant is tolerating direct breastfeeding well, with appropriate urine output and nor-
mal stool frequency and consistency. Mother was supplementing with formula but
stopped a day before admission.
In the emergency department, her temperature is 97.6 F (36.4 C), with a
heart rate of 134 beats/min, a respiratory rate of 40 breaths/min, and oxygen sat-
uration of 98% on room air. She weighs 3.63 kg and demonstrates adequate
weight gain from birth. Examination is significant for intermittent crying and
fussiness; otherwise, the infant is in no acute distress. She has a flat and open
anterior fontanelle. Her eyes, ears, and nose are normal. Oral examination dem-
onstrates normal suck without ulcer or abrasion but with a small erythematous
macule on her hard palate. Her cardiopulmonary examination findings are nor-
mal. Her abdomen is soft, nontender, and without organomegaly or masses.
She has no gross deformities or bony tenderness of limbs, rash, visible bruising,
or joint swelling. She has full passive range of motion of upper and lower
extremities. Her capillary refill is less than 2 seconds, and she appears well
hydrated. No hair tourniquet is present. Her neurologic examination findings are
normal.
Initial laboratory findings demonstrate a normal complete blood cell
Dr Wikle’s current affiliation is the
count and a normal complete metabolic panel aside from an elevated total bili- general pediatrics department, University
rubin level of 2.6 mg/dL (44.5 mmol/L) (reference range, 0.05–0.68 mg/dL of Iowa, Iowa City, IA. Dr Nassiri’s current
affiliation is Department of Pediatric
[0.86–11.63 mmol/L]). Total bilirubin level on day of life 3 was 9.8 mg/dL
Emergency Medicine, Children’s Hospital
(167.6 mmol/L). Urinalysis results are within normal limits, including the ab- of New Orleans, New Orleans, LA.
sence of red and white blood cells. A flat and upright abdominal radiograph
AUTHOR DISCLOSURE: Drs Wikle,
demonstrates a normal gas pattern without free air or bowel dilation. No ileocolic Sanchez, Nassiri, and Onyebuchi have
intussusception is identified on focused ultrasonography, but it does demonstrate disclosed no financial relationships
a small volume of complex free fluid in the lower pelvis. The patient is admitted relevant to this article. This commentary
does not contain a discussion of an
to the hospital for observation, serial abdominal examinations, and continued unapproved/investigative use of a
evaluation. commercial product/device.

Vol. 44 No. 12 D E C E M B E R 2 0 2 3 713

Downloaded from /pediatricsinreview/issue/44/12


by Hospital de Peditría 'Prof. Dr. Juan P. Garrahan' user
DISCUSSION
Differential Diagnosis
The differential diagnosis for a fussy infant is broad and
encompasses many etiologies, including sepsis, meningitis,
gastroesophageal reflux disease, anatomic gastrointestinal
etiologies such as intussusception or malrotation, heart fail-
ure or arrhythmia, hair tourniquet, corneal abrasion, nonac-
cidental trauma, and colic. Although sepsis and colic seem
to be more prevalent conditions in this age group, the rest
are less commonly seen. Absence of history of arching move-
ments and emesis makes gastroesophageal reflux disease less Figure. Long axis cross section of 6.6 × 4.1 × 4.6-cm complex cystic lesion
in the right lower quadrant extending into the subhepatic region. The
likely. Intussusception is most common between 3 and hypoechoic cystic fluid is contrasted with hyperechoic debris settling at the
9 months of age but can occur in younger infants. Our base. Markers indicate measurement of the width of the cystic structure.

patient’s examination findings are grossly normal, making


hair tourniquet less likely. Although corneal abrasion is on torsion, even fewer include patients younger than 1 year.
the differential diagnosis, a fluorescein test is not com- One such study demonstrated that the incidence of ovarian
pleted. The erythematous papule on the hard palate raises torsion in pediatric patients is bimodal, with increases in in-
concern for nonaccidental trauma, herpes simplex virus, or fancy (16%) and early adolescence (9–14 years of age; 52%).
trauma (burn from a hot bottle). This finding is subtle and (2)(3)
quickly resolves. In addition, the laboratory assessment did Risk of torsion increases with ovarian cysts, tumors,
not demonstrate electrolyte abnormalities or other abnor- and pregnancy. As the size of the ovary increases due to a
malities such as leukocytosis or leukopenia that could be mass, the risk of torsion increases; the risk is highest
concerning for infection. Although colic is a diagnosis of when the mass becomes 5 cm or greater in a pediatric pa-
exclusion, our initial investigation demonstrates complex tient. (1) The right ovary is more likely to torse compared
free fluid in the lower pelvis, which prompts further evalua- with the left, possibly due to the longer utero-ovarian liga-
tion. Further imaging reveals the diagnosis. ment and/or the presence of the colon on the left side of the
abdomen, which may help prevent ovarian torsion. (4) In
Actual Diagnosis utero exogenous estrogen exposure is thought to be the etiol-
Complete abdominal ultrasonography reveals a 6.5-cm cys- ogy of cystic lesions noted prenatally or in infancy, which
tic lesion with debris in the right lower quadrant with a predisposes fetuses and neonates to ovarian torsion. Some
nonvisualized right ovary (Fig). Pediatric general surgery incidences of infantile ovarian torsion do not have evidence
is consulted to review the ultrasonographic image and pro- of enlarged ovaries or cysts. The proposed mechanism of tor-
vide recommendations regarding management. The infant sion in these cases is congenitally elongated utero-ovarian
undergoes a diagnostic laparoscopy that day. Intraopera- ligaments compared with the size of a premenarchal uterus,
tively, a necrotic chocolate cyst is seen in the right upper prompting a higher risk for adnexal movement.
quadrant that is avulsed from the adnexa and twisted in Abdominal pain is the only consistent symptom of
the omentum. The uterus and left ovary appear healthy. ovarian torsion in all pediatric age ranges and can often be
She is diagnosed as having a hemorrhagic ovarian cyst similar to acute appendicitis in the older child. (1) Other
and right ovarian torsion with autoamputation of the right findings in infants often also include the palpation of a
adnexa. mass, feeding intolerance, and intermittent fussiness. The
most common factor in infantile ovarian torsion is prenatal
The Condition imaging demonstrating an enlarged ovary, predisposing the
Ovarian torsion occurs when the adnexa rotates about its infant to torsion. (2) Most symptoms are nonspecific and
ligamentous support, which results in occlusion of blood lead to further evaluation for more common conditions
flow to the ovary and can lead to edema, enlargement, and based on the patient’s age. In younger children, the differ-
eventual tissue necrosis. This also can lead to adnexal ential diagnosis usually includes urinary tract infections,
avulsion, as seen in our patient. Ovarian torsion is rare in intussusception, and nonaccidental trauma. In all patients,
the pediatric population, with an incidence of 0.5 to 2 per nephrolithiasis, appendicitis, and mesenteric adenitis should
10,000 patients. (1) Of the few studies of pediatric ovarian be considered. (3)

714 Pediatrics in Review

Downloaded from /pediatricsinreview/issue/44/12


by Hospital de Peditría 'Prof. Dr. Juan P. Garrahan' user
Imaging is the cornerstone of evaluation for ovarian important, but detorsion is the ultimate modality for pain
torsion. The most common ultrasonography finding is an resolution. This further highlights the importance of keeping
enlarged heterogeneous mass with an inability to identify ovarian torsion on the differential diagnosis for premenar-
the ipsilateral ovary, as seen in our patient. Doppler ultra- chal children and infants with fussiness or abdominal pain
sonography has a high false-negative rate and should not and potential use of diagnostic laparotomy in highly suspi-
exclude the diagnosis of ovarian torsion if the clinical sus- cious clinical scenarios.
picion is high. One study demonstrated flow in 62% of
torsed ovaries. (3) Doppler flow may be appreciated in a Patient Course
torsed ovary without complete cessation of blood flow be- An oophorectomy is performed and the infant does well
cause the ovary has a dual blood supply from collateral postoperatively, with complete resolution of intermittent
vessels (eg, utero-ovarian vessels, infundibulopelvic vessels). crying episodes. Postoperatively, the infant’s pain is man-
More recently, assessments that evaluate laboratory value aged with scheduled acetaminophen. She tolerates human
ratios, such as lymphocyte-to–C-reactive protein ratio, have milk and has a bowel movement before discharge. The in-
been explored to assist in diagnosis, but they have not been fant is discharged on postoperative day 1 with instructions
found useful in patients younger than 1 year. (5) The gold to follow up with pediatric general surgery 2 weeks after
standard for the diagnosis of ovarian torsion is intraopera- discharge. Two days after discharge, the infant presents
tive visualization of a torsed ovary. back to the emergency department with inconsolable fussi-
Note that this patient was evaluated in the emergency ness and decreased oral intake. A serious bacterial infec-
department of a major regional children’s hospital. This tion, potentially related to her recent surgery, is suspected,
setting provides quick and easy access to diagnostic testing, and a full evaluation for sepsis is performed, which is unre-
which may prompt further evaluation of the crying infant, markable. Of note, her laboratory assessment demonstrates
specifically laboratory assessment and imaging as described an improved total bilirubin level, which is now 0.8 mg/dL
previously herein. In this patient’s case, examination of the (13.7 mmol/L). A normal left ovary and corresponding vessels
infant did demonstrate clear periods of calmness within with no evidence of intussusception are seen on ultraso-
episodes of inconsolable crying, which prompted further nography. The infant is admitted for observation. In the
evaluation rather than a more conservative approach. morning, the fussiness resolves, she is drinking well, and
she is discharged home.
Management
Preferred management is ovarian detorsion rather than
Lessons for the Clinician
oophorectomy due to evidence of ovarian viability after de-
• If an infant presents with increased fussiness and has
torsion. (6) Detorsion is often performed to assess viability
normal examination findings, intra-abdominal and pelvic
because ovarian necrosis is rare and is evident when the
pathology should still be ruled out.
ovary “falls apart” when manipulated. (7) In 1 large study,
• Consider early surgical consultation if acute intra-
younger patient age was associated with oophorectomy.
abdominal pathology is suspected.
(8) One would suspect that infrequency of premenarchal
• Ultrasonography is a cost-efficient, safe, and quick way
ovarian torsion would lead to delay in diagnosis, increased
to assess intra-abdominal and pelvic pathology, but one
tissue death, and increased need for oophorectomies in
younger patients. Unlike testicular torsion, which has a com- should not rule out ovarian torsion if Doppler findings
monly understood window of tissue viability of 6 to 8 hours are normal and there is high clinical suspicion for ovarian
after symptom initiation, ovarian necrosis does not have an torsion.
obvious correlation to duration of symptoms. (9) Pain may
originate from venous congestion before arterial compromise References for this article can be found at
and can be very severe. (10) Adequate pain management is https://doi.org/10.1542/pir.2022-005671.

Vol. 44 No. 12 D E C E M B E R 2 0 2 3 715

Downloaded from /pediatricsinreview/issue/44/12


by Hospital de Peditría 'Prof. Dr. Juan P. Garrahan' user
INDEX OF SUSPICION

Fatigue, Weight Loss, and Acute Chest Pain in a


15-year-old Boy
Madeline F.E. Parr, MD,* Katharine N. Clouser, MD, FAAP,* Meghan Tozzi, MD, FAAP,* Sejal M. Bhavsar, MD, FAAP*
*Joseph M. Sanzari Children’s Hospital, Hackensack, NJ

PRESENTATION
A 15-year-old boy with a history of membranous ventricular septal defect (VSD),
not surgically repaired, presents to the emergency department (ED) with a 1-month
history of fatigue, 5-lb (2.3-kg) weight loss, and productive cough, along with 3 days
of right-sided chest pain. The patient went to his pediatrician approximately a
month earlier due to cough, fatigue, and decreased activity and was presumed to
have Mycoplasma pneumoniae and was treated empirically with azithromycin. His
cough improved mildly, but he continued to have low energy. Two weeks before
presentation, he developed night sweats without documented fever. Three days be-
fore presentation, the patient went to a gun range and was struck on his right
chest by his rifle recoiling while he was shooting. After this event, he had constant
chest pain and was taken to an orthopedic surgeon, where a chest radiograph was
performed that, per verbal report, did not reveal evidence of fracture, and the
patient’s mother was not informed of any cardiopulmonary findings. His chest
pain worsened with deep inspiration and he began having shortness of breath,
which prompted his mother to bring him to the ED.
The patient denies any history of substance abuse or sexual activity, and he
lives with his mother, father, and siblings. The family has had their home cleaned
for the past 8 years by a housekeeper who had emigrated from El Salvador and
developed an undiagnosed chronic cough 4 months ago. Travel history includes a
trip to Florida 6 months ago, and he has traveled to Israel multiple times, with
his most recent trip 3 years before presentation. He had COVID-19 pneumonia
3 months before presentation and did not require hospitalization. He received
2 doses of the Pfizer-BioNTech COVID-19 vaccine 3 weeks apart, with his second
dose 2 months before presentation.
In the ED he appears fatigued but is afebrile and normotensive with normal
heart and respiratory rates and is saturating at 98% on room air. He has mild
right-sided chest wall tenderness overlying the T-4 level in the axillary line. He
has decreased air entry to his bilateral lung bases, but no adventitious sounds
are auscultated. He has a IV/VI harsh holosystolic murmur at Erb’s point and is
warm and well perfused, with peripheral pulses 21 in all extremities. On oral
AUTHOR DISCLOSURE: Drs Parr, Clouser,
examination he has braces on his upper and lower teeth and no evidence of car- Tozzi, and Bhavsar have disclosed no
ies or gingival inflammation. No lymphadenopathy, rash, abdominal tenderness, financial relationships relevant to this
or organomegaly are appreciated. No evidence of cutaneous septic or vascular article. This commentary does not
contain a discussion of an unapproved/
phenomena, such as Osler nodes, Janeway lesions, or nail bed hemorrhages, are investigative use of a commercial
appreciated. product/device.

716 Pediatrics in Review

Downloaded from /pediatricsinreview/issue/44/12


by Hospital de Peditría 'Prof. Dr. Juan P. Garrahan' user
Laboratory results are significant for a white blood cell count
of 9,900/mL (9.9 × 109/L [reference range, 4,500–13,000/mL
(4.5–13.0 × 109/L)]) with a differential count of 73% neutro-
phils (reference range, 40%–62%), 15% lymphocytes (refer-
ence range, 27%–40%), 10% monocytes (reference range,
2%–8%); C-reactive protein level, 12.3 mg/dL (123 mg/L
[reference range, <0.5 mg/dL (<5 mg/L)]); erythrocyte sedi-
mentation rate, 99 mm/h (reference range, 0–15 mm/h);
and hemoglobin level, 11.3 g/dL (113 g/L [reference range,
13.0–16.0 g/dL (130–160 g/L)]), with a mean corpuscular
volume of 79.1 fL (reference range, 78–98 fL). A complete
metabolic panel demonstrates normal serum levels of elec-
trolytes and transaminases. His COVID-19 polymerase chain Figure 1. Apical computed tomographic view of the chest with consolida-
tions in the left lower lung lobe and in the right middle lung lobe demon-
reaction, respiratory pathogen panel, rheumatoid factor, and strating central necrosis (red arrows).
antineutrophil cytoplasmic antibody panel are negative. His
electrocardiogram demonstrates sinus arrhythmia with nor-
(TTE) revealed a small, semimembranous VSD with minimal
mal axis and intervals and normal voltages and repolariza-
mitral regurgitation and no evidence of pedunculated vegeta-
tion. Two large-volume blood cultures at different peripheral
tions. A transesophageal echocardiogram (TEE) was recom-
sites are collected.
mended to evaluate for small sessile lesions, but the family
declined due to the more invasive nature of the study. After
DIFFERENTIAL DIAGNOSIS
72 hours of incubation, both initial blood cultures grew Strep-
The diagnostic possibilities included diverse infectious tococcus mutans, which is sensitive to ceftriaxone, penicillin G,
etiologies due to the patient’s history and risk factors, and vancomycin. All blood cultures obtained after starting cef-
including international and domestic travel, subacute triaxone treatment were negative. The patient was diagnosed
fatigue, weight loss, night sweats, and acute chest pain. as having S mutans endocarditis due to his positive blood cul-
Differential diagnoses at the time of admission included tures, historical cardiac defect, and evidence of septic emboli
pulmonary contusion, pneumothorax, pulmonary tuber- on chest imaging. He was treated with 6 weeks of ceftriaxone
culosis, multifocal pneumonia, sarcoidosis, endocardi- therapy. At his 4-week outpatient follow-up appointment, he
tis, distal infection with pulmonary septic emboli, and continued to be afebrile and reported no complaints of chest
sequelae from COVID-19 infection. A chest radiograph pain, cough, or fatigue. Repeated chest radiography showed
demonstrated right middle lobe parenchymal infiltrate interval improvement of bilateral opacities.
associated with right pleural effusion and possible left
lower lobe pneumonia versus parenchymal contusion. A
computed tomographic scan of the chest with contrast
revealed multifocal consolidations most significant in
the right middle lobe and left lower lobe. The consolida-
tion in the left lower lobe demonstrated central necrosis,
raising the suspicion for necrotizing pneumonia, pulmo-
nary tuberculosis, lung abscesses, and septic emboli (Figs 1
and 2). Thoracic adenopathy was appreciated, and this con-
stellation of findings suggested an infectious etiology rather
than trauma.
The patient was started on ceftriaxone and placed on air-
borne isolation precautions due to concern for pulmonary
tuberculosis. Early-morning sputum samples were collected
for acid-fast bacilli smear and culture, and 3 consecutive
morning samples had negative acid-fast bacilli smears.
Interferon-g release assay was negative, and the purified pro- Figure 2. Apical computed tomographic view of the chest with a consoli-
tein derivative was 0 mm. A transthoracic echocardiogram dation in the right middle lung lobe (red arrow). Motion artifact is present.

Vol. 44 No. 12 D E C E M B E R 2 0 2 3 717

Downloaded from /pediatricsinreview/issue/44/12


by Hospital de Peditría 'Prof. Dr. Juan P. Garrahan' user
DISCUSSION valves, or abnormalities of the thoracic cage or chest wall
Infective endocarditis (IE) is a rare diagnosis among pediatric that make TTE less accurate. (2) Blood cultures are also
patients in the United States, with an estimated incidence of highly important in the evaluation of IE, and 3 large-volume
0.43 cases per 100,000 children compared with 15 cases per peripheral blood cultures drawn at different sites and under
100,000 adults. (1) Underlying cardiac malformations con- aseptic technique are preferred. Smaller volumes can
tribute significantly to the risk of IE, as approximately 53% be appropriate for smaller patients, such as 1 to 3 mL in
of diagnosed cases are in children with congenital heart de- infants or 5 to 7 mL in young children. (2)
fects. (1) Disruptions in cardiac architecture from malfor- IE can arise from a variety of infectious organisms.
mations such as atrial septal defects or VSDs create areas Staphylococcal IE continues to be the most frequently di-
of increased flow in the heart chambers, which can damage agnosed subtype, but the incidence of streptococcal IE is
the endocardium and increase the risk of platelet thrombus increasing. (1) Among streptococcal infections, the viri-
formation. (2) Platelet thrombi can subsequently be seeded dans group of streptococci is most frequently identified.
in a state of transient bacteremia, leading to infectious S mutans is a rare cause of endocarditis, primarily giving a
thrombus formation. Due to bacteria hidden deep within a subacute clinical picture, and can have significant morbidity
platelet thrombus, the treatment of IE typically requires 4 to and mortality. (5) S mutans is a bacterial species that is most
8 weeks of antibiotics to eradicate an infection. (2) frequently isolated in the oral cavity and can form biofilms
Diagnosis of IE is based on the modified Duke criteria, that increase virulence. (6) Dental caries and poor oral hy-
which has been adapted from the original Duke criteria. giene contribute to the formation of Streptococcus-containing
IE is categorized as definite, possible, or rejected based on biofilms, which can predispose the host to transient bacter-
pathologic, major, and minor clinical criteria. Definite di- emia, as can routine dental brushing and flossing. (7) It is
agnosis requires all subcriteria to be met within these cate- proposed that transient bacteremia, in conjunction with en-
gories, and possible diagnosis requires 2 major criteria, docardial malformations seen in congenital heart defects,
1 major and 2 minor criteria, or 3 minor criteria. (3) The provides an environment for bacterial colonization, which
pathologic criteria are histologic confirmation of a cardiac can precipitate the development of subacute IE. (2) In a
or embolized vegetation or a cardiac lesion such as an ab- 4-year-old patient with S mutans IE, poor oral hygiene
scess. Major criteria include evidence of endocardial in- was determined to be the source of her bacteremia. (8)
volvement on imaging or a positive blood culture of an Although our patient had appropriate oral hygiene, he
organism with a propensity for IE. Minor criteria are fever, did have frequent orthodontic appointments for the manage-
predisposing cardiac lesion or drug use, immunologic ment of braces, and it is possible that gingival manipulation
phenomena, vascular phenomena such as septic emboli, during procedures or with daily oral care may have been the
or microbiological evidence of infection. Our patient’s di- source of his bacteremia. In addition, before presentation,
agnosis of endocarditis was categorized as “possible” and our patient was treated with a 5-day course of azithromycin,
relied on 3 minor criteria: his history of VSD, positive which likely partially treated the infection, accounting for his
blood cultures, and the presence of multifocal pulmonary initial improvement of overall symptoms. Current guidelines
lesions that were attributed to septic emboli. Pulmonary do not recommend antibacterial prophylaxis against endocar-
septic emboli typically result from the showering of small ditis for children with a VSD undergoing routine orthodontic
bacterial thrombi from a central cardiac infectious thrombus. care, as in our patient. (2) However, prophylaxis should be
A TEE was recommended to confirm the diagnosis and to considered in high-risk children, such as those with cardiac
evaluate for sessile vegetations, but the family declined the transplants, previous cases of IE, cyanotic heart disease, or
more invasive test. prosthetic valves.
TEE demonstrates a significantly superior ability to
diagnose endocarditis, with sensitivity of 94% compared Lessons for the Clinician
with 24% with TTE among adults. (4) Many families • Infective endocarditis (IE) is a rare but vital differential diag-
can be hesitant to pursue TEE, but clinicians should nosis in pediatric patients presenting with subacute illness.
consider strong recommendations of TEE in the setting Risk factors such as congenital heart malformation or poor
of IE that may warrant additional intervention. Indications oral hygiene should raise a clinician’s suspicion for IE.
for considering surgical intervention include electrocardio- • The modified Duke criteria are favored in diagnosing IE
graphic conduction changes that would suggest abscess, in children. Transesophageal echocardiography should
significant change in valvular flow, children with prosthetic be performed when transthoracic echocardiography is

718 Pediatrics in Review

Downloaded from /pediatricsinreview/issue/44/12


by Hospital de Peditría 'Prof. Dr. Juan P. Garrahan' user
inadequate and when additional findings, such as an ab- sites and under aseptic technique should be collected at
scess, would change patient management. initial presentation. Blood cultures should be repeated
• Staphylococcal species are most frequently implicated in within 24 to 48 hours of starting therapy.
IE, although streptococcal infections are on the rise.
• Peripheral blood cultures should be collected whenever References for this article can be found at
IE is suspected, and 3 large-volume cultures at different https://doi.org/10.1542/pir.2022-005552.

ANSWER KEY FOR DECEMBER PEDIATRICS IN REVIEW


Athlete Screening and Sudden Cardiac Death: 1. A; 2. B; 3. D; 4. C; 5. D.
Ensuring Emergency Readiness in the Pediatric Primary Care Setting: An Updated Guideline: 1. D; 2. C; 3. C; 4. E; 5. E.
Fever: Three Patient Cases: 1. C; 2. B; 3. D; 4. E; 5. D.

Vol. 44 No. 12 D E C E M B E R 2 0 2 3 719

Downloaded from /pediatricsinreview/issue/44/12


by Hospital de Peditría 'Prof. Dr. Juan P. Garrahan' user
IN BRIEF

Pneumocystis Pneumonia
Asif Noor, MD,* Leonard R. Krilov, MD*
*Division of Pediatric Infectious Diseases, Department of Pediatrics, NYU Long Island School of Medicine, Mineola, NY

Pneumocystis is an opportunistic pathogen that causes lower respiratory tract infection.


Over time, the terminology has changed from Pneumocystis carinii to Pneumocystis
jirovecii, and its placement in taxonomy has shifted from protozoa to fungi. During
the past 50 years, it has become an important cause of pneumonia in people with a
weakened immune system from the effects of human immunodeficiency virus
(HIV) or other immunosuppressive states. Pneumocystis pneumonia (PCP) remains
an important clinical entity as the number of children with altered immune sys-
tems, receiving chronic immunosuppressive therapy, or receiving bone marrow and
organ transplants is rapidly increasing.
Modern molecular mapping has established that the DNA sequence of
Pneumocystis differs markedly based on the host species. The former name, P carinii,
is now reserved for the Pneumocystis species that causes lung infection in rats.
Pneumocystis affecting human lungs is P jirovecii (pronounced “yee row vet zee”)
based on the nucleotide sequence of the organism. This new name is in honor of
the Czech parasitologist Otto Jerovic who first described this pathogen in humans.
P carinii can still be acceptable terminology, but only with the annotation forma
specialis or f. sp. to identify the host of origin: the organism infecting humans is
named as P carinii f. sp. homini. The term PCP is still used to refer to the clinical
syndrome Pneumocystis pneumonia.
Pneumocystis was reclassified as a fungus in 1988 based on molecular studies
of the organism demonstrating the presence of chitin and 1,3-ß-D-glucan, although AUTHOR DISCLOSURE: Drs Noor and
Krilov have not disclosed any financial
the organism also has protozoan features, including the presence of different relationships relevant to this article. This
stages: cysts, sporozoites, and trophozoite forms. commentary does not contain a discussion
Transmission is through the airborne route from human to human. Although of an unapproved/investigative use of a
commercial product/device.
the pathogen is found in the lungs of several animals other than humans (eg,
rats and mice), interspecies transmission has not been demonstrated. People A New Name (Pneumocystis jiroveci) for
with normal immune systems carry asymptomatic infection (colonization) and Pneumocystis from Humans. Stringer JR,
serve as a pool for transmission. Most healthy children acquire infection by Beard CB, Miller RF, Wakefield AE, et al.
Emerg Infect Dis. 2002;8(9):891–896
2 years of age: in a prospective cohort study in Chile, 85% of healthy infants had
seroconverted by 20 months of age. Although a few cases have been reported in Primary Infection by Pneumocystis carinii
immunocompetent infants, symptomatic disease occurs almost exclusively in in a Cohort of Normal, Healthy Infants.
immunocompromised children. Vargas SL, Hughes WT, Santolaya ME, et al.
Clin Infect Dis. 2001;32(6):855–861
The severity of infection depends on the degree of immunosuppression, with
the single most important factor being the compromise of cell-mediated immu- Pneumocystis jirovecii Infections. In: Red
nity. The risk of PCP in children with HIV depends on their CD4 count, which Book: 2018–2021 Report of the Committee
normally is highest at birth and falls through childhood. Infants younger than on Infectious Diseases. 31st ed. Kimberlin
DW, Brady MT, Jackson MA, Long SS, eds.
12 months are at risk with a CD4 count less than 1,500/mL; children 1 to 5 years Itasca, IL: American Academy of Pediatrics;
of age at less than 500/mL; and, similar to adults, less than 200/mL in children 2021:651–656

720 Pediatrics in Review

Downloaded from /pediatricsinreview/issue/44/12


by Hospital de Peditría 'Prof. Dr. Juan P. Garrahan' user
6 years and older. In children with non-HIV conditions,
the risk of PCP is proportional to the extent of immuno-
suppression. For example, in children receiving 3 or more
chemotherapeutic agents, the risk is 22% vs 2% to 5% in
children who are taking fewer than 3 chemotherapeutic
agents. Similarly, the addition of radiation to chemothera-
peutic agents increases the risk by 35%. The use of anti–
tumor necrosis factor–modulating agents in children with in-
flammatory bowel disease also increases the risk of PCP.
Lungs are primarily affected, and hypoxia is the hallmark
clinical finding. The clinical course varies greatly depending
on the underlying T-cell defect and individual host factors.
Disease onset may be either fulminant or indolent, and dura-
tion is variable. An abrupt onset is typically seen in children
with non-HIV causes of T-cell suppression. Often, the onset
of PCP in patients without HIV precedes discontinuation or
tapering of corticosteroids. The course in patients without Figure 1. Bilateral perihilar infiltrates due to Pneumocystis jirovecii. Courtesy
of AAP Red Book.
HIV is rapidly progressive over a few days. To the contrary,
an indolent onset is observed in children with HIV infection, Specimen retrieval to identify P jirovecii can be done
and the clinical course may span 3 weeks or more. Due to its through several methods. The least invasive method is induced
slow progression, PCP in children with HIV often presents sputum. Limitations include low sensitivity of 20% to 40%
only later in its course, with worsening fevers and hypoxia. and the need for cooperation from the child in obtaining the
Signs and symptoms in children and adolescents can specimen. A tracheal aspirate can be obtained from an intu-
include fever, cough, tachypnea, and dyspnea, but fever is bated patient and has sensitivity of 50% to 60%. A specimen
not universal with PCP. On auscultation, there is a startling obtained by bronchoalveolar lavage is the preferred method for
absence of adventitious lung sounds disproportionate to the detecting P jirovecii. When performed through a flexible
degree of tachypnea (60–80 breaths/min). The alveolar- fiberoptic bronchoscope, the sensitivity is 75% to 95%. A
arterial gradient, which measures the difference in oxygen- transbronchial lung biopsy (small sample, close to 20 alveoli)
ation between alveolar and arteriolar blood, is often greater in conjunction with bronchoalveolar lavage increases the yield.
than 30 mm Hg (reference range, 5–10 mm Hg) in HIV- An open lung biopsy provides a larger specimen and allows
associated PCP. for histopathologic analysis, and sensitivity is 90% to 100%;
The classic chest radiograph with PCP demonstrates however, the disadvantage is the requirement for general anes-
symmetrical bilateral interstitial infiltrates. Early in the dis- thesia and complications such as pneumothorax and bleeding.
ease course, the infiltrates are perihilar, which progress to Several diagnostic tests are used to identify P jirovecii.
involve the peripheral lung fields (Fig 1). The apical lung PCR assay on respiratory fluid, especially from bronchoal-
areas are spared. Uncommon chest radiography findings veolar lavage, is widely used but has the drawback that
may include asymmetrical distribution, unilateral lobar con- PCR does not distinguish colonization from infection. In
solidation, pneumothorax, pneumomediastinum, and pleural addition, there is currently no Food and Drug Administra-
effusion. In cases of PCP with a normal chest radiograph, a tion (FDA)–approved PCR test for this organism. Immuno-
high-resolution computed tomographic scan may reveal fluorescence stain using fluorescent monoclonal antibodies
bilateral ground glass opacification or cystic lesions. and stains such as Gomori methenamine silver and Giemsa
Clinical features of PCP overlap with other causes of are commonly used (Fig 2).
pneumonia. Definitive testing through polymerase chain Other nonspecific laboratory tests, such as lactate dehy-
reaction (PCR) assay or stains should be considered in a drogenase levels, eosinophil count, and albumin levels,
suspected case. Confirmation of PCP requires demonstration may support the diagnosis of PCP. However, abnormalities
of P jirovecii in the pulmonary parenchyma or bronchoalveo- seen on these tests can also be reflect the child’s underlying
lar fluid. The likelihood of detection is higher in children disease, as in a child with malignancy whose baseline lactate
with HIV versus those with non-HIV conditions, reflecting dehydrogenase levels might be elevated from increased cell
the high burden of infection in children with AIDS. turnover.

Vol. 44 No. 12 D E C E M B E R 2 0 2 3 721

Downloaded from /pediatricsinreview/issue/44/12


by Hospital de Peditría 'Prof. Dr. Juan P. Garrahan' user
Figure 2. A. Giemsa stain of trophozoites of Pneumocystis jiroveci. B. Cysts of P jirovecii stained with methenamine silver and hematoxylin and eosin. C. Direct
immunofluorescence antibody stain. Courtesy of the Centers for Disease Control and Prevention.

Empirical treatment should be initiated when there is a with a history of a severe reaction to a sulfonamide: anaphy-
high clinical suspicion for PCP in children with known laxis, Steven-Johnsons syndrome, toxic epidermal necrolysis.
risk factors who are acutely ill. Management has 3 funda- Atovaquone, although expensive, is safe and as effective as
mentals: supportive care, antibiotic treatment, and anti- dapsone. Aerosolized pentamidine is reserved for children
inflammatory therapy. The basis of supportive care is to who cannot tolerate the other 3 agents but is less effective
maintain the partial pressure of oxygen at 70% or greater and requires patient cooperation. Its dose is 300 mg once
with supplemental oxygen and, if necessary, ventilatory monthly in children 5 years and older and adolescents. The
support. Trimethoprim-sulfamethoxazole (TMP-SMZ) is rate of breakthrough PCP infection is up to 6% in children
the antibiotic of choice, and it can be administered either receiving TMP-SMZ as opposed to 12% in children receiving
intravenously or orally. Adverse effects are more common alternative prophylaxis.
with HIV than with non-HIV underlying conditions. The Comments: This In Brief strikes a personal note for
most common adverse effect is a maculopapular rash, me. Just out of fellowship, I began my career in the Bronx
which is usually transient. In a child unable to tolerate in 1983, just as the AIDS epidemic was exploding. New
TMP-SMZ, intravenous pentamidine isethionate is used. York State’s newborn screening program in the 1980s
The treatment duration is 21 days. In a child with partial showed that 2% to 5% of women delivering babies in the
pressure of oxygen less than 70% and/or alveolar-arterial Bronx were infected with HIV, and given a transmission
oxygen gradient greater than 35%, an anti-inflammatory rate of 25% to 35%, 1 in 200 to 1 in 50 babies born in the
agent, such as methylprednisolone, should be considered. Bronx were likewise infected—an astonishing number not
PCP is highly preventable with appropriate antibiotic very different from that of sub-Saharan Africa. With no
prophylaxis. Children at risk for PCP with malignancy HIV-specific treatment available beyond supportive care,
undergoing intensive chemotherapy, with a cell-mediated the prognosis for those babies was dire, and a frequent
immune deficiency such as severe combined immunode- cause of their deaths was PCP. Long before zidovudine
ficiency, transplant recipients, and patients with HIV infec- was approved as the first antiretroviral agent for use
tion and an abnormally low CD4 count, should be placed on against HIV for adults in 1997 and for children in
PCP prophylaxis. The agent of choice is TMP-SMZ, taken 2000, the single most effective intervention we had was
3 times a week. Dapsone, which is both effective and inex- reliable prophylaxis against PCP with TMP-SMZ—truly
pensive, is an option in children who cannot tolerate TMP- a lifesaver.
SMZ. The dose is 2 mg/kg per day, or 4 mg/kg per week. Henry M. Adam, MD
Both TMP-SMZ and dapsone should be avoided in children Associate Editor, In Brief

722 Pediatrics in Review

Downloaded from /pediatricsinreview/issue/44/12


by Hospital de Peditría 'Prof. Dr. Juan P. Garrahan' user
IN BRIEF

Implications of Food and Drug Administration


Approval of Respiratory Syncytial Virus Prophylactic
Medication
Ashley L. Saint-Fleur, MD,* Catherine Kier, MD†
*Division of Pulmonary Medicine, Boston Children’s Hospital, Boston, MA
†Division of Pulmonary Medicine, Department of Pediatrics, Stony Brook Children’s Hospital, Stony Brook, NY

Therapies for prevention of respiratory syncytial virus (RSV), a major cause of


lower respiratory tract infections (LRTIs) among infants and toddlers, are ex-
panding. The Food and Drug Administration (FDA) recently approved adminis-
tration of nirsevimab, a monoclonal antibody aimed at preventing medically
attended RSV LRTIs, for all infants and especially those at high risk.
RSV is the most common cause of LRTI and hospitalization for respiratory AUTHOR DISCLOSURE: Drs Saint-Fleur
and Kier have disclosed no financial
illness among infants and young children. RSV is a highly contagious pathogen
relationships relevant to this article. This
belonging to the Pneumoviridae family of negative-strand RNA viruses that oc- commentary does not contain a
curs in a somewhat predictable seasonal epidemic. Although most infants ex- discussion of an unapproved/
perience mild, coldlike symptoms, some infants, particularly with a primary investigative use of a commercial
product/device.
infection in the first year after birth, can experience more severe symptoms
that often require urgent medical attention. Infants with chronic lung disease Single-Dose Nirsevimab for Prevention of
(CLD), prematurity, or congenital heart disease are at particularly high risk for RSV in Preterm Infants. Griffin MP, Yuan Y,
Takas T, et al. N Engl J Med.
severe disease. Nonetheless, the most hospitalizations due to RSV occur in
2020;383(5):415–425
healthy term infants. Therefore, preventing RSV illness in all infants is a ma-
jor public health priority. New Strategies for the Prevention of
The FDA approved nirsevimab in July 2023 for the prevention of RSV LRTI Respiratory Syncytial Virus (RSV). Messina
A, Germano C, Avellis V, et al. Early Hum
in neonates and infants born during or entering their first RSV season and in
Dev. 2022;174:105666
children up to 24 months of age who remain vulnerable to severe RSV disease
through their second RSV season. Nirsevimab is a recombinant human immu- FDA Approves New Drug to Prevent RSV
noglobulin G1 kappa monoclonal antibody. Nirsevimab binds the F1 and F2 sub- in Babies and Toddlers. FDA news release.
Available at: https://www.fda.gov/news-
units of the RSV fusion (F) protein. The F protein is necessary for cell infection events/press-announcements/fda-approves-
and remains a main target for vaccines and monoclonal antibodies, as the F pro- new-drug-prevent-rsv-babies-and-toddlers.
tein mediates the virus entry process by binding to the cell and enabling fusion Published July 17, 2023. Accessed August 21,
2023
of the virus envelope with the host cell plasma membrane. By binding the F1
and F2 subunits of the F protein at a highly conserved epitope, nirsevimab locks Nirsevimab: First Approval. Keam SJ.
the RSV F protein in the prefusion conformation to block viral entry into the Drugs. 2023;83(2):181–187
host cell. The enhanced neutralizing activity of nirsevimab and a modification of
Use of Nirsevimab for the Prevention of
the Fc region of this monoclonal antibody to promote extension of the half-life
Respiratory Syncytial Virus Disease
support a vaccinelike strategy to protect infants from RSV with a single intramuscular Among Infants and Young Children:
dose administered once per RSV season, which typically spans 5 months. Recommendations of the Advisory
Griffin et al demonstrated that nirsevimab provided healthy newborns of at Committee on Immunization Practices—
United States, 2023. Jones JM, Fleming-
least 35 weeks’ gestational age a stark reduction of RSV-associated LRTI that re- Dutra KE, Prill MM, et al. MMWR Morb
quired medical attention with an efficacy of 74.5% (95% confidence interval [CI], Mortal Wkly Rep. 2023;72(34):920–925

Vol. 44 No. 12 D E C E M B E R 2 0 2 3 723

Downloaded from /pediatricsinreview/issue/44/12


by Hospital de Peditría 'Prof. Dr. Juan P. Garrahan' user
49.6% to 87.1%; P < .001) and a reduction of hospitaliza- with cystic fibrosis who have manifestations of severe lung
tions for RSV-associated LRTI with an efficacy of 62.1% disease or weight-for-length below the 10th percentile,
(95% CI, 8.6% to 86.8%; P 5 .07) from the start of the American Indian children, and Alaskan native children.
RSV season, including those born out of the RSV season. Infants born shortly before or during the RSV season
This single intramuscular injection provides this extended should receive nirsevimab in their first week of life either in
benefit over 5 months without the need for serial, monthly the hospital or in an outpatient setting. Newborns with a pro-
injections. longed hospital stay should receive nirsevimab shortly before
The safety and efficacy of nirsevimab has been evalu- or after discharge. For other infants and eligible toddlers, nir-
ated through 3 clinical trials (Trials 03, 04, and 05). The sevimab should be administered shortly before the beginning
primary outcome measure was the number of participants of RSV season. Standard-risk children who did not receive a
with medically attended RSV LRTI evaluated during the dose at the start of the season can receive a dose at any time
150 days after nirsevimab administration. Medically at- during the season. High-risk children should get a dose in
tended RSV LRTI included all health-care provider visits both their first and second seasons, even if they are younger
(physician’s office, urgent care, and emergency depart- than 8 months when entering their second season. Children
ment visits and hospitalizations) for LRTI with worsening who received palivizumab in their first season can get nirsevi-
clinical severity and a positive RSV test result. These ran- mab in their second season if they meet eligibility. Children
domized, double-blind, placebo-controlled (Trials 03 and 04) who receive nirsevimab should not receive palivizumab later
and active (palivizumab)-controlled (Trial 05) multicenter
that season. Although the typical RSV season is October
clinical trials provided evidence for the use of nirsevimab to
through March, in tropical climates and Alaska, where the
prevent medically attended RSV LRTI in preterm, late pre-
RSV season may be different than the rest of the United
term, and term infants who were entering their first RSV
States, clinicians should consult state, local, or territorial guid-
season at the time of screening along with high-risk infants
ance on timing.
eligible to receive palivizumab entering their first or second
Nirsevimab has been compared with palivizumab, a spe-
RSV season.
cific RSV immunoglobulin G administered in 5 monthly in-
Noted possible adverse effects of nirsevimab include
jections with license limited to infants who are at highest risk
rash and injection site reactions. Nirsevimab comes with
for severe disease, namely, infants who are born at 35 weeks
precautions about serious hypersensitivity reactions, in-
or less of gestational age or children younger than 2 years
cluding anaphylaxis, that have been observed with other
with CLD or hemodynamically significant congenital heart
human IgG1 monoclonal antibodies. Nirsevimab should
disease. Nirsevimab has greater neutralizing activity and a lon-
be given with caution to infants and children with clini-
ger serum half-life (63- to 73-day half-life of nirsevimab com-
cally significant bleeding disorders. In clinical trials, there
pared with 19- to 27-day half-life of palivizumab) without the
was no increased incidence of serious adverse events
among infants randomized to taking nirsevimab com- need for monthly injections. In addition, nirsevimab has been
pared with control patients. evaluated in healthy late-preterm and term infants, with fewer
Nirsevimab is recommended by the American Academy limitations on eligible cohorts. Furthermore, additional targets
of Pediatrics (AAP) and approved by the FDA and the Ad- for RSV prevention are under investigation, including vac-
visory Committee on Immunization Practices (ACIP). The cines, oral RSV F protein fusion inhibitors, and other fusion
ACIP to the Centers for Disease Control and Prevention antibodies, that if successful, may provide additional preven-
(CDC), which includes representatives from the AAP and tive options and potentially lower costs.
the American Academy of Family Physicians, recom- The AAP has stated that equitable access to nirsevimab is of
mends that all infants younger than 8 months born dur- the highest priority, particularly because nirsevimab can be
ing or entering their first RSV season receive nirsevimab offered to otherwise healthy infants rather than to only those
to protect them from RSV. Nirsevimab is also recom- affected by potential risk factors. As with all new preventive treat-
mended for children 8 months through 19 months who ments administered to healthy children, caregivers may require
are at increased risk for severe RSV disease and are enter- additional counseling to understand nirsevimab’s indications
ing their second RSV season. This includes children with and to gain confidence in its benefits to infants. Clinicians
CLD of prematurity who require medical support during should be prepared with handouts and links to reputable web-
the 6 months before the start of the second RSV season, sites, such as from the AAP and the CDC, to aid caregivers
children who are severely immunocompromised, children in their decision-making.

724 Pediatrics in Review

Downloaded from /pediatricsinreview/issue/44/12


by Hospital de Peditría 'Prof. Dr. Juan P. Garrahan' user
VISUAL DIAGNOSIS

Exfoliative Erythematous Rash in an 11-day-old


Preterm Infant
Janani Rajkumar, MBBS,* Jamie B. Warren, MD, MPH,† Jina Park, MD†
*Department of Pediatrics and †Division of Neonatal-Perinatal Medicine, Department of Pediatrics, Oregon Health & Science University, Portland, OR

PRESENTATION
A male neonate is born prematurely via cesarean delivery at 30 weeks and 2 days of
gestation to a 29-year-old gravida 2 para 1 woman. The mother was diagnosed
as having esophageal cancer during the pregnancy, and the premature delivery
was elective so that she could begin chemotherapy. Routine prenatal fetal sonograms
were normal, and prenatal laboratory tests were remarkable only for unknown
gonorrhea and chlamydia status.
The neonate required intubation at birth for respiratory distress syndrome
and need for surfactant. He was extubated to continuous positive airway pressure
(CPAP) the following day. The following day, he developed emesis and increased
work of breathing. Abdominal and chest radiographs were notable for bowel loop
dilation and diffuse moderate ground glass opacity and findings of pulmonary inter-
stitial emphysema. A blood culture was obtained, and he was started on ampicillin
and gentamicin. He completed 36 hours of antibiotic therapy; the blood culture
remained negative, and emesis improved. At 4 days of age, the neonate was rein-
tubated for ongoing increased respiratory distress, then extubated again to CPAP
on day 6. At 11 days of age, he is noted to have skin breakdown around the nasal
CPAP prongs and cannula adhesive. On physical examination he is noted to have
superficial peeling skin erosions with erythematous base and irregular borders
over the nasal bridge, malar cheeks, and left upper eyelid (Fig 1). He also has a
similar-appearing 1-cm erosion in the right axilla with a yellow-tinged exudate at
the base (Fig 2). The rest of his examination findings and vital signs are normal.
During the next 12 hours, additional skin lesions appear around the umbilicus,
gluteal cleft, left hand (Fig 3), chest (Fig 4), and scrotum. Nasal polymerase
Dr Rajkumar’s current affiliation is
chain reaction for methicillin-sensitive (MSSA) and methicillin-resistant (MRSA) Department of Neonatal and Pulmonary
Staphylococcus aureus and superficial swabs for bacterial culture from facial lesions Medicine, Cincinnati Children’s Hospital,
and oral, umbilical, and anal regions are obtained. A new blood culture is also Cincinnati, OH.

obtained. The clinical pattern confirms the diagnosis. Dr Park’s current affiliation is Division of
Neonatology, Department of Pediatrics,
Kaiser Permanente Northern California,
DIAGNOSIS Walnut Creek, CA.
The differential diagnosis for exfoliative skin lesions includes bullous impetigo, epi-
AUTHOR DISCLOSURE: Drs Rajkumar,
dermolysis bullosa, bullous mastocytosis, neonatal pemphigus, contact dermatitis, Peterson, and Park have disclosed no
chemical skin burns, and toxic epidermal necrolysis/Stevens-Johnsons syndrome. financial relationships relevant to this
The infant in this case was diagnosed as having staphylococcal scalded skin syn- article. This commentary does not
contain a discussion of an unapproved/
drome (SSSS) based on the clinical appearance of the lesions. In particular, the ery- investigative use of a commercial
thematous blanching macular rash followed by the classic exfoliative erythematous product/device.

Vol. 44 No. 12 D E C E M B E R 2 0 2 3 e33

Downloaded from /pediatricsinreview/issue/44/12


by Hospital de Peditría 'Prof. Dr. Juan P. Garrahan' user
Figure 3 Superficial peeling skin erosion with erythematous base on left
hand.
Figure 1 Superficial peeling skin erosions with erythematous base and ir-
regular borders over nasal bridge, malar cheeks, and left upper eyelid. Certain strains of S aureus release epidermolytic exotoxins A
and B. (1). These toxins spread systemically and function as
rash with skin peeling is consistent with SSSS. In addition, proteases that target desmoglein 1, an intercellular attachment
SSSS usually involves the face (mouth and nose) first and protein found in the zona glomerulosa of the epidermis. (2).
then spreads to the axilla, groin, abdomen, umbilicus, and Approximately 5% of S aureus strains produce epidermolytic
trunk, sparing the mucous membranes. The rash in this case exfoliative exotoxins. Exotoxin A makes up approximately 80%
evolved in this pattern. of exotoxins produced. (3)(4).
SSSS primarily affects children and neonates. This is
DISCUSSION thought to be due to the immature immune system and a
The characteristic finding of SSSS is an exfoliative erythem- lower renal clearance capacity of exfoliative toxins in these
atous rash with desquamation. It is caused by exotoxins age groups. (5). In addition, the composition of the skin is
released by S aureus, a gram-positive bacterial organism. variable and the proportion of desmoglein-1 in keratinocytes

Figure 2 Right axillary skin erosion 1 cm in size with a yellow-tinged exu- Figure 4 Superficial peeling skin erosion with erythematous base located
date at the base. on the chest and abdomen.

e34 Pediatrics in Review

Downloaded from /pediatricsinreview/issue/44/12


by Hospital de Peditría 'Prof. Dr. Juan P. Garrahan' user
decreases with increasing age, which results in a greater a high prevalence of MRSA. (2)(3)(4)(5) The lesions are
number of intercellular attachment protein targets for the dressed with emollients and nonstick dressings to assist
exfoliative exotoxins in younger individuals. (6). with healing and reduce fluid losses. Other components of
The initial finding is usually an erythematous blanching care include pain management, adequate nutrition, and
macular rash. This is followed by the classic exfoliative ery- monitoring and management of fluid and electrolyte imbal-
thematous rash with skin peeling. (3). In some cases, friction ances. (5) There are typically no long-term sequelae, and re-
with the skin results in flaccid bullae, which break to form covery occurs over 2 to 3 weeks without significant scarring
exfoliated erythematous areas when rubbed (Nikolsky sign). or disfigurement. Occasionally, it is associated with postin-
(2)(7). Lesions usually start in the face (mouth and nose) and flammatory hypopigmentation. Complications of extensive
then spread to the axillae, groin, abdomen, umbilicus, and skin involvement include sepsis and fluid and electrolyte imbal-
trunk. Mucous membranes are characteristically spared. ances. Mortality varies from 3% to 11% in children. (3)
A complete blood cell count may show elevated or SSSS has been associated with outbreaks in nurseries and
normal white blood cell counts. Erythrocyte sedimenta- NICUs. These outbreaks are usually spread by asymptomatic
tion rate and C-reactive protein level may be elevated. carriers to susceptible individuals. Contact isolation precau-
Electrolytes may show fluid/electrolyte imbalances due tions are a key factor in containing outbreaks. (13)(14)(15)
to increased insensible losses through the damaged skin
barrier. (6). Skin cultures from the lesions and blood PATIENT COURSE
cultures may be positive for growth of S aureus. The sub- Initially, the patient was placed under contact and droplet
typing of S aureus and determining exotoxin production is isolation precautions. Treatment was initiated with nafcillin.
usually not necessary. (8) Dermatology and pediatric infectious diseases were consulted.
The diagnosis of SSSS is based on the clinical appear- To avoid further irritation from nasal CPAP prongs, the in-
ance of the rash. Bullous impetigo, epidermolysis bullosa, fant was transitioned to high-flow nasal cannula for respira-
bullous mastocytosis, neonatal pemphigus, contact derma- tory support. Pain was managed with as-needed enteral
titis, chemical skin burns, and toxic epidermal necrolysis/ acetaminophen, intravenous morphine, and enteral gaba-
Stevens-Johnson syndrome may present with similar skin pentin. Within 12 hours of skin lesion onset, the skin le-
lesions, but there are important differences. Lesions in sions spread to the perineum, scrotum, lip, and left foot.
epidermolysis bullosa form at sites of friction against the Given the progression of the skin lesions, enteral clindamy-
skin and can also involve the oral cavity and eye. (9) Neona- cin was added to the antibiotic regimen to cover for MRSA.
tal pemphigoid is an autoimmune dermatologic condition The nasal swab MSSA/MRSA polymerase chain reac-
and is differentiated from SSSS by a negative Nikolsky sign. tion was positive for MSSA. There was no growth in the
(10) Chemical skin burns and contact dermatitis are differ- cultures from the blood or the oral cavity, but the cultures
entiated based on exposure history. The lesions in bullous from the facial lesions were positive for S aureus resistant to
mastocytosis are recurrent, (11) but the recurrence of SSSS clindamycin and erythromycin. Therefore, clindamycin was
is rare. (12) In toxic epidermal necrolysis, there is involve- discontinued. After receiving a 2-day course of nafcillin,
ment of mucous membranes, but SSSS rarely involves antibiotics were narrowed to cefazolin. After receiving cefa-
the mucous membranes. In addition, toxic epidermal nec- zolin for 48 hours, the infant was transitioned to enteral
rolysis can be differentiated from epidermolysis bullosa by cephalexin to complete a total 12-day course of antibiotics.
widespread involvement of the skin, with no clear topical
trigger. In SSSS, the culture of fluid may or may not have
bacterial growth. (13) However, bacteria or toxins are usually
found in the bullous lesions, and S aureus can be cultured
from the bullous fluid of bullous impetigo. Early initiation
of therapy with empirical antistaphylococcal antibiotics
such as nafcillin or oxacillin is essential until cultures with
sensitivity are available to narrow the therapy. If a patient
has an allergy to penicillins, clarithromycin or cefuroxime
may be used. Vancomycin should be considered in patients
with extensive skin involvement, critical illness, lack of
clinical improvement despite antibiotics, or when there is Figure 5 Resolving skin lesions.

Vol. 44 No. 12 D E C E M B E R 2 0 2 3 e35

Downloaded from /pediatricsinreview/issue/44/12


by Hospital de Peditría 'Prof. Dr. Juan P. Garrahan' user
Emollients were applied generously to the lesions and covered
• The characteristic finding of SSSS is an exfoliative
by nonadherent dressings.
erythematous rash with desquamation. The initial
Forty-eight hours after the initiation of antibiotics, the le-
finding is usually an erythematous blanching macular
sions began to improve and stabilize. Lesions healed completely
rash. This is followed by the classic exfoliative
(Fig 5) within 7 days of onset (18 days of age). He did not
have recurrence of the skin lesions throughout the rest of his erythematous rash with skin peeling.
hospitalization (1 month). The infant’s initial symptoms (vom- • SSSS is caused by exotoxins released by Staphylococcus
iting, respiratory distress) are not traditionally associated with aureus, a gram-positive bacterial organism.
SSSS. In addition, there were no other known risk factors for
• Early initiation of therapy with empirical anti-
SSSS in this case. There were no other infants in the unit
staphylococcal antibiotics such as nafcillin or oxacillin
with similar infections, and no source was identified.
is essential until cultures with sensitivity are available
to narrow the therapy.
Summary
• Staphylococcal scalded skin syndrome (SSSS)
primarily affects children and neonates.
References for this article can be found at
https://doi.org/10.1542/pir.2022-005739.

e36 Pediatrics in Review

Downloaded from /pediatricsinreview/issue/44/12


by Hospital de Peditría 'Prof. Dr. Juan P. Garrahan' user

You might also like